Tsopméné P Matrix Algebra 2022
Tsopméné P Matrix Algebra 2022
Exercise Book
Practice Problems
Concept Summaries
Very Detailed Solution To Each Problem
by Paul Tsopméné
Source File. To get the source file, please get in touch with the author
at the following email address: paul.tsopmene@ubc.ca
iii
For Instructors
Dear instructors,
Thank you for your interest in this open exercise book. I hope you find it a
valuable addition to your existing teaching material.
I would love to hear your feedback and adoption decision for your course.
Your feedback will be highly appreciated and will be used for future im-
provement of the book.
Please take a few minutes to fill in the survey, which can be accessed in the
following way:
Survey Link
Thank you for your time and support on this open exercise book develop-
ment.
Best Regards,
Paul Tsopméné
Preface vii
I Exercises 1
1 Systems of Linear Equations 3
1.1 Introduction to Systems of Linear Equations . . . . . . . . . . . . . . . . . 3
1.2 Echelon Matrices . . . . . . . . . . . . . . . . . . . . . . . . . . . . . . . . 4
1.3 Solving Linear Systems Using Matrices . . . . . . . . . . . . . . . . . . . . 8
3 Linear Transformations 25
3.1 Introduction to Linear Transformations . . . . . . . . . . . . . . . . . . . . 25
3.2 The Matrix of a Linear Transformation . . . . . . . . . . . . . . . . . . . . 28
3.3 One-To-One and Onto Linear Transformations . . . . . . . . . . . . . . . . 31
4 Matrix Algebra 35
4.1 Matrix Operations . . . . . . . . . . . . . . . . . . . . . . . . . . . . . . . 35
4.2 The Inverse of a Matrix . . . . . . . . . . . . . . . . . . . . . . . . . . . . 40
5 Subspaces of Rn 45
5.1 Subspaces, Column and Null Spaces . . . . . . . . . . . . . . . . . . . . . . 45
5.2 Basis, Dimension, and Rank . . . . . . . . . . . . . . . . . . . . . . . . . . 50
6 Determinants 55
6.1 Introduction to Determinants . . . . . . . . . . . . . . . . . . . . . . . . . 55
6.2 Properties of Determinants . . . . . . . . . . . . . . . . . . . . . . . . . . . 57
v
vi Contents
II Solutions to Exercises 67
1 Systems of Linear Equations 69
1.1 Introduction to Systems of Linear Equations . . . . . . . . . . . . . . . . . 69
1.2 Echelon Matrices . . . . . . . . . . . . . . . . . . . . . . . . . . . . . . . . 76
1.3 Solving Linear Systems Using Matrices . . . . . . . . . . . . . . . . . . . . 95
5 Subspaces of Rn 245
5.1 Subspaces, Column and Null Spaces . . . . . . . . . . . . . . . . . . . . . . 245
5.2 Basis, Dimension, and Rank . . . . . . . . . . . . . . . . . . . . . . . . . . 261
6 Determinants 285
6.1 Introduction to Determinants . . . . . . . . . . . . . . . . . . . . . . . . . 285
6.2 Properties of Determinants . . . . . . . . . . . . . . . . . . . . . . . . . . . 294
The author wrote this exercise book when teaching MATH 221 (Matrix Algebra) at the
University of British Columbia Okanagan. The book is for students taking a first course
in linear algebra.
Many students learn math by studying examples/practice problems. While going
through the solution to a problem, students are often faced with several issues. They
may not see the connection between the concept/theory taught in class and the solution.
Others may not understand the solution because a step is missing or there are not enough
explanations.
My main goal in this book is to address these issues to help students learn more effi-
ciently and get better results. To that end, I have included the following features.
Concept Summaries Boxes: They allow students to review the material and make
the book self-contained. Each concept involved in a problem is summarized in a box
right before the solution. Just the relevant information needed to solve the problem
is given. And the solution clearly shows how that information is used.
The book is divided into two parts. The first part includes a variety of problems. The
idea is not to have as many problems as in a traditional textbook but a fair amount of
problems that cover essential/basic skills in matrix algebra. The second part includes full
solutions to every problem. Hyperlinks are added to ease the navigation of the book
(they are mainly used to go back and forth between the two parts).
To the Student: If you try to solve a problem and have no idea what to do, you can look
at the solution and try to understand it (if there is a box, you should first try to read and
understand its content). Try to solve the subsequent few problems yourself before checking
the solution. It’s not a big deal if you make a mistake; you will learn better from that
mistake.
vii
viii
PartI
Exercises
1
Chapter1
2. For each equation, three points are given. Determine which one is a solution to the
equation. [Solution on page 70]
(a) 2x1 + 3x2 = 14 Points: (3, 3), (1, 4), and (5, 1).
(b) −4x1 − 5x2 + 21x3 = 7 Points: (−2, 3, 0), (−2, −3, 0), and (2, −3, 0).
4. For each system, determine whether the given point is a solution to the system.
[Solution on page 71]
x1 − 2x2 = 3
(a) (1, −1)
3x1 + x2 = 2
3
4 Chapter 1. Systems of Linear Equations
−3x1 + 4x2 = 18
(b) (−2, 3)
5x1 − 6x2 = −27
x1 − x2 + x3 = 3
(c) 2x1 + 3x2 − x3 = −5 (0, −1, 2)
−3x1 + 4x2 + 2x3 = 1
x1 + 2x3 − x4 = 7
(d) 2x1 + x2 − 4x3 = −9 (2, −1, 3, 1)
− 5x2 + 10x4 = 15
6. Find a linear system in two variables with two different equations that satisfies the
following two conditions: [Solution on page 75]
Every coefficient is a nonzero number.
The point (5, −7) is a solution to the system.
7. Find a linear system in three variables with three different equations that satisfies
the following two conditions: [Solution on page 75]
Every coefficient is a nonzero number.
The point (−2, 3, −5) is a solution to the system.
8. Suppose we have a linear system in three variables with two equations. Also suppose
that the system is consistent. Is it possible to find a point (a, b) that satisfies the
system? Why or why not? [Solution on page 76]
Zero Row? (Yes or No) Nonzero Row? (Yes or No) Leading Entry
Row 1
Row 2
Row 3
Row 4
1.2. Echelon Matrices 5
3 1 4 0 0 0 0 0 0 0
0 2 0 0 0 8 0 0 0 5
(a) (b)
0 0 0 0 0 0 0 −7 7 9
0 0 0 1 0 0 0 0 0 0
3. Is it possible to find a matrix in echelon form which is not in reduced echelon form?
Justify your answer. [Solution on page 78]
4. Consider the matrices below. Determine which ones are in echelon form. Also deter-
mine which matrices are in reduced echelon form. [Solution on
page 79]
1 0 0 1 0 0 1
(a) 0 1 0 (e) 0 0 5 0
0 0 1 0 0 0 0
1 0 0 0 1 0 0 0
(b) 0 0 1 0 (f ) 0 0 0 0
1 0 0 0 0 0 0 1
1 0 1 0 1 7 0 −2 0
(c) 0 0 1 0 (g) 0 0 1 8 0
0 0 0 1 0 0 0 0 1
0 1 0 0 4 1 0 −5 1 0
(d) 0 0 0 1 6 (h) 0 1 0 0 0
0 0 0 0 0 0 0 0 1 0
7. In each case, find the single elementary row operation that was performed on A to
get B. [Solution on page 82]
1 2 1 2
(a) A = B=
−2 4 0 8
−1 3 0 −1 3 −1
(b) A = 1 −2 −3 B = 1 −2 −3
−1 3 −1 −1 3 0
−2 3 −1 −2 3 −1
(c) A = 0 2 −2 B = 0 2 −2
2 −3 −1 −2 3 1
3 5 −4 1 5 2
(d) A = −1 0 3 B = −1 0 3
1 5 2 1 5 2
3 −12 −2 3 −12 −2
(e) A = 2 −3 −7 B = −2 13 −3
1 −4 −1 1 −4 −1
1 1 2 1 1 2
1 4 2 R2 − R3 → R2 −2 −1 −4
R3 − R2 → R3
3 5 6 2 1 4
9. Find the pivots, pivot positions, and pivot columns of each matrix. [Solution on
page 85]
1 4 7 −3 4 0
(a) 0 9 2 (b) 0 8 2
0 0 5 0 0 0
1.2. Echelon Matrices 7
0 4 0 1 1 0 2 0
(c)
0 0 1 13 0 0 0 1
(d)
0
0 0 0
0 0 0 0
10. (True or False) Let A be a matrix. Then it is possible to find two echelon forms of
A that are not the same. [Solution on page 86]
11. (True or False) Let A be a matrix. Then it is possible to find two reduced echelon
forms of A that are not the same. [Solution on page 87]
12. (True or False) Let A be a matrix. Let B and C be two echelon forms of A. Then
B and C have the same pivots. [Solution on page 89]
13. (True or False) Let A be a matrix. Let B and C be two echelon forms of A. Then
B and C have the same pivot positions. [Solution on page 89]
14. In each case, show that A and B are row equivalent. (In other words, show that B is
an echelon form of A.) [Solution on page 89]
1 0 2 1 0 2
(a) A= 1 3 3 B= 0 3 1
−1 3 3 0 0 4
1 −2 4 1 −2 4
(b) A = 3 −7 15 B = 0 −1 3
1 0 3 0 0 1
0 2 1 1 1 3
(c) A = −3 1 −2 B= 0 4 7
1 1 3 0 0 −5
1 −6 −4 1 1 0 0 1
(d) A= 0 2 3 5 B = 0 1 0 −2
0 0 1 3 0 0 1 3
15. Row reduce the following matrices to reduced echelon form.[Solution on page 91]
1 2 3 4 −3 1 1 −1
(a) 1 3 1 1 (d) 4 −2 −2 0
2 7 1 4 5 0 −3 −1
3 −5 13 −2 1 2 3 −2 7
(b) −2 2 −12 14 1 2 3 −2 6
(e)
1 −2 3 1 1 2 3 −1 4
−1 −2 −4 3 −2
3 9 0 −3
−1 −2 2 −2 0 4 −3 −1
(c)
4 11 −1 3 (f ) −3 1 1 2
3 9 0 −3 −2 −4 2 −4
8 Chapter 1. Systems of Linear Equations
3x1 + 4x2 = 5
(a)
6x1 − 7x2 = 8
−x1 − 3x2 + x3 = −2
(b) x1 − x2 − 4x3 = 0
−5x1 + x3 = 6
x2 − x3 + x4 = 1
(c) x1 + 4x2 − 5x3 + 13x4 = −9
−2x1 − x4 = 6
3. (True or False) Two linear systems are equivalent if and only if they have the same
set of solutions. [Solution on page 97]
4. (True or False) Two linear systems are equivalent if and only if the corresponding
augmented matrices are not row equivalent. [Solution on page 97]
x1 = 5 x1 − x2 + x3 = 3
(a)
x2 = 7 (c) 5x2 − 3x3 = 0
11x3 = 55
x1 + x2 = 7
(b)
3x2 = 12
x1 + 2x2 = 5 x1 + x2 = 0
(a) (b)
2x1 − x2 = −10 3x1 − 4x2 = 7
2x1 − 3x2 = 7
(c)
5x1 + 7x2 = 3
x1 − 2x2 − x3 = −6
(a) −x1 + 3x2 + 2x3 = 11
−2x1 + 5x2 + 4x3 = 20
x1 − x2 + x3 = 2
(b) 2x1 + x2 = −4
−3x1 + 2x3 = 14
x1 + x2 + x4 = 4
x1 + x3 = 1
(c)
2x2 − x3 + x4 = 4
2x1 − x2 + x3 − 3x4 = −5
x1 + x2 + 2x3 = 3 x1 − 2x2 + x3 = 0
(a) 3x1 + 4x2 + 8x3 = 5 (b) x1 − x2 + 4x3 = 1
− x2 − 2x3 = 2 −x1 + 2x2 − x3 = 5
9. Identify the leading variables and the free variables of each system. [Solution on
page 102]
3x1 + x2 − x3 = 4
(a)
5x2 − 2x3 = −8
x1 − x2 − x3 = 1
(b) −x2 + x3 = 7
2x3 = 5
x1 + 5x2 − 7x3 + x4 = −9
(c)
7x3 − x4 = 9
10. Determine whether each statement is True or False. [Solution on page 103]
(a) If all the variables of a linear system are leading variables, then the system has
a unique solution.
(b) A linear system that has at least one free variable must be consistent.
x1 − 2x2 + 4x3 = 5
(a) −x1 + 3x2 − 2x3 = −2
3x1 − 4x2 + 16x3 = 21
x1 + 3x2 + 2x3 = 7
(b) 2x1 + 7x2 + 3x3 = 10
−4x1 − 13x2 − 7x3 = −24
x1 + x2 + 2x3 = 1
(c) x1 + 2x2 + 2x3 = 4
−x1 − 3x2 − 2x3 = −2
x1 − 2x2 − x4 = 2
(d) 2x1 − 4x2 + x3 − 5x4 = 6
x1 − 2x2 − x3 + 2x4 = 0
x1 = 2+s−t
x2 = −3 + 7t
s and t arbitrary
x3 = s
x4 = t
Write down a linear system of two equations in four variables that has this general
solution. [Solution on page 107]
x1 + x2 − x3 = a
x1 + 2x3 = b
−x1 − 2x2 + 4x3 = c
Find conditions on a and b such that the system has no solution, one solution, or
infinitely many solutions.
Chapter2
11
12 Chapter 2. Vectors in Rn and Linear Systems
0 −2 5 10
(b) x1 + x2 + x3 =
1 1 0 12
5. For each system, write the corresponding vector equation. [Solution on page 118]
x 1 − x2 = 3
(a)
2x1 + 7x2 = −8
2x1 − x2 + x3 = 5
(b) −3x1 + 4x2 + 5x3 = 9
8x1 + 6x3 = 11
x1 + 2x2 = 4
−4x1 − 3x2 = −1
1 4 1
(b) a1 = −2 , a2 = −11 , b= 1
3 −1 29
2.1. Vector Equations and Span 13
1 2 0 −10
10. Let A = 1 1 3 and b = 12 . Determine whether b is a linear combina-
0 −1 4 27
tion of the columns of A. [Solution on
page 122]
11. In each case, find three different vectors in Span{a1 , a2 }. [Solution on page 123]
1 −2 −1 −4
(a) a1 = , a2 =
2 3 (b) a1 = 0 , a2 = 3
2 6
−1 4 3
12. Let a1 = 2 , a2 = −5 , and b = −3 . Determine whether b is in
3 0 4
Span{a1 , a2 }. [Solution on page 123]
Span{v1 , v2 } = Span{v1 , v2 , u}
18. (True or False). It is possible to find two vectors, v1 and v2 , of R3 such that
Span{v1 , v2 } = R3
2. (True or False). If A is of size 5×8, then the vector b in the matrix equation Ax = b
has 8 entries. [Solution on page 128]
3. In each case, write the system as a matrix equation. [Solution on page 128]
7x1 − 6x2 = 13
(a)
−8x1 + 9x2 = 5
x1 + x2 − x3 = 0
(b) 2x1 + x3 = 1
3x2 − 4x3 = 7
4. Given A, x, and b, write the linear system corresponding to the matrix equation
Ax = b. Also write the vector equation corresponding to Ax = b. [Solution on
page 128]
1 3 −4 x1 0
(a) A = −5 2 1 , x = x2 , b = −3
6 −1 −17 x3 8
x1
0 0 −2 3 1 x2 4
2 −1 3 0 4
, x = x3 , b = 23
(b) A = 1 1 12 0 0
x4
0
−3 −2 0 −9 9 32
x5
5. Let
4 −10 10 x1 38
A = 3 −5 15 , x = x2 , b = 16
3 −6 12 x3 21
[Solution on page 129]
2.2. The Matrix Equation Ax = b 15
(a) Show that the system Ax = b does not have a solution for all possible b.
(b) Give a geometric description of the set of all b for which the system Ax = b does
have a solution.
1 −2 −5 b1
7. Let A = 2 4 22 and b = b2 . [Solution on page 132]
−3 5 11 b3
(a) Show that the system Ax = b does not have a solution for all possible b.
(b) Describe the set of all b for which the system Ax = b does have a solution.
(a) Reduce A to an echelon form, and find the number of pivot positions of A.
(b) Does the equation Ax = b have a solution for every b in R4 ?
(c) Find the number of pivot positions of B.
(d) Does the columns of B span R4 ?
(e) Is it possible to find a vector in R4 which is not a linear combination of the
columns of A?
1 2 3 b1
11. Let A = 2 3 5 and b = b2 . [Solution on page 134]
−1 4 4 b3
12. In each case, compute the product Ax using the dot product rule. [Solution on
page 135]
16 Chapter 2. Vectors in Rn and Linear Systems
1 3 5
(a) A = , x=
2 4 6
3
−1 0 −3
(b) A = , x= 4
2 4 −5
5
1 −1
7
(c) A = 2 −3 , x=
−2
3 0
x1 = 3t
(a) x2 = −4t t is arbitrary
x3 = t
x1 = 5 + 6t
(b) x2 = 10 − 8t t is arbitrary
x3 = t
x1 = 1 + s + 9t
x2 = −4 + 13t
(c) s and t are arbitrary
x3 = s
x4 = t
x1 = 4s − 7t
x2 = s
(d) x3 = −2 + 5t s and t are arbitrary
x4 = −3
x5 = t
2.3. Writing the General Solution of a Linear System in Vector Form 17
4. Describe the solution set to the equation Ax = 0 in vector form. Also find two
nontrivial solutions (if any). [Solution on page 139]
−3 5 1 2 −2
(a) A =
6 −9 (b) A = −1 −1 7
2 3 −9
0 1
8. (True or False). The solution set of Ax = b is the set of all vectors of the form
P +Vh , where P is a particular solution to Ax = b, and Vh is a solution to the equation
Ax = 0. [Solution on page 141]
(a) Solve the homogeneous system Ax = 0 and describe the solution set geometri-
cally (sketch a graph of the solution set).
18 Chapter 2. Vectors in Rn and Linear Systems
3
(b) Solve the system Ax = b, where b = , and describe the solution set
−12
geometrically (sketch a graph of the solution set).
x2
4
−3 4 x1
−3 L
(a) (True or False) The line L is the solution set to a nonhomogeneous linear
system.
(b) From the graph, find two nontrivial solutions to the linear system whose solution
set is L.
(c) Find a linear system with two equations and two variables whose solution set is
L.
be the augmented matrix of a system of linear equations. Write the general solution
in vector form.
12. Solve each system and write the general solution in vector form. [Solution on
page 145]
1 −1 2 x1 0
(a) Ax = b, where A = −1 4 −7 , x = x2 , b = 0
−1 10 −17 x3 0
x1 + 3x2 + 4x3 = 1
(b) −2x1 + x2 + 5x3 = 2
7x1 + 7x2 + 2x3 = −1
2.3. Writing the General Solution of a Linear System in Vector Form 19
x1 + 2x2 + x3 + 5x4 = −4
(c) 2x1 + 4x2 + 3x3 + 7x4 = 5
−3x1 − 6x2 − 5x3 − 9x4 = −14
1 −2 −3 4 −9
(d) x1 1 + x2 −1 + x3 −4 + x4 10 = 9
−2 3 7 −14 0
x1 − 2x2 + 3x3 + x4 = −3
−x1 + 2x2 − 2x3 + x4 = 5
2x1 − 4x2 + 5x3 = −8
14. A zero matrix is a matrix where every entry is 0. Suppose A is a 2 × 2 zero matrix.
Describe the solution set of Ax = 0. [Solution on page 149]
17. Find a linear system with two equations and three variables that has
3 −4
−1 + s 2
0 1
18. Find a linear system with three equations and four variables that has
2 −3 7
0
+ s 1 + t 0
5 0 −1
0 0 1
4 −12
(b) v1 = , v2 =
−5 15
0 1 2
(c) v1 = −1 , v2 = 0 , v3 = −3
1 1 5
2 4 0
(d) v1 = 3 , v2 =
0 , v3 = 7
0 −5 6
2. Determine if the statement is True or False, and justify your answer. [Solution on
page 155]
(a) If an echelon form of a matrix A has a pivot in every row, then the columns of
A are linearly independent.
(b) If an echelon form of a matrix A has a pivot in every column, then the columns
of A are linearly independent.
3. In each case, determine if the columns of the matrix form a linearly independent set.
[Solution on page 155]
−3 4 27
(a) 2 −1 −13
4 7 1
2.4. Linear Independence 21
1 3 −4
−1 −2 9
(b)
1 2 −8
3 11 −1
1 0 1
4. Let v1 = 2 , v2 = 1 , v3 = h .
[Solution on page 157]
3 1 4
6. Determine if the statement is True or False, and justify your answer. [Solution on
page 158]
8. Determine if the statement is True or False, and justify your answer. [Solution on
page 161]
(a) Let S be a set of vectors in Rn . If one of the vectors in S is the zero vector,
then S is linearly dependent.
(b) A linearly dependent set must contain the zero vector.
(c) Let {v1 , · · · , vp } be a set of vectors in Rn . If p > n, then the set {v1 , · · · , vp } is
linearly dependent.
(d) If a set {v1 , · · · , vp } of vectors in Rn is linearly dependent, then p > n.
22 Chapter 2. Vectors in Rn and Linear Systems
0
(b) v = , S = {v}.
0
1 1 7 1
(c) v1 = 7 , v2 = 0 , v3 = 8 , v4 = 1 , S = {v1 , v2 , v3 , v4 }.
6 8 9 2
−8 0 10
(d) v1 = −7 , v2 = 0 , v3 = 11 , S = {v1 , v2 , v3 }.
13 0 12
−2 4
4 −8
(e) v1 = , S = {v1 , v2 }.
6 , v2 = −12
10 −21
10. Consider the following vectors in R2 . Let 0 denote the zero vector. [Solution on
page 162]
4 v2
v1
v4
−2 4 x
−2 v3
11. Determine if the statement is True or False, and justify your answer. [Solution on
page 163]
2.4. Linear Independence 23
(a) If every entry in the bottom row of a matrix is 0, then the columns of A are
linearly dependent.
(b) If every entry in the rightmost column of a matrix A is 0, then the columns of
A are linearly dependent.
5
12. Let A = a1 a2 a3 be a 3 × 3 matrix and suppose that x = −2 is a solution
1
to the homogeneous equation Ax = 0. [Solution on page 163]
14. Let A be a 2 × 2 matrix with linearly dependent columns. Write out all possible
reduced echelon form of A. For entries that can have an arbitrary value, use an
asterisk ∗ symbol. [Solution on page 164]
15. Determine if the statement is True or False, and justify your answer. [Solution on
page 164]
(a) If the equation Ax = 0 has a nontrivial solution, then the columns of A are
linearly dependent.
(b) If the columns a1 , a2 , · · · , an of an m × n matrix A span Rm , then the set
{a1 , a2 , · · · , an } is linearly independent.
(c) If the columns of an m × n matrix A are linearly independent, then they span
Rm .
(d) Let A be an m × n matrix. If the columns of A are linearly independent, then
for every b in Rm the linear system Ax = b has a unique solution.
16. Let u, v be vectors in Rm such that the set {u, v} is linearly independent. Prove that
the set {u + v, u − v} is also linearly independent. [Solution on page 164]
24 Chapter 2. Vectors in Rn and Linear Systems
Chapter3
Linear Transformations
is linear.
is linear.
3. Determine if the statement is True or False, and justify your answer. [Solution on
page 169]
4. In each case, show that the transformation T is not linear. [Solution on page 171]
2 x1 √
(a) T : R → R defined by T = 3 x1 x2 .
x2
x1
x1
(b) T : R2 → R3 is defined by T = −x1 + x2 .
x2
5|x1 |
25
26 Chapter 3. Linear Transformations
2 2 x1 9x2 + 8
(c) T : R → R defined by T = .
x2 x1
(a) Show that there exists a unique pair (c1 , c2 ) such that v = c1 u1 + c2 u2 .
(b) Find T (v).
1 0 −1 3
10. Let e1 = , e2 = , v1 = , and v2 = . Let T : R2 → R2 be a
0 1 2 4
linear transformation that maps e1 into v1 and e2 into v2 . [Solution on page 174]
3.1. Introduction to Linear Transformations 27
12
(a) Find T .
−13
x1
(b) Find T (x) for an arbitrary x = .
x2
12. Determine if the statement is True or False, and justify your answer. [Solution on
page 176]
0 0
(c) A = , T : R2 → R2 is defined by T (x) = Ax.
0 1
0 1
(d) A = , T : R2 → R2 is defined by T (x) = Ax.
1 0
2. The unit square in R2 is the set of all points (x1 , x2 ) in R2 such that 0 ≤ x1 ≤ 1 and
0 ≤ x2 ≤ 1. In each case, sketch the graph of the image of the unit square under T .
[Solution on page 184]
3 0
(a) T (x) = Ax where A =
0 3
−2 0
(b) T (x) = Ax where A=
0 3
1 2
(c) T (x) = Ax where A=
0 1
1 0
(d) T (x) = Ax where A=
−3 1
−2 −1
(e) T (x) = Ax where A=
1 2
" #
cos π4 − sin π
4
(f ) T (x) = Ax where A=
sin π4 π
cos 4
3. In each case find the standard matrix of the linear transformation T . [Solution on
page 190]
2
3x1
(a) T : R → R is defined by T x1 =
−5x1
x1 7x1 − x2
(b) T : R2 → R2 is defined by T =
x2 3x2
x1
x 2 − 4x 3
(c) T : R3 → R2 is defined by T x2 =
−x1 + 6x2 − 9x3
x3
5x1
x1
(d) T : R2 → R3 is defined by T = 0
x2 x1 −x2
3
x1 x1 − 3x2 + 6x3
(e) T : R3 → R3 is defined by T x2 = −x1 + 4x2 − 7x3
x3 2x1 + 5x2 + x3
x2
T (e2 )
e2
e1
−2 5 x1
T (e1 )
−2
x2
T (e1 )
e2
T (e2 )
−2 e1 5 x1
−2
6. In each case, suppose that T is a linear transformation. Find the standard matrix of
T. [Solution on page 195]
π
(g) T : R2 → R2 first reflects points through the x1 -axis and then rotates points 2
(clockwise).
2. Determine if the statement is True or False, and justify your answer. [Solution on
page 200]
3. Determine if the statement is True or False, and justify your answer. [Solution on
page 202]
1
(a) Find an x in R3 whose image under T is u = 1 .
1
(b) Is T one-to-one? If not, find three different vectors in the domain that have the
same image.
(c) Is T onto? If not, find three different vectors in the codomain that are not in
the range of T . (The range of T is defined as the set of all vectors b in the
codomain for which the equation T (x) = b is consistent.)
(d) Describe geometrically the range of T .
(e) The kernel of T is the set of all vectors x in the domain such that T (x) = 0.
Describe geometrically the kernel of T .
34 Chapter 3. Linear Transformations
Chapter4
Matrix Algebra
35
36 Chapter 4. Matrix Algebra
(a) A + B (d) C + D + B
(b) C + D (e) 2C
(c) A + C (f ) 3C − 2D
4. Consider the matrices A and B from Exercise 2. Compute −5A + 2B − 3I2 . [Solution
on page 212]
5 −3 −2 8
5. Solve the the equation X + = −6 , where X is a 2 × 2 matrix.
1 6 1 0
[Solution on page 212]
6. Let α and β be two scalars (real numbers), and let A be a matrix. Prove that
(α + β)A = αA + βA. [Solution on page 213]
7. Determine if the statement is True or False, and justify your answer. [Solution on
page 213]
1 2 5 6
(h) If A = and B = , then
3 4 7 8
1(5) + 2(7) 3(5) + 4(7) 19 43
AB = =
1(6) + 2(8) 3(6) + 4(8) 22 50
1 2 5 6
(i) If A = and B = , then
3 4 7 8
1(5) + 2(7) 1(6) + 2(8) 19 22
AB = =
3(5) + 4(7) 3(6) + 4(8) 43 50
9. Let A and B be matrices such that A is of size 5 × 7 and the number of columns of
AB is 11. What is the size of B? [Solution on page 217]
10. Let A and B be matrices such that the product BA is of size 17 × 32. [Solution on
page 217]
(a) How many rows does B have?
(b) How many columns does A have?
11. Let A be an m × n matrix, and let B be an n × p matrix. Show that every column of
the product AB is a linear combination of the columns of A. [Solution on page 217]
1 2
12. Let A = . Let a1 and a2 be the columns of A (of course, a1 is the first column
3 4
and a2 is the second). Consider the following linear combinations. [Solution on
page 217]
0 2
2a1 − a2 = and − 4a1 + 3a2 =
2 0
0 2
Find a 2 × 2 matrix B such that AB = .
2 0
38 Chapter 4. Matrix Algebra
1 −2 5 5
13. Let A = . Find a 2 × 2 matrix B such that AB = . [Solution on
3 −1 5 5
page 218]
14. Suppose the third column of B is all zeros. What can you say about the third column
of AB? [Solution on page 219]
17. Let A and B be two matrices such that BA = Im , where Im is the m × m identity
matrix. [Solution on page 221]
(a) Show that the equation Ax = 0 has only the trivial solution.
(b) Explain why the number of rows of A is greater than or equal to the number of
columns.
(c) Is the linear transformation T (x) = Ax one-to-one? Why or why not?
18. Determine if the statement is True or False, and justify your answer. In the following,
A, B, and C are arbitrary matrices such that the indicated products and sums are
defined. [Solution on page 221]
(a) AB = BA
(b) AC + BC = (A + B)C
(c) AB + CA = A(B + C)
(d) A(BC) = (AC)B
(e) If AB = AC, then B = C
(f ) If AB = 0, then A = 0 or B = 0
(g) If A = 0 or B = 0, then AB = 0
4.1. Matrix Operations 39
19. Let A, B, and C be n × n matrices. Assume that A and B commute. Also assume
that A and C commute. Show that A commutes with BC. [Solution on page 222]
0 −1 2
1 2
20. Let A = and B = 3 1 0 . Compute A2 and B 3 . [Solution on
3 4
−2 0 1
page 223]
−1
21. If A = −2 3 5 , B = 2 , compute A2 , AB, BA, and B 2 when they are
4
defined. [Solution on page 223]
a 0
22. Let A = be a 2 × 2 diagonal matrix. [Solution on page 224]
0 b
Find AT , B T , C T , and DT .
4 −1 2
24. Let A = . Compute AAT − 5I2 and 8I3 − AT A. [Solution on
−2 1 3
page 225]
25. Determine if the statement is True or False, and justify your answer. In the following,
A, B, and C are arbitrary matrices such that the indicated products and sums are
defined. [Solution on page 225]
(a) (AT )T = A
(b) (A + B)T = AT + B T
(c) (AB)T = AT B T
(d) (AB)T = B T AT
(e) (ABC)T = C T B T AT
(f ) (AB)2 = A2 B 2
(g) A2 − In = (A − In )(A + In )
26. If A and B are matrices, can you conclude in general that (A+B)2 = A2 +2AB +B 2 ?.
If this is not true in general, give a specific example of A and B that makes the equality
false. [Solution on page 226]
40 Chapter 4. Matrix Algebra
27. A student tries to solve the matrix equation X 2 + BX = XC (where all of these are
n × n matrices) as follows:
X 2 =XC − BX
X 2 =X(C − B)
X 2 − X(C − B) =0
X(X − (C − B)) =0
X = 0 or X = C − B
Explain two different matrix algebra mistakes that the student made. [Solution on
page 227]
4. Find the inverse of each matrix. If the inverse does not exist, explain why. [Solution
on page 229]
4.2. The Inverse of a Matrix 41
1 2 −2 3
(a) A = (c) A =
2 6 −4 8
−6 7 15 −18
(b) A = (d) A =
12 −14 −10 12
k −5
5. Let A = . Find the values of k that make A invertible. [Solution on
−4 k + 1
page 230]
6. In each case, solve the linear system by finding and using the inverse of the coefficient
matrix. [Solution on page 230]
−3x1 − 6x2 = 10 4x1 − 5x2 = 11
(a) (b)
−x1 + 3x2 = 10 7x1 + 6x2 = −3
7. Find the inverse of each matrix. If the inverse does not exist, explain why. [Solution
on page 231]
1 1 1 1 −6 5
(a) A = 0 1 1 (c) A = −2 13 −17
2 3 4 3 −19 22
0 2 1
(d) A = −2 −2 −2
−1 0 1
1 −3 −2
1 0 0 0
(b) A = −1 5 2 2 1 0 0
−3 7 7 (e) A =
3
1 1 0
4 1 1 1
9. Let A and B be square matrices of the same size. Suppose that A and B are both
invertible. [Solution on page 235]
10. Determine if the statement is True or False, and justify your answer. In the following
the identity matrix of any size is denoted I. [Solution on page 237]
42 Chapter 4. Matrix Algebra
13. Let A, B, C, and X be n×n matrices with X and B invertible. Consider the equation
XA = B + XC. [Solution on page 240]
14. Let A, B, C, and X be n×n matrices with A, C, and X invertible. Solve the following
equation for X. [Solution on page 241]
BAX T = C + 2AX T
(If you need to invert a matrix, explain why that matrix is invertible.)
15. Suppose that A, B, X, are n × n matrices and assume that A and X are invertible.
Solve the equation (XA)−1 (A + X) = B for the matrix X in terms of A and B,
showing your steps. State clearly which matrix you need to assume is invertible.
Prove that it is invertible. [Solution on page 241]
16. Determine if the statement is True or False, and justify your answer. [Solution on
page 242]
(b) If the equation Ax = 0 has only the trivial solution, then there is a pivot position
in every row of A.
(c) If there is a pivot position in every column of A, then A is invertible.
(d) If A is a square matrix and if there is a pivot position in every column, then A
is invertible.
(e) If A and C are two matrices such that CA = I, then then A is invertible.
(f ) If A and C are square matrices such that CA = I, then A is invertible.
(g) If a linear transformation T (x) = Ax is one-to-one, then A is invertible.
(h) If an n × n matrix A is invertible, then the columns of A span Rn .
(i) If A is a square matrix, the linear transformation T (x) = Ax is one-to-one if
and only if it is onto.
(j) It is possible that A is invertible and AT is not.
(k) If A and B are square matrices such that AB = I, then A and B commute.
(l) It is possible to find a 4 × 4 invertible matrix whose columns do not span R4 .
(m) If A is a square matrix whose columns are not linearly independent, then A is
not invertible.
(n) If the linear transformation T : Rn → Rn is not onto, then it is not one-to-one.
18. Let A be an n × n matrix with two identical columns. Is A invertible? Why or why
not? [Solution on page 244]
19. Let A be an invertible matrix. Show that the columns of A−1 are linearly independent.
[Solution on page 244]
20. Let A be a square n × n matrix. Suppose the columns of A span Rn . Show that the
columns of A3 are linearly independent. [Solution on page 244]
44 Chapter 4. Matrix Algebra
Chapter5
Subspaces of Rn
45
46 Chapter 5. Subspaces of Rn
x2
x1
x2
x1
x2
x1
x2
x1
x2
x1
3
H 2
2
S
1
−2 −1 1 2
1 2 3
8. Determine if the statement is True or False, and justify your answer. [Solution on
page 250]
(a) If A is a matrix, the column space of A is the set of all linear combinations of
the columns of A.
(b) If A is an m × n matrix, the column space of A is a subspace of Rn .
(c) If A is an m × n matrix, the column space of A is a subspace of Rm .
(d) The column space of a 3 × 2 matrix can be R2 .
(e) The column space of a 3 × 2 matrix is always a plane in R3 .
(f ) If the columns of a 3 × 3 matrix A are linearly dependent, then Col(A) = R3 .
(g) If the columns of a 3 × 3 matrix A are linearly independent, then Col(A) = R3 .
(h) If the columns of an m×n matrix A are linearly independent, then Col(A) = Rm .
(i) If A is an m × n matrix and if b is a vector in Rm , then the linear system Ax = b
has at least one solution if and only if b is in Col(A).
(j) Let A be a 2 × 2 matrix such that Col(A) is a line in R2 . Let b be a vector in
R2 . If b does not lie on Col(A), the system Ax = b is consistent.
1 2 7
9. Let v1 = −1 , v2 = −3 , and u = −9 . [Solution on page 253]
−2 4 2
1
50 Chapter 5. Subspaces of Rn
1 −1
2. Let v1 = and v2 = . [Solution on page 262]
2 3
4. In each case, determine if the set B is a basis for Rn . [Solution on page 266]
18
(a) n = 2, v = , B = {v}.
−25
9 −3
(b) n = 2, v1 = , v2 = , B = {v1 , v2 }.
−12 4
1 0
(c) n = 3, v1 = 2 , v2 = 1 , B = {v1 , v2 }.
0 3
5.2. Basis, Dimension, and Rank 51
−4 −1 −5
(d) n = 3, v1 = 3 , v2 = 2 , v3 = 0 , B = {v1 , v2 , v3 }.
7 6 −4
−17 29 2 −51
(e) n = 3, v1 = 2 , v2 = 32 , v3 = −43 , v4 = −3 , B =
8 9 −12 −61
{v1 , v2 , v3 , v4 }.
2 1 1
5. Let v1 = 0 , v2 = −1 , x = 3 . Let H = Span{v1 , v2 }, and let
1 3 −7
B = {v1 , v2 }. [Solution on page 266]
6. In each case find a basis for Null(A) and a basis for Col(A). [Solution on page 267]
1 2 4 5 −30 40
(a) A = 3 7 7 (b) A = 3 −18 24
−4 −9 −11 −4 24 −32
7. Determine if the statement is True or False, and justify your answer. [Solution on
page 270]
(l) If A is a matrix, the dimension of the null space of A is equal to the number of
free variables of the system Ax = 0.
2 x1
8. Let H be the subspace of R consisting of vectors such that x1 − 5x2 = 0.
x2
[Solution on page 271]
11. Determine if the statement is True or False, and justify your answer. [Solution on
page 272]
12. In each case, find (i) a basis for Null(A); (ii) the dimension of Null(A); (iii) a basis
for Col(A); (iv) the dimension of Col(A); (v) the rank of A. [Solution on page 274]
1 −1 1 1 2 −1
(a) A = 0 1 1 (b) A = −1 −1 4
−1 2 1 −3 −6 3
5.2. Basis, Dimension, and Rank 53
1 3 0 2 1 −5 2 0 −1
0 0 2 −8 (d) A = 3 −15 6 1 −6
(c) A =
−1 −3
2 −10 −1 5 −2 −1 4
2 6 −2 12
3 1 −1 2 1
−3 −1 1 −4 6
13. Let A =
0
. [Solution on page 280]
0 0 0 0
−6 −2 2 −4 −2
16. Let A be a 9 × 14 matrix. Suppose that an echelon form of A has exactly five zero
rows. [Solution on page 281]
17. Suppose A is a 5 × 10 matrix that has four pivot columns. Is Col(A) = R4 ? What is
the dimension of Null(A)? Justify your answers. [Solution on page 281]
18. Determine if the statement is True or False, and justify your answer. [Solution on
page 281]
19. Let A be a 7 × 7 matrix. Suppose that the columns of A are linearly independent.
[Solution on page 282]
Determinants
55
56 Chapter 6. Determinants
0 4 0 −5 −2 −1 1 2 0
1 −6 −3 5 1 3 2 1 −4
(a) A =
0 7 0 −2 (c) A =
0 0 2 0 0
4 −6 0 2 −1 1 5 6 0
2 1 6 3 0
3 0 7 −2 −4
2 8 −1 6
−2 0 8 0 9
−3 21 4 −4 (d) A = 1 −6 −9 2 −8
(b) A =
0 −2
0 0 −5 0 6 −1 −3
5 19 −7 3 0 0 5 0 0
7. Determine if the statement is True or False, and justify your answer. [Solution on
page 294]
0 0 0
(a) If A = , the determinant of A is 0.
0 0 0
(b) The determinant of a matrix A is defined if and only if A is a square matrix.
(c) The cofactor expansion of det A across a row is not equal to the cofactor expan-
sion down a column.
(d) The cofactor expansion of det A does not depend on the row or column we
choose.
5 2
(e) The determinant of is 30 − 14 = 16.
−7 6
0 4 5
(f ) The determinant of −5 1 2 is
0 2 3
4 5
−5 = −5(12 − 10) = −10
2 3
a b
(g) If A = , det(3A) = 9 det A.
c d
4 9 −6
(h) The matrix 0 2 −2 is lower triangular.
0 0 −3
(i) The determinant of a triangular matrix is the product of the entries on the main
diagonal.
6.2. Properties of Determinants 57
(a) A R2 − R1 → R2 B
(b) A R3 + 5R1 → R3 B
(c) A R2 ↔ R3 B
(d) A 3R2 → R2 B
1
(e) A R
4 1
→ R1 B
4
R1 ↔ R3 , R2 + 6R1 → R2 , −3R2 → R2 , R3 − 7R1 → R3 , R
5 3
→ R3
3. Compute the determinant of each matrix by row reduction to echelon form. [Solution
on page 296]
5 −10 10 0 −5 −3 −8
(a) A = −2 8 1 −1 4 0 3
(b) A =
3 −10 4 1 −1 2 3
2 2 4 13
0 1 4 −2
−3 7 −1 −5
4. Let A = . Compute det A. [Solution on page 297]
0 2 3 −2
3 −7 6 8
a b c
6. Let A = d e f . Suppose det A = 7. Find the determinant of the following
g h i
matrices. [Solution on page 299]
2a 2b 2c 3d 3e 3f
B = d − a e − b f − c and C = a + d b + e c + f
g + 3a h + 3b i + 3c g − 2d h − 2e i − 2f
7. In each case, use the determinant to determine whether the set S is linearly indepen-
dent. [Solution on
page 300]
2 1 4
(a) S = {v1 , v2 , v3 }, where v1 = 0 , v2 = 9 , and v3 = 3 .
1 2 3
(b) S = {v1 , v2 , v3 , v4 }, where
1 0 3 2
−1
, and v4 = −8
5 1
v1 = 2
, v2 =
4 , v3 = 3
12
−3 −8 −3 −22
k 0 3
8. Let A = 7 1 −8 . Find the values of k for which A is invertible. [Solution
2 0 k+1
on page 301]
9. Let A and B be matrices such that det A = 2 and det B = −3. [Solution on
page 301]
10. Let A be a square matrix such that A2 = A. (Such a matrix is called idempotent.)
Show that det A = 0 or det A = 1. [Solution on page 302]
11. Construct two matrices A and B such that det(A + B) 6= det A + det B. [Solution
on page 302]
12. Determine if the statement is True or False, and justify your answer. [Solution on
page 302]
4. In each case, find a basis for the eigenspace corresponding to the eigenvalue λ. [So-
lution on page 307]
3 −3
(a) A = , λ = 5.
−2 2
0 1 0
(b) A = 3 0 1 , λ = −1.
2 0 0
0 3 1
(c) A = 1 −2 −1 , λ = 1.
−3 9 4
61
62 Chapter 7. Eigenvalues and Eigenvectors
5 −3 0 4 0
(a) A =
−10 6 (d) A = 1 0 0
0 0 1
4 7 −5
−1 1 4
(b) A = 0 −2 5
(e) A = 0 2 0
0 0 3
3 −3 −2
3 0 0 0 3 1
(c) A = −1 2 0 (f ) A = 1 −2 −1
8 5 3 −3 9 4
4. Let A be a 2 × 2 matrix. Define tr(A) as the sum of the entries on the main diagonal
of A. (tr(A) is called the trace of A.) [Solution on page 314]
5. Let A be a 2 × 2 symmetric matrix. Show that all the eigenvalues of A are real
numbers. [Solution on page 316]
6. Determine if the statement is True or False, and justify your answer. [Solution on
page 317]
7.3 Diagonalization
1. Determine if the statement is True or False, and justify your answer. [Solution on
page 318]
2 0 0
(a) The matrix D = 4 0 0 is a diagonal matrix.
0 0 7
−3 0 0
(b) The matrix D = 0 −1 0 is a diagonal matrix.
0 0 8
1 0 n 1 0
(c) If D = , then D = for every n ≥ 1.
0 5 0 5n
(d) A square matrix A is diagonalizable if there exist a diagonal matrix D and an
invertible matrix P such that A = P DP −1 .
(e) A 2 × 3 matrix can be diagonalizable.
(f ) If A is a diagonal matrix, then A is diagonalizable.
−1 2 −2
3. Let A = 0 1 0 . [Solution on page 327]
4 −4 5
(a) Show that A is diagonalizable, and find an invertible matrix P and a diagonal
matrix D such that A = P DP −1 .
(b) Compute Ak for an arbitrary integer k ≥ 1.
7.3. Diagonalization 65
4. Find a 2 × 2 matrix A satisfying the following two conditions: [Solution on page 331]
4
λ = 1 is an eigenvalue of A with eigenvector v1 = .
5
−1
λ = −2 is an eigenvalue of A with eigenvector v2 = .
4
7. Determine if the statement is True or False, and justify your answer. [Solution on
page 332]
Solutions to Exercises
67
Chapter1
Linear Equations
A linear equation is an equation of the form
a1 x 1 + a2 x 2 + · · · + an x n = b (1.1.1)
where
n is a positive integer;
x1 , x2 , · · · , xn are variables.
69
70 Chapter 1. Systems of Linear Equations
3 3
x1 + x3 = 1
4 2
2. [Exercise on page 3]
a1 x1 + a2 x2 + · · · + an xn = b
From the table, only the point (2, −3, 0) is a solution to the equation −4x1 −
5x2 + 21x3 = 7.
3. [Exercise on page 3]
x1 − x2 + x3 − x4 = 0
3x1 − 4x2 = −1
2x1 − 3x2 − 5x4 = 2
5x1 + 7x2 = 12
x2 − x3 + x4 = −3
The first has two variables and two equations, while the second has four vari-
ables and three equations.
Note. In a linear system every equation is linear. If an equation is not linear,
the system is not a linear system.
(a) Here the system is a linear system since every equation is linear.
(b) The system here is a linear system.
(c) The system here is not a linear system since the third equation, −2x1 + ex3 =
−2, is not linear.
√
(d) Here the system is not a linear system since the first equation, 34x1 − 5x2 =
−4, (as well as the second equation) is not linear.
4. [Exercise on page 3]
72 Chapter 1. Systems of Linear Equations
x1 − x2 = 1
x1 + x2 = 3
This table shows that (0, −1, 2) satisfy the first two equations, but not the third.
So (0, −1, 2) is not a solution to the system.
(d) Consider the following table.
1.1. Introduction to Systems of Linear Equations 73
From the table, we see that (2, −1, 3, 1) satisfies each equation of the system.
So (2, −1, 3, 1) is a solution to the system.
5. [Exercise on page 4]
x1 + x2 = 1
−x1 + x2 = −1
x2
−x1 + x2 = −1
1 3 x1
−1
x1 + x2 = 1
From the figure, the lines intersect at (1, 0), which is the solution to the given
system. Since the system has at least one solution, it is consistent.
(b) Sketching the graphs of −x1 + 2x2 = 3 and 2x1 − 4x2 = −5, we get the following
figure.
x2
−x1 + 2x2 = 3
2x1 − 4x2 = −5
−1 1 x1
Since the two lines are parallel, there is no intersection. This means that the
system has no solution. In other words, the system is inconsistent.
(c) The figure below shows the graphs of 2x1 + 3x2 = 5 and −3x1 + x2 = 9.
x2
10 −3x1 + x2 = 9
3
2x1 + 3x2 = 5
−2 2 4 x1
From the figure, the system is consistent because the lines intersect. The point
of intersection is (−2, 3), which is also the unique solution to the system.
1.1. Introduction to Systems of Linear Equations 75
(d) First, observe that if we divide both sides of the second equation, −2x1 + 2x2 =
−4, by −2, we get the first equation. This means that the two equations have
the same graph as shown in the following figure.
x2
2 x1 − x2 = 2
−2 2 x1
−2
−2x1 + 2x2 = −4
Since the graphs of the two equations coincide, the system has infinitely many
solutions. So it is consistent and a solution is for example the point (2, 0).
Actually, every point on the line is a solution to the system.
6. [Exercise on page 4]
We want to form a system in two variables with two equations whose every coefficient
is nonzero and (5, −7) is a solution. The idea is to take combinations of 5 and −7.
For example, 5 + (−7) = −2. This means that (5, −7) is a solution to the
equation x1 + x2 = −2.
From the combination 5(2) + 3(−7) = −11, it follows that (5, −7) is a solution
to the equation 2x1 + 3x2 = −11.
x1 + x2 = −2
2x1 + 3x2 = −11
Note. If you consider different combinations, you will end up with a different system
(which is fine).
7. [Exercise on page 4]
As before we need to consider combinations of −2, 3, and −5.
From the combination (−2) + (3) + (−5) = −4, it follows that (−2, 3, −5) is a
solution to the equation x1 + x2 + x3 = −4.
From the combination 2(−2) + (3) + (−5) = −6, it follows that (−2, 3, −5) is a
solution to the equation 2x1 + x2 + x3 = −6.
From the combination 3(−2) + (3) + (−5) = −8, it follows that (−2, 3, −5) is a
solution to the equation 3x1 + x2 + x3 = −8.
76 Chapter 1. Systems of Linear Equations
x1 + x2 + x3 = −4
2x1 + x2 + x3 = −6
3x1 + x2 + x3 = −8
Of course, this is not the unique answer.
8. [Exercise on page 4]
It is not possible to find a point (a, b) that satisfies the system. Indeed, a solution
to the system is a list (s1 , s2 , s3 ) of three numbers (not two numbers!) since the sys-
tem has three variables.
What is a Matrix?
A matrix can be defined as rectangular array of numbers organized in rows
and columns. Examples of matrices (plural of matrix) include:
1 2 3 0 0 2 15
4 0 0 and 0 0 0 0
−1 −3 6 −6 2 1 0
A leading entry of a row is the first nonzero entry from the left.
1.2. Echelon Matrices 77
7 2 3 4
For example, consider the matrix 0 0 0 0. The second row is a zero row,
0 5 0 9
while row 1 and row 3 are both nonzero rows. In row 1, the leading entry is
7. In row 2, there is no leading entry because by definition a leading entry is a
nonzero number (and every entry in row 2 is 0). In row 3, the leading entry is
5.
all the rows are nonzero rows except the third row. The leading entry of row
1 is 3, the leading entry of row 2 is 2, and the leading entry of row 3 does not
exist (DNE) since every entry in that row is 0 and by definition a leading entry
is a nonzero number. These are summarized in the following table.
Zero Row? (Yes or No) Nonzero Row? (Yes or No) Leading Entry
Row 1 No Yes 3
Row 2 No Yes 2
Row 3 Yes No DNE
Row 4 No Yes 1
0 0 0 0 0
8 0 0 0 5
(b) For the matrix , we have the following table.
0 0 −7 7 9
0 0 0 0 0
Zero Row? (Yes or No) Nonzero Row? (Yes or No) Leading Entry
Row 1 Yes No DNE
Row 2 No Yes 8
Row 3 No Yes -7
Row 4 Yes No DNE
2. [Exercise on page 5]
78 Chapter 1. Systems of Linear Equations
(1) Every nonzero row is above any zero rows. (This amounts to saying that
all zero rows are at the bottom.)
(2) Every leading entry is strictly to the right of the leading entry of the row
above it.
3 0 4
(a) The matrix A = 0 2 5 satisfies conditions (1) and (2). Indeed,
0 0 6
there is no nonzero rows in A. This implies that (1) is satisfied.
The leading entries are 3 (in row 1), 2 (in row 2), and 6 (in row 3). The
leading entry 6 is to the right of the leading entry of the row above it (which
is 2), and the leading entry 2 is to the right of the leading entry of the row
above it (which is 3). There is no rows above row 1. So condition (2) is
satisfied.
Thus, the matrix is in echelon form.
−1 1 2
(b) Likewise, the matrix 0 −2 4 is in echelon form.
0 0 0
4 0 0 0
(c) However, the matrix 0 0 0 0 is not in echelon form because (1) is
0 0 8 0
not satisfied as the zero row (row 2) is not at the bottom.
−7 0 1 0 2
0 0 4 3 2
(d) The matrix 0 6 0 0 6 is not in echelon form because it violates
0 0 0 0 1
condition (2). Indeed, the leading entry 6 in row 3 is not to the right of the
leading entry of the row above it (which is 4).
−9 0 3 0 5
0 8 4 3 2
(e) The matrix 0 2 0 0 −2 is not in echelon form because condition
0 0 0 0 0
(2) is violated. Indeed, the leading entry 2 in row 3 is not to the right of the
leading entry of the row above it (which is 8).
3. [Exercise on page 5]
1.2. Echelon Matrices 79
(4) Each leading entry is the only nonzero entry in the column containing it.
Yes, it is possible to find a matrix in echelon form which is not in reduced echelon
form. For example the matrix
1 3
0 2
is in echelon form, but not in reduced echelon form because it violates condition (3)
(it also violates condition (4)).
4. [Exercise on page 5]
1 0 0
(a) The matrix 0 1 0 is clearly in echelon form and in reduced echelon
0 0 1
form.
1 0 0 0
(b) The matrix 0 0 1 0 is not in reduced echelon form because it is
1 0 0 0
not even an echelon matrix (since it violates condition (2)).
1 0 1 0
(c) The matrix 0 0 1 0 is in echelon form, but not in reduced echelon
0 0 0 1
form because it violates condition (4). Indeed, the leading 1 in row 2 is not the
only nonzero entry in the column containing it.
0 1 0 0 4
(d) One can easily check that the matrix 0 0 0 1 6 is in echelon form
0 0 0 0 0
and in reduced echelon form.
1 0 0 1
(e) The matrix 0 0 5 0 is in echelon form, but not in reduced echelon
0 0 0 0
form because it does not satisfy condition (3) as the leading entry in row 2, 5,
is not 1.
80 Chapter 1. Systems of Linear Equations
1 0 0 0
(f ) The matrix 0 0 0 0 is not in reduced echelon form since it is not
0 0 0 1
even en echelon matrix (it does not satisfy condition (1)).
1 7 0 −2 0
(g) The matrix 0 0 1 8 0 is in echelon form and in reduced echelon
0 0 0 0 1
form since it satisfies conditions (1), (2), (3), and (4).
1 0 −5 1 0
(h) The matrix 0 1 0 0 0 violates condition (4) as the leading entry in
0 0 0 1 0
row 3, which is 1, is not the only nonzero entry in column 4. So it is not in
reduced echelon form. But is an echelon matrix.
5. [Exercise on page 5]
(iii) Replace a row with the sum of itself and a multiple of another row.
Notation. We use the following notation. We write Ri for the ith row. For
example, R1 will represent the first row at each stage, R2 will represent the
second row at each stage, and so on. Using this notation, the elementary row
operations become:
Warning. Any row operation which is not in one of the forms Ri ↔ Rj , aRi →
Ri , or Ri + bRj → Ri is not an elementary row operation. For example,
? i = k and a = 1 or
? j = k and b = 1.
If an operation is an elementary row operation, we will write yes next to it. Otherwise,
we will write no.
6. [Exercise on page 6]
(a)
1 3 −5 2 6 −10
−2 0 4 2R1 → R1 −2 0 4
4 −1 −6 4 −1 −6
(b)
1 3 −5 1 3 −5
−2 0 4 −4R3 → R3 −2 0 4
4 −1 −6 −16 4 24
82 Chapter 1. Systems of Linear Equations
(c)
1 3 −5 −2 0 4
−2 0 4 R1 ↔ R2 1 3 −5
4 −1 −6 4 −1 −6
(d)
1 3 −5 1 3 −5
−2 0 4 R2 ↔ R3 4 −1 −6
4 −1 −6 −2 0 4
(e)
1 3 −5 1 3 −5
−2 0 4 R2 + R1 → R2 −1 3 −1
4 −1 −6 4 −1 −6
(f )
1 3 −5 −1 3 −1
−2 0 4 R1 + R2 → R1 −2 0 4
4 −1 −6 4 −1 −6
(g)
1 3 −5 1 3 −5
−2 0 4 R2 + 2R1 → R2 0 6 −6
4 −1 −6 4 −1 −6
(h)
1 3 −5 7 3 −17
−2 0 4 R1 − 3R2 → R1 −2 0 4
4 −1 −6 4 −1 −6
(i)
1 3 −5 1 3 −5
−2 0 4 R3 − 21 R1 → R3 −2 0 4
7
4 −1 −6 2
− 2 − 72
5
7. [Exercise on page 6]
If the operation that was performed is Ri ↔ Rj , then one should be able to see this
immediately. If only one operation (aRi → Ri or Ri + bRj → Ri ) is performed on A
to get B, then every row of B will appear in A except one (the one affected by the
operation). In this case, one needs to first find the row of B that does not appear in
A. Then one needs to figure out the correct operation that was performed.
1.2. Echelon Matrices 83
1 2 1 2
(a) A = B=
−2 4 0 8
First, observe that the first row of B appears in A, but the second one does
not. So the second row of A is the only one that changed.
Since only one row changed, the operation that was performed is either on
the form aRi → Ri or on the form Ri + bRj → Ri .
– Is it possible to multiply the second row of A by a nonzero number to
get the second row of B? Clearly the answer is no. So the required
operation is not on the form aRi → Ri .
– So it is on the form R2 + bR1 → R2 . By inspection, one can see that the
correct operation that was performed on A to get B is R2 +2R1 → R2 .
−1 3 0 −1 3 −1
(b) A = 1 −2 −3 B = 1 −2 −3
−1 3 −1 −1 3 0
The operation that was performed is R1 ↔ R3 .
−2 3 −1 −2 3 −1
(c) A = 0 2 −2 B = 0 2 −2
2 −3 −1 −2 3 1
Since only one row has changed (row 3), the required operation is either on
the form aRi → Ri or on the form Ri + bRj → Ri .
Is it possible to multiply the third row of A by a nonzero number a and
obtain the third row of B? Yes, a = −1.
So the required operation is: −R3 → R3 .
(d)
3 5 −4 1 5 2
A = −1 0 3 R1 + 2R2 → R1 −1 0 3 = B
1 5 2 1 5 2
(e)
3 −12 −2 3 −12 −2
A = 2 −3 −7 R2 − 4R3 → R2 −2 13 −3 = B
1 −4 −1 1 −4 −1
8. [Exercise on page 6]
The point here is that these two operations can be performed at the same time
on A:
1 2 1 1 2 1
R2 − R1 → R2
A= 1 5 3 0 3 2 = A2
R3 − 2R1 → R3
2 7 4 0 4 3
Warning. One has to be careful when performing more than one operation at
the same time on a matrix. A common mistake is to do the following:
1 2 1 1 2 1
R2 − R1 → R2
A= 1 5 3 0 3 2
R3 − R2 → R3
2 7 4 1 2 1
The second operation should have involved R20 (not R2 ) where R20 = R2 − R1 .
Here is the correct sequence:
1 2 1 1 2 1 1 2 1
1 5 3 R2 − R1 → R2 0 3 2 R3 − R2 → R3 0 3 2
2 7 4 1 2 1 1 −1 −1
The following
1 1 2 1 1 2
1 4 2 R2 − R3 → R2 −2 −1 −4
R3 − R2 → R3
3 5 6 2 1 4
is incorrect for the same reason as the one mentioned above: R2 is replaced by
something (by performing the first operation) and the same R2 is involved in the
second operation. The correct sequence should be:
1 1 2 1 1 2 1 1 2
1 4 2 R2 − R3 → R2 −2 −1 −4 R3 − R2 → R3 −2 −1 −4
3 5 6 3 5 6 5 6 10
9. [Exercise on page 6]
Pivots
Let A be an echelon matrix.
The pivots of A are 2, −5, and 7. The pivot positions are (1, 1), (2, 3), and
(3, 4). And the pivot columns are column 1, column 3, and column 4.
Now, we focus on the second column. The 1 in the second position of the
second row makes a good pivot. Using that pivot, we can create a 0 in
the position right below it by performing the operation R3 + 3R2 → R3 .
We obtain
1 −2 3 1 −2 3
0 1 2 R3 + 3R2 → R3 0 1 2
0 −3 5 0 0 11
1 −2 3
The matrix B = 0 1 2 is an echelon matrix. Since it has been
0 0 11
obtained from A by a sequence of elementary row operations, it is an echelon
form of A.
1 0 0
So a reduced echelon form of A is the matrix 0 1 0 .
0 0 1
The process that enabled us to transform A into a reduced echelon matrix is
referred to as Gauss-Jordan elimination. In other words, the Gauss-Jordan
elimination is the combination of the forward and backward phases. While the
Gaussian elimination is just the forward phase.
Although a matrix can have many echelon forms as we saw above, the reduced
echelon form is unique as stated in the following theorem.
We come back to the exercise. Using Theorem 1.2.1, the answer to the question is:
False.
We thus obtain two echelon forms of A, namely B and C. Clearly, they don’t have
the same pivots as the pivots of B are 1 and 4 while the pivots of C are 1 and 1.
Row Equivalence
Two matrices are said to be row equivalent if one can be obtained from the
other by performing a finite sequence of elementary row operations.
Notation. If two matrices A and B are row equivalent, we write A ∼ B.
1 0 2 1 0 2
R2 − R1 → R2
A= 1 3 3 0 3 1
R3 + R1 → R3
−1 3 3 0 3 5
1 0 2
R3 − R2 → R3 0 3 1 = B
0 0 4
This shows that A and B are row equivalent (or A is row equivalent to B).
(b)
1 −2 4 1 −2 4
R2 − 3R1 → R2
A = 3 −7 15 0 −1 3
R3 − R1 → R3
1 0 3 0 2 −1
1 −2 4
R3 + 2R2 → R3 0 −1 3
0 0 5
1 −2 4
1
R → R3
5 3
0 −1 3 = B
0 0 1
This shows that A and B are row equivalent.
(c) Since the upper left position of A is 0, it can’t serve as a pivot. So we first need
to interchange row 1 with a row below it in order to place a nonzero number in
the upper left position. There are two possibilities: either we use the operation
R1 ↔ R2 or we use R1 ↔ R3 . The first operation will place a −3 in the pivot
while the second will place a 1. Since it is easier to work with 1 as a pivot, we
will use the operation R1 ↔ R3 first.
0 2 1 1 1 3
A = −3 1 −2 R1 ↔ R3 −3 1 −2
1 1 3 0 2 1
1.2. Echelon Matrices 91
1 1 3
R2 + 3R1 → R2 0 4 7
0 2 1
1 1 3
2R3 → R3 0 4 7
0 4 2
1 1 3
R3 − R2 → R3 0 4 7 =B
0 0 −5
First, the rightmost pivot is 1 which is located in the third row and third
column. We need to create zeros in each position above that pivot. Per-
forming the operations R2 − 3R3 → R2 and R1 + 4R3 → R1 , we get the
matrix
1 −6 0 13
0 2 0 −4
0 0 1 3
Next, we create a zero above the pivot located in the second row and second
column. Performing the operation R1 + 3R2 → R1 , we get the matrix
1 0 0 1
0 2 0 −4
0 0 1 3
we get
1 2 3 4
0 1 −2 −3
0 3 −5 −4
(b)
3 −5 13 −2 1 −2 3 1
−2 2 −12 14 R1 ↔ R3 −2 2 −12 14
1 −2 3 1 3 −5 13 −2
1 −2 3 1
R2 + 2R1 → R2 0 −2 −6 16
R3 − 3R1 → R3
0 1 4 −5
1 −2 3 1
− 21 R2 → R2 0 1 3 −8
0 1 4 −5
1 −2 3 1
Forward Phase Ends R3 − R2 → R3 0 1 3 −8
0 0 1 3
1 −2 0 −8
R2 − 3R3 → R2
Backward Phase Starts 0 1 0 −17
R1 − 3R3 → R1
0 0 1 3
1 0 0 −42
R1 + 2R2 → R1 0 1 0 −17
0 0 1 3
1.2. Echelon Matrices 93
(c)
3 9 0 −3 1 3 0 −1
1
−1 −2
2 −2
R
3 1
→ R1 −1 −2
2 −2
4 11 −1 3 1
R → R4 4 11 −1 3
3 4
3 9 0 −3 1 3 0 −1
1 3 0 −1
R2 + R1 → R2 0 1 2 −3
R3 − 4R1 → R3
0 −1 −1
7
R4 − R1 → R4
0 0 0 0
1 3 0 −1
0 1 2 −3
Forward Phase Ends R3 + R2 → R3
0 0 1
4
0 0 0 0
1 3 0 −1
0 1 0 −11
Backward Phase Starts R2 − 2R3 → R2
0
0 1 4
0 0 0 0
1 0 0 32
0 1 0 −11
R1 − 3R2 → R1
0
0 1 4
0 0 0 0
(d)
−3 1 1 −1 1 −1 −1 −1
4 −2 −2 0 R1 + R2 → R1 4 −2 −2 0
5 0 −3 −1 5 0 −3 −1
1 −1 −1 −1
R2 − 4R1 → R2 0 2 2 4
R3 − 5R1 → R3
0 5 2 4
1 −1 −1 −1
1
R
2 2
→ R2 0 1 1 2
0 5 2 4
1 −1 −1 −1
R3 − 5R2 → R3 0 1 1 2
0 0 −3 −6
1 −1 −1 −1
Forward Phase Ends − 13 R3 0 1 1 2
0 0 1 2
1 −1 0 1
R2 − R3 → R2 0
Backward Phase Starts 1 0 0
R1 + R3 → R1
0 0 1 2
94 Chapter 1. Systems of Linear Equations
1 0 0 1
R1 + R2 → R1 0 1 0 0
0 0 1 2
(e)
1 2 3 −2 7 1 2 3 −2 7
R2 − R1 → R2
1 2 3 −2 6 0 0 0 0 −1
R3 − R1 → R3
1 2 3 −1 4 0 0 0 1 −3
R4 + R1 → R4
−1 −2 −4 3 −2 0 0 −1 1 5
1 2 3 −2 7
0 0 −1 1 5
R2 ↔ R4
0 0 0 1 −3
0 0 0 0 −1
1 2 3 −2 7
0 0 −1 1 5
Forward Phase Ends −R4 → R4
0 0 0 1 −3
0 0 0 0 1
1 2 3 −2 0
R3 + 3R4 → R3 0 0 −1 1 0
Backward Phase Starts R2 − 5R4 → R2
0
0 0 1 0
R1 − 7R4 → R1
0 0 0 0 1
1 2 3 0 0
R2 − R3 → R2 0 0 −1 0 0
R1 + 2R3 → R1 0 0 0 1 0
0 0 0 0 1
1 2 0 0 0
0 0 −1 0 0
R1 + 3R2 → R1
0 0
0 1 0
0 0 0 0 1
1 2 0 0 0
0 0 1 0 0
−R2 → R2
0 0 0 1 0
0 0 0 0 1
(f )
0 4 −3 −1 0 4 −3 −1
−3 1 1 2 − 21 R3 → R3 −3 1 1 2
−2 −4 2 −4 1 2 −1 2
1 2 −1 2
R1 ↔ R3 −3 1 1 2
0 4 −3 −1
1.3. Solving Linear Systems Using Matrices 95
1 2 −1 2
R2 + 3R1 → R2 0 7 −2 8
0 4 −3 −1
1 2 −1 2
R2 − 2R3 → R2 0 −1 4 10
0 8 −6 −2
1 2 −1 2
R3 + 8R2 → R3 0 −1 4 10
0 0 26 78
1 2 −1 2
1
Forward Phase Ends R
26 3
→ R3 0 −1 4 10
0 0 1 3
1 2 0 5
R2 − 4R3 → R2 0 −1 0 −2
Backward Phase Starts
R1 + R3 → R1
0 0 1 3
1 0 0 1
R1 + 2R2 → R1 0 −1 0 −2
0 0 1 3
1 0 0 1
−R2 → R2 0 1 0 2
0 0 1 3
The augmented matrix of (S) is the matrix containing all the coeffi-
cients and all the constant terms of (S).
96 Chapter 1. Systems of Linear Equations
3x1 + 4x2 = 5
(a) For the system
6x1 − 7x2 = 8
the coefficient matrix is
3 4
6 −7
the augmented matrix is
3 4 5
6 −7 8
−x1 − 3x2 + x3 = −2
(b) For the system x1 − x2 − 4x3 = 0
−5x1 + x3 = 6
The coefficient matrix is
−1 −3 1
1 −1 −4
−5 0 1
2. [Exercise on page 8]
1 −1 3 9
(a) The linear system corresponding to the augmented matrix −2 4 −8 0
3 −5 6 12
is
x1 − x2 + 3x3 = 9
−2x1 + 4x2 − 8x3 = 0
3x1 − 5x2 + 6x3 = 12
1.3. Solving Linear Systems Using Matrices 97
1 −2 4 10
(b) The linear system corresponding to the augmented matrix 0 −3 −9 −41
0 0 2 23
is
x1 − 2x2 + 4x3 = 10
− 3x2 − 9x3 = −41
2x3 = 23
1 5 0 −2 −3
(c) The linear system corresponding to the augmented matrix 0 0 1 4 −1
0 0 0 1 7
is
x1 + 5x2 − 2x4 = −3
x3 + 4x4 = −1
x4 = 7
3. [Exercise on page 8]
True. This follows from the following definition.
4. [Exercise on page 8]
False. This is an immediate consequence of the following important theorem.
Step 3. Solve the linear system corresponding to the reduced echelon form.
(This system is pretty easy to solve.)
5. [Exercise on page 8]
1 0 3
R1 − R2 → R1
0 1 4
The corresponding system is
x1 = 3
x2 = 4
So the solution is (3, 4).
Alternate Solution
x1 +x2 = 7
The system can be solved by using the so called
3x2 = 12
backward substitution:
6. [Exercise on page 8]
(a) We reduce the augmented matrix.
1 2 5 1 2 5
R2 − 2R1 → R2
2 −1 −10 0 −5 −20
1 2 5
− 15 R2 → R2
0 1 4
1.3. Solving Linear Systems Using Matrices 99
1 0 −3
R1 − 2R2 → R1
0 1 4
x1 = −3
x2 = 4
1 1 0 1 1 0
R2 − 3R1 → R2
3 −4 7 0 −7 7
1 1 0
− 17 R2 → R2
0 1 −1
1 0 1
R1 − R2 → R1
0 1 −1
x1 = 1
x2 = −1
2 −3 7 2 −3 7
R2 − 2R1 → R2
5 7 3 1 13 −11
1 13 −11
R1 ↔ R2
2 −3 7
1 13 −11
R2 − 2R1 → R2
0 −29 29
1 1 13 −11
− 29 R2 → R2
0 1 −1
1 0 2
R1 − 13R2 → R1
0 1 −1
x1 = 2
x2 = −1
7. [Exercise on page 9]
1 −1 0 −2
R2 + 2R3 → R2 0 3 0 0
R1 − R3 → R1
0 0 1 4
1 −1 0 −2
1
R
3 2
→ R2 0 1 0 0
0 0 1 4
1 0 0 −2
R1 + R2 → R1 0 1 0 0
0 0 1 4
1.3. Solving Linear Systems Using Matrices 101
x1 = −2
x2 = 0
x3 = 4
1 1 0 0 2
R3 + R4 → R3 0 1 −1 0 1
R2 − R4 → R2
0
0 1 0 0
R1 − R4 → R1
0 0 0 1 2
1 1 0 0 2
0 1 0 0 1
R2 + R3 → R2
0 0 1 0 0
0 0 0 1 2
1 0 0 0 1
0 1 0 0 1
R1 − R2 → R1
0 0 1 0 0
0 0 0 1 2
102 Chapter 1. Systems of Linear Equations
8. [Exercise on page 9]
9. [Exercise on page 9]
3x1 + x2 − x3 = 4
(a) The augmented matrix of the system is
5x2 − 2x3 = −8
3 1 −1 4
N=
0 5 −2 −8
Observe that N is in echelon form. The leading entries are 3 and 5. And
the variables corresponding to these entries are x1 and x2 respectively. So
the leading variables are x1 and x2 .
The other variable, x3 , is a free variable.
(b) For the system
x1 − x2 − x3 = 1
−x2 + x3 = 7
2x3 = 5
the leading variables are x1 , x2 , and x3 . There is no free variable.
(c) For the system
x1 + 5x2 − 7x3 + x4 = −9
7x3 − x4 = 9
the leading variables are x1 and x3 . The other variables, x2 and x4 , are
free.
x1 = 3
x2 = 5
0 = 7
Clearly, in this system, all the variables (x1 and x2 ) are leading variables. But
the system has no solution because the third equation is not satisfied.
For a system to have a unique solution, all the variables has to be leading
variables and one more condition has to be satisfied as one can see in the following
theorem.
Theorem 1.3.2. Let (S) be a linear system, and let M be the augmented
matrix of (S). Let N be an echelon form of M .
(A) The system (S) is inconsistent if and only if the matrix N has a row
104 Chapter 1. Systems of Linear Equations
of the form
0 0 ··· 0 b
with b 6= 0. (This amounts to saying that (S) is inconsistent if and
only if the rightmost column of N is a pivot column.)
(B) The system (S) has a unique solution if and only if the following two
conditions are satisfied:
(1) The matrix N has no row of the form 0 0 · · · 0 b with
b 6= 0. (This amounts to saying that the system (S) is consistent
thanks to part (I).)
(2) All the variables of (S) are leading variables.
(C) The system (S) has infinitely many solutions if and only if the fol-
lowing two conditions are satisfied:
(i) The matrix N has no row of the form 0 0 · · · 0 b with
b 6= 0. (This amounts to saying that the system (S) is consis-
tent.)
(ii) At least one variable of (S) is a free variable.
x1 + x2 = 6
0 = 1
This system has a free variable (x2 ), but it is not consistent as the second
equation is not satisfied.
Step 1. Transform the augmented matrix into a reduced echelon form using
elementary row operations.
Step 2. If there is a row of the form 0 0 · · · 0 b , with b 6= 0, the
system is inconsistent.
Step 3. If the system is consistent, solve the system corresponding to the re-
duced echelon form for leading variables (if there are free variables,
express each leading variable in terms of the free variables).
1.3. Solving Linear Systems Using Matrices 105
x1 − 2x2 + 4x3 = 5
−x1 + 3x2 − 2x3 = −2
3x1 − 4x2 + 16x3 = 21
x3 = s
Now we solve each equation for the leading variables (x1 and x2 ) in terms of s.
Replacing x3 with s into the first equation, we get x1 + 8s = 11. Solving
this for x1 , we get x1 = 11 − 8s.
Replacing x3 with s into the second equation, we get x2 + 2s = 3. This
implies that x2 = 3 − 2s.
Thus, the general solution to the system is
x1 = 11 − 8s
x2 = 3 − 2s
x3 = s
where s can be any real number.
Warning. Do not forget to write the sentence “where s can be any real number”.
(b) First, we reduce the augmented matrix.
1 3 2 7 1 3 2 7
R2 − 2R1 → R2
2 7 3 10 0 1 −1 −4
R3 + 4R1 → R3
−4 −13 −7 −24 0 −1 1 4
1 3 2 7
R3 + R2 → R3 0 1 −1 −4
0 0 0 0
106 Chapter 1. Systems of Linear Equations
1 0 5 19
R1 − 3R2 → R1 0 1 −1 −4
0 0 0 0
The linear system corresponding to the reduced echelon form is
x1 + 5x3 = 19
x2 − x3 = −4
In this latter system x3 is a free variable. So we set x3 equal to a parameter:
x3 = s. Solving the the first and second equations for the leading variables (x1
and x2 ), we get x1 = 19 − 5s and x2 = −4 + s. So the general solution is
x1 = 19 − 5s
x2 = −4 + s
x3 = s
where s can be any real number.
(c) We reduce the augmented matrix.
1 1 2 1 1 1 2 1
1 R2 − R1 → R2
2 2 4 0 1 0 3
R3 + R1 → R3
−1 −3 −2 −2 0 −2 0 −1
1 1 2 1
R3 + 2R2 → R3 0 1 0 3
0 0 0 5
We do not need to go further because the third equation is 0 = 5, which is not
true. So the system has no solution.
(d) Transforming the augmented matrix:
1 −2 0 −1 2 1 −2 0 −1 2
2 −4 R2 − 2R1 → R2
1 −5 6 0 0 1 −3 2
R3 − R1 → R3
1 −2 −1 2 0 0 0 −1 3 −2
1 −2 0 −1 2
R3 + R2 → R3 0 0 1 −3 2
0 0 0 0 0
The linear system corresponding to the reduced echelon form is
x1 − 2x2 − x4 = 2
x3 − 3x4 = 2
In this latter system x2 and x4 are free variables. So we set them equal to
parameters: x2 = t and x4 = s. Solving the the first and second equations for
the leading variables (x1 and x3 ), we get x1 = 2 + 2t + s and x3 = 2 + 3s. So
the general solution is
x1 = 2 + 2t + s
x2 = t
x3 = 2 + 3s
x4 = s
where s and t can be any real numbers.
1.3. Solving Linear Systems Using Matrices 107
3 8 −3 −14 2 1 −2 1 10 0
2 3 −1 −2 1 2 3 −1 −2 1
1 −2
R1 ↔ R3
1 10 0 3 8 −3 −14 2
1 5 −2 −12 1 1 5 −2 −12 1
1 −2 1 10 0
R2 − 2R1 → R2 0 7 −3 −22 1
R3 − 3R1 → R3
0 14 −6 −44 2
R4 − R1 → R4
0 7 −3 −22 1
1 −2 1 10 0
R3 − 2R2 → R3 0
7 −3 −22 1
R4 − R2 → R4 0 0 0 0 0
0 0 0 0 0
1 −2 1 10 0
1
0 3
1 − 7 − 22 1
R
7 2
→ R2
0
7 7
0 0 0 0
0 0 0 0 0
1 26 2
1 0 7 7 7
0 1 − 73 − 22 1
R1 + 2R2 → R1 7 7
0 0 0 0 0
0 0 0 0 0
(Note. Here we can’t avoid fractions.) The linear system corresponding to the
reduced echelon form is
1 26 2
x1 + x
7 3
+ 7 4
x = 7
3 22 1
x2 − x
7 3
− 7 4
x = 7
The leading variables are x1 and x2 , and the free ones are x3 and x4 . Set x3 = s and
x4 = t . Then the general solution is
2
x1 = 7
− 17 s − 26
7
t
1
x2 = + 37 s +
7
22
7
t
x3 = s
x4 = t
where s and t can be any real numbers.
12. [Exercise on page 10]
x1 − x3 + x4 = 2
x2 − 7x4 = −3
−1 3 2 −8
R3 − 12R2 → R3 0 1 1 −2
0 0 a−6 b
1 −3 −2 8
−R1 → R1 0 1 1 −2
0 0 a−6 b
At this stage, the matrix is just in an echelon form, which is enough to solve the
problem.
x1 − 3x2 − 2x3 = 8
x2 + x3 = −2
0 = 0
Since this latter (echelon) system has a free variable (x3 ) and since there is no
equation of the form 0 = d with d 6= 0, it follows that the system has infinitely
many solutions (thanks to Theorem 1.3.2–Part (C)).
If a 6= 6, then the system becomes
x1 − 3x2 − 2x3 = 8
x2 + x3 = −2
(a − 6)x3 = b
Since all the variables of this latter (echelon) system are leading variables and
since there is no equation of the form 0 = d with d 6= 0, it follows that the
system has a unique solution (thanks to Theorem 1.3.2–Part (B)).
110 Chapter 1. Systems of Linear Equations
Chapter2
Vectors – Definition
Let n ≥ 1 be an integer.
For example,
4
u= is a vector in R2 with entries 4 (first entry) and −3 (second
−3
entry).
−2
The vector 5 is in R3 with entries −2 (first entry), 5 (second entry),
0
and 0 (third entry).
111
112 Chapter 2. Vectors in Rn and Linear Systems
Then the sum u + v is the vector of Rn whose ith entry is the sum of the
ith entry of u and the ith entry of v. That is,
u1 + v1
u2 + v2
u+v =
..
.
un + vn
1 −3 1 + (−3) −2
(a) u + v = + = =
2 4 2+4 6
6 10(6) 60
(b) 10w = 10 = =
−5 10(−5) −50
6 1 6 −2 4
(c) w − 2u = −2 = + =
−5 2 −5 −4 −9
−3 −3(−3) 9
(d) −3v = −3 = =
4 −3(4) −12
(f ) The vector −u − 3v + 4w is
1 −3 6
−u − 3v + 4w = − −3 +4
2 4 −5
−1 9 24 32
= + + =
−2 −12 −20 −34
y
4
−1 4 x
−1
y
4
−1 4 x
−1
y
4 u+v
−1 4 x
−1
y
3 3u
−2 3 x
−2u −2
(a) and (b) The vectors 3u and −2v are shown below.
y
3u
v
u
−4 5 x
−2v −4
y
3 u+v
v
u
−3 3 x
u−v
−v
(e) and (f ) The vectors −2u + 3v and −4u − v are shown below.
y
6 3v
−2u + 3v
3
v
u
−6 −3 3 6 x
−v −2u
−3
−4u
−6
−4u − v
u1 = v1 , u2 = v2 , · · · , un = vn
u1 0
u2 0
u+0= +
.. ..
. .
un 0
u1 + 0
u2 + 0
= By definition of vector addition
..
.
un + 0
u1
u2
= =u
..
.
un
u1 + v1
u2 + v2
= a By definition of vector addition
..
.
un + vn
au1 + av1
au2 + av2
= By definition of scalar multiplication
..
.
aun + avn
au1 av1
au2 av2
= + By definition of vector addition
.. ..
. .
aun avn
u1 v1
u2 v2
= a .. + a .. By definition of scalar multiplication
. .
un vn
= au + av
Vector Equations
Let n, m ≥ 1 be integers. A vector equation in n variables x1 , x2 , · · · , xn is
an equation of the form
x1 a1 + x2 a2 + · · · xn an = b,
By the definition of equality of two vectors, the latter equation gives the following
linear system:
x1 − 2x2 = 5
2x1 − x2 = 4
3x1 = 6
is
− 2x2 + 5x3 = 10
x1 + x2 = 12
x1 − x2 = 3
2x1 + 7x2 = −8
is
1 −1 3
x1 + x2 =
2 7 −8
2x1 − x2 + x3 = 5
−3x1 + 4x2 + 5x3 = 9
8x1 + 6x3 = 11
is
2 −1 1 5
x1 −3 + x2 4 + x3 5 = 9
8 0 6 11
x1 + 2x2 = 4
−4x1 − 3x2 = −1
(a) Solution. The graphs of x1 + 2x2 = 4 and −4x1 − 3x2 = −1 are shown below.
2.1. Vector Equations and Span 119
x2
x1 + 2x2 = 4
3 −4x1 − 3x2 = −1
−2 3 x1
−2
Geometrically, we see that the lines intersect at (−2, 3). This means that the
solution to the system is (−2, 3). The figure above is referred to as the row
picture of the system.
(b) To solve the system algebraically, we need to reduce the augmented matrix.
1 2 4 1 2 4
R2 + 4R1 → R2
−4 −3 −1 0 5 15
1 1 2 4
R
5 2
→ R2
0 1 3
1 0 −2
R1 − 2R2 → R1
0 1 3
So the solution is (−2, 3), which is, as expected, the same as the one found in
part (a).
(c) The vector equation corresponding to the system is
1 2 4
x1 + x2 =
−4 −3 −1
y
−2u 8
−9 b 8 x
v
u
−9 3v
Geometrically we see that −2u + 3v is indeed equal to b. The figure here is
referred to as the column picture of the system above.
Linear Combinations
Let n, p ≥ 1 be integers, and let a1 , · · · , ap be vectors in Rn . A linear
combination of a1 , · · · , ap is a vector of the form x1 a1 + · · · + xn an , where
x1 , · · · , xn are real numbers.
x1 − 2x2 = 8
This amounts to determining if the system −3x1 + 4x2 = −18 has a so-
5x2 = −15
lution. To solve this, we need to reduce the augmented matrix.
1 −2 8 1 −2 8
−3 4 −18 R2 + 3R1 → R2 0 −2 6
0 5 −15 0 5 −15
1 1 −2 8
− 2 R2 → R2
1
0 1 −3
R3 → R 3
5 0 1 −3
1 −2 8
R3 − R2 → R3 0 1 −3
0 0 0
1 0 2
R1 + 2R2 → R1 0 1 −3
0 0 0
x1 + 4x2 = 1
This amounts to determining if the system −2x1 − 11x2 = 1 has a so-
3x1 − x2 = 29
lution. To solve this, we need to reduce the augmented matrix.
1 4 1 1 4 1
−2 −11 1 R2 + 2R1 → R2 0 −3 3
R3 − 3R1 → R3
3 −1 29 0 −13 26
1 1 4 1
− 3 R2 → R2 0 1 −1
1
− 13 R3 → R3
0 1 −2
1 4 1
R3 − R2 → R3 0 1 −1
0 0 −1
Since the third equation of the corresponding system is of the form 0 = d with
d = −1 6= 0, the system has no solution. This implies that b is not a linear
combination of a1 and a2 .
1 2 0 −10 1 2 0 −10
1 1 3 12 R2 − R1 → R2 0 −1 3 22
0 −1 4 27 0 −1 4 27
1 2 0 −10
(−1)R2 → R2 0 1 −3 −22
0 −1 4 27
1 2 0 −10
R3 + R2 → R3 0 1 −3 −22
0 0 1 5
1 2 0 −10
R2 + 3R3 → R2 0 1 0 −7
0 0 1 5
1 0 0 4
R1 − 2R2 → R1 0 1 0 −7
0 0 1 5
2.1. Vector Equations and Span 123
Spanning Set
Let a1 , a2 , · · · , ap be vectors in Rn . The span of a1 , a2 , · · · , ap , denoted
Span{a1 , · · · , ap }, is the set of all linear combinations of a1 , · · · , ap .
are in Span{a1 , a2 }.
−1 4 3 1 −4 −3
2 −5 −3 (−1)R1 → R1 2 −5 −3
3 0 4 3 0 4
1 −4 −3
R2 − 2R1 → R2 0 3 3
R3 − 3R1 → R3
0 12 13
1 −4 −3
1
R
3 2
→ R2 0 1 1
0 12 13
124 Chapter 2. Vectors in Rn and Linear Systems
1 −4 −3
R3 − 12R2 → R3 0 1 1
0 0 1
1 30 1
1
R
7 3
→ R3 0
15
2
1 0 03 1
− 3 R4 → R4
0 03 1
1 3 1 0
0 1 2 5
R4 − R3 → R4
0
0 1 3
0 0 0 0
1 3 0 −3
R1 − R3 → R1 0
1 0 −1
R2 − 2R3 → R2 0 0 1 3
0 0 0 0
1 0 0 0
0 1 0 −1
R1 − 3R2 → R2
0
0 1 3
0 0 0 0
2.1. Vector Equations and Span 125
1 −3 α 1 −3 α
0 1 1 R3 + 3R1 → R3 0 1 1
−3 6 2 0 −3 2 + 3α
1 −3 α
R3 + 3R2 → R3 0 1 1
0 0 5 + 3α
x1 − 3x2 = α
x2 = 1
0 = 5 + 3α
1 2 8 h 1 2 8 h
−2 R2 + 2R1 → R2
3 7 k 0 7 23 k + 2h
R3 − 3R1 → R3
3 −1 2 l 0 −7 −22 l − 3h
1 2 8 h
R3 + R2 → R3 0 7 23 k + 2h
0 0 1 l+k−h
1 2 0 −8l − 8k + 9h
R1 − 8R3 → R1 0 7 0 −23l − 22k + 25h
R2 − 23R3 → R2
0 0 1 l+k−h
1 2 0 −8l − 8k + 9h
1
R → R2 0 1 0 − 23 l − 22 k + 25 h
7 2 7 7 7
0 0 1 l+k−h
0 0 − 10 l − 12 k + 13
1 7 7 7
h
R1 − 2R2 → R1 0 1 0 − 23 l − 22 k + 25 h
7 7 7
0 0 1 l+k−h
x1 = − 10
7
l− 12
7
k + 13
7
h
x2 = − 23
7
l− 22
7
k + 25
7
h
x3 = l + k − h,
it follows that b is a linear combination of the columns of A, and since this holds for
an arbitrary b in R3 , it follows that the columns of A span R3 .
Span{v1 , v2 , · · · , vp } = Span{v1 , v2 , · · · , vp , u}
2.2. The Matrix Equation Ax = b 127
6 Rn
Span{v1 , v2 , · · · , vp } =
In words, this theorem says that if the number of vectors in a set is less than
n, that set cannot span Rn . So, to span Rn one needs at least n vectors.
defined as
Ax = x1 a1 + x2 a2 + · · · + xn an
Note. The product Ax is defined if and only if the number of columns of A is
equal to the number of entries (or rows) of x.
(a)
1 3 5 1 3
Ax = =5 +6 By definition
2 4 6 2 4
5 + 18 23
= =
10 + 24 34
(b)
3
−1 0 −3 4 =3 −1 0 −3
Ax = +4 +5
2 4 −5 2 4 −5
5
−3 + 0 − 15 −18
= =
6 + 16 − 25 −3
(d)
1 −1 1 −1 7+2 9
7
Ax = 2 −3 = 7 2 − 2 −3 = 14 + 6 = 20
−2
3 0 3 0 21 − 0 21
Matrix Equation
x1
A matrix equation in one variable x = ... is an equation of the form
xn
Ax = b
where
7x1 − 6x2 = 13
(a) For the system , the corresponding matrix equation is
−8x1 + 9x2 = 5
Ax = b, where
7 −6 x1 13
A= , x= , b=
−8 9 x2 5
x1 + x2 − x3 = 0
2x1 + x3 = 1
3x2 − 4x3 = 7
is Ax = b, where
1 1 −1 x1 0
A= 2 0 1 , x = x2 , b= 1
0 3 −4 x3 7
x1 + 3x2 − 4x3 = 0
−5x1 + 2x2 + x3 = −3
6x1 − x2 − 17x3 = 8
− 2x3 + 3x4 + x5 = 4
2x1 − x2 + 3x3 + 4x5 = 23
x1 + x2 + 12x3 = 0
−3x1 − 2x2 − 9x4 + 9x5 = 32
a vector equation, or as
a matrix equation.
The following theorem says that whatever point of view you choose, you will
get the same solution set.
x1 a1 + x2 a2 + · · · + xn an = b,
and
(a) Thanks to Theorem 2.2.1, solving the equation Ax = b amounts to solving the
linear system whose augmented matrix is
4 −10 10 38
A b = 3 −5 15 16
3 −6 12 21
Reducing this, we get
4 −10 10 38 4 −10 10 38
3 −5 15 16 1
R
3 3
→ R3 3 −5 15 16
3 −6 12 21 1 −2 4 7
1 −2 4 7
R1 ↔ R3 3 −5 15 16
4 −10 10 38
1 −2 4 7
R2 − 3R1 → R1 0 1 3 −5
R3 − 4R1 → R1
0 −2 −6 10
1 −2 4 7
R3 + 2R2 → R3 0 1 3 −5
0 0 0 0
1 0 10 −3
R1 + 2R2 → R1 0 1 3 −5
0 0 0 0
The linear system corresponding to the reduced echelon form is
x1 10x3 = −3
x2 + 3x3 = −5
Set x3 = s. Then x1 = −3 − 10s and x2 = −5 − 3s. So the solutions to the
matrix equation Ax = b are vectors of the form
−3 − 10s
x = −5 − 3s
s
where s is a real number. (So the equation Ax = b has infinitely many solutions.)
2.2. The Matrix Equation Ax = b 131
(b) According to Theorem 2.2.1, the vector equation x1 a1 + x2 a2 + x3 a3 = b has the
same solution set as the linear system whose augmented matrix is a1 a2 a3 b .
So, using the result from part (a), the general solution to the equation x1 a1 +
x2 a2 + x3 a3 = b is
x1 = −3 − 10s
x2 = −5 − 3s
x3 = s
where s can be any real number.
6. [Exercise on page 15]
(a) We first need to reduce the augmented matrix.
−1 4 b1 1 −4 −b1
(−1)R1 → R1
2 −8 b2 2 −8 b2
1 −4 −b1
R2 − 2R1 → R2
0 0 b2 + 2b1
If the variables are x1 and x2 , the corresponding linear system will be
x1 − 4x2 = −b1
0 = b2 + 2b1
So the system has a solution if and only if 2b1 + b2 = 0. Thus, the system does
not have a solution for all possible b1 and b2 because some choices of b1 and b2 can
make 2b1 + b2 nonzero. For example, if b1 = 0 and b2 = 1, then 2b1 + b2 = 1 6= 0.
(b) From part (a), the system Ax = b does have a solution if and only if the entries
of b satisfy 2b1 + b2 = 0. This is the equation of a line through the origin in R2 .
To be more precise, we have the following description:
b1 b1
= Because b2 = −2b1
b2 −2b1
1
= b1 Factoring out b1
−2
So the system Ax = b has a solution
if and only if the vector b lies on the line
1
in R2 spanned by the vector . (That line is shown below.)
−2
b2
−2 −1 1 2 b1
−2
−4
132 Chapter 2. Vectors in Rn and Linear Systems
Clearly,
every
column of A is a pivot column except the right most one. But, for
0
b = 0 , the system Ax = b does not have a solution.
1
Since this latter matrix has three pivot columns, it follows that the matrix A
has three pivot positions ((1, 1), (2, 2), and (3, 3)).
134 Chapter 2. Vectors in Rn and Linear Systems
(b) No. Indeed, from part (a), an echelon form of A does not have a pivot in
every row (actually there is a pivot in every row except the fourth one). So the
statement (iii) from Theorem 2.2.2 is false for this matrix A. This implies that
the statement (i) is also false, that is, the equation Ax = b does not have a
solution for every b in R4 .
(c) To find the number of pivot positions of the matrix B, we need to put it into an
echelon form.
1 0 0 0 1 0 0 0
0 1 0
1
R3 + 3R1 → R3 0
1 0 1
−3 0 1 −1 R4 − 4R1 → R4 0 0 1 −1
4 0 0 1 0 0 0 1
From this latter matrix, it follows that the number of pivot positions of B is 4.
(d) Yes. Since any two echelon forms of B have the same pivot positions (thanks
to Theorem 1.2.2) and since the echelon form above has four pivot positions, it
follows that any echelon form of B has four pivot positions. Since this number
is equal to the number of rows of B, it follows that any echelon form of B has
a pivot in every row. So the statement (iii) from Theorem 2.2.2 is true for the
matrix B. Therefore the statement (ii) from the same theorem is also true. So
the columns of B span R4 .
(e) Yes. As we saw above, any echelon form of A does not have a pivot in every row
(as there is no pivot in the fourth row). So the columns of A do not span R4 .
Therefore, there exists at least one vector in R4 which is not a linear combination
of the columns of A.
Alternate Solution
By row reducing the coefficient matrix (not the augmented matrix), we
get
1 0 0
0 1 0
0 0 1
Since this matrix has a pivot in every row, it follows that the system
Ax = b has a solution for all possible b1 , b2 , b3 (thanks to Theorem 2.2.2
and Theorem 1.2.2).
(b) Yes. Indeed, from part (a), the system Ax = b has a solution for each b in R3 .
This implies (by Theorem 2.2.2) that the columns of A span R3 .
(b)
3
−1 0 −3 4 = (−1)(3) + 0(4) + (−3)(5)
2 4 −5 2(3) + 4(4) + (−5)(5)
5
−3 + 0 − 15 −18
= =
6 + 16 − 25 −3
136 Chapter 2. Vectors in Rn and Linear Systems
(c)
1 −1 1(7) + (−1)(−2) 9
2 −3 7
= 2(7) + (−3)(−2) = 20
−2
3 0 3(7) + 0(−2) 21
(i) A(u + v) = Au + Av
Vector Form
Let x be the general solution to a linear system. Writing x in vector form
amounts to writing x as a linear combination of a set of vectors.
x1 = 3t
(a) A vector form of x2 = −4t is
x3 = t
x1 3t
x = x2 = −4t
x3 t
2.3. Writing the General Solution of a Linear System in Vector Form 137
3
= t −4 Factor out t
1
(b)
x1 5 + 6t
x = x2 = 10 − 8t
x3 t
5 6t
= 10 + −8t Split x into two vectors
0 t
5 6
= 10 + t −8 Factor out t
0 1
(c)
x1 1 + s + 9t
x2 −4 + 13t
x3 =
x=
s
x4 t
1 s 9t
−4 0 13t
= 0 + s + 0
Split x into three vectors
0 0 t
1 1 9
−4
+ s 0 + t 13
= 0 1 0 Factor out s and t
0 0 1
(d)
x1 4s − 7t
x2
s
x=
x3 =
−2 + 5t
x4 −3
x5 t
0 4s −7t
0 s
0
=
−2 + 0 + 5t
Split x into three vectors
−3 0 0
0 0 t
0 4 −7
0
1
0
=
−2 + s 0 + t
5
Factor out s and t
−3 0 0
0 0 1
138 Chapter 2. Vectors in Rn and Linear Systems
(e)
x1 24 + 3s1 − 8s2 + 9s3
x2
s1
x
x3 =
−7 + 15s2 + 6s3
x4 s2
x5 s3
24 3s1 −8s2 9s3
0
s1
0 0
=
−7 + 0 + 15s2 + 6s3
0 0 s2 0
0 0 0 s3
24 3 −8 9
0
1
0
0
=
−7 + s1 0 + s2 15 + s3 6
0 0 1 0
0 0 0 1
Homogeneous System
A linear system Ax = b is called homogeneous if b is the zero vector. So, a
homogeneous linear system is an equation of the form Ax = 0.
Trivial Solution
Let Ax = 0 be a homogeneous system. Clearly, if x is the zero vector, then
Ax = 0. So the zero vector is a solution to the equation
Ax = 0. In other
x1
x2
words, if A is an m × n matrix, the vector x = .. where
.
xn
x1 = 0, x2 = 0, · · · , xn = 0
Since the system has infinitely many solutions, we can find nontrivial solutions.
For example,
12 12
if s = 1, then x = 1 −5 = −5 is a nontrivial solution.
1 1
12 24
If s = 2, then x = 2 −5 = −10 is also a nontrivial solution.
1 2
140 Chapter 2. Vectors in Rn and Linear Systems
x4 t 0 t
1 2
1 0
= s
0 + t −2
0 1
where s and t can be any real numbers. This shows that the solution set is the set of
linear combinations of the vectors
1 2
1 0
a1 = 0 and a2 = −2
0 1
In other words, the solution set is Span{a1 , a2 }.
6. [Exercise on page 17]
False. Suppose the solution set to a nonhomogeneous linear system Ax = b (b 6= 0)
is the span of some vectors a1 , a2 , · · · , ap . Then every linear combination
x 1 a1 + x 2 a2 + · · · + x p ap
where s can be any real number. For each value of s, the vector x lies in Rn = R3
(since the vector x has three entries). So the solution set is not a subset of Rm = R2 .
Note. In general the solution set to a system Ax = b, where A is an m × n matrix,
is a subset of Rn .
8. [Exercise on page 17]
True. It is true because of the following theorem.
P is a solution to Ax = b, and
1 −2 3 1 −2 3
R2 + 4R1 → R2
−4 8 −12 0 0 0
3 2
where P = and u = . Geometrically, this means that a solution to
0 1
the equation Ax = b is the vector P plus a multiple of the vector u. In other
words, the solution set to Ax = b is the line through the point (3, 0) parallel
to the solution to Ax = 0 as shown below. This amounts to saying that the
solution set to Ax = b is obtained by translating the solution set to Ax = 0 to
the right 3.
x2
Ax = 0
Ax = b
su
P + su
u
−3 P 7 x1
−3
x2
4
−3 4 x1
−3 L
0
(a) False. Since the line L passes through the origin, it contains the vector .
0
0
So if (S) is a linear system whose solution set is L, then will be a solution
0
to (S). Therefore, (S) will be a homogeneous system.
(b) To find two nontrivial solutions,it suffices
to find
two nonzero vectors that lie
−2 2
in L. For example, the vectors and are nontrivial solutions to
3 −3
the linear system whose solution set is L.
−2
(c) In part (b), we saw that the vector u = is a solution. Actually, any
3
other solution is alinearcombination of u (or a multiple of u), that is, a vector
−2 x1
of the form x = s , where s is a real number. If x = , then
3 x2
x1 = −2s
x2 = 3s
x2
Solving the second equation for s, we find s = 3
. Substituting this into the first
equation, we get
x
2
x1 = −2
3
3x1 = −2x2 Multiply both sides by 3
3x1 + 2x2 = 0 Adding 2x2 to both sides
3x1 + 2x2 = 0
0 = 0
is L.
144 Chapter 2. Vectors in Rn and Linear Systems
Alternate Approach
From the graph, one can see that the slope of the line L is m = − 32 . So
an equation of L is of the form
3
x2 = mx1 + b = − x1 + b
2
Since L passes through the origin, it follows that b = 0. So the equation
of L is x2 = − 32 x1 . Multiplying both sides by 2, we get 2x2 = −3x1 or
3x1 + 2x2 = 0. We thus obtain the same equation as above.
147 −1 −4
0
1
0
where P =
−101 , u =
0 , and v =
3 .
0 0 1
11 0 0
12. [Exercise on page 18]
1
1 0 3
0
R1 + R2 → R1 0 1 − 35 0
0 0 0 0
A vector form is
1 1
−3t −3
x1
x = x2 = 3 t = t 53 = tv,
5
x3 t 1
− 13
5
where v = .
3
1
146 Chapter 2. Vectors in Rn and Linear Systems
1 3 4 1 1 3 4 1
−2 1 5 R2 + 2R1 → R2
2 0 7 13 4
R3 − 7R1 → R3
7 7 2 −1 0 −14 −26 −8
1 3 4 1
R3 + 2R2 → R3 0 7 13 4
0 0 0 0
1 3 4 1
1
R
7 2
→ R2 0 1 13 7
4
7
0 0 0 0
1 0 − 11
7
− 5
7
13 4
R1 − 3R2 → R1 0 1
7 7
0 0 0 0
The variables x1 and x2 are the leading variables while x3 is free. Set x3 = t.
Then the general solution is
x1 = − 57 + 11
7
t
4 13
x2 = 7
− 7
t
x3 = t
A vector form is
5 11 5 11
−7 + 7 t −7
x1 7
x = x2 = 74 − 13
4 13
t = + t − = P + tv,
7 7 7
x3 t 0 1
5 11
−7 7
where P = 47 and v = − 13 .
7
0 1
(c) Solution. As usual, we first need to reduce the augmented matrix.
1 2 1 5 −4 1 2 1 5 −4
R2 − 2R1 → R2
2 4 3 7 5 0 0 1 −3 13
R3 + 3R1 → R3
−3 −6 −5 −9 −14 0 0 −2 6 −26
2.3. Writing the General Solution of a Linear System in Vector Form 147
1 2 1 5 −4
R3 + 2R2 → R3 0 0 1 −3 13
0 0 0 0 0
1 2 0 8 −17
R1 − R2 → R1 0 0 1 −3 13
0 0 0 0 0
The corresponding system is
x1 + 2x2 + 8x4 = −17
x3 − 3x4 = 13
0 = 0.
The leading variables are x1 and x3 , and the others are free. Set x2 = s and
x4 = t. Then the general solution in vector form is
−17 − 2s − 8t −17 −2 −8
s 0
+ s 1 + t 0 = P + su + tv,
x= =
13 + 3t 13 0 3
t 0 0 1
−17 −2 −8
0
1 , v = 0 , and s, t can be any real
where P = 13 , u = 0 3
0 0 1
numbers.
(d) Reducing the augmented matrix, we get
1 −2 −3 4 −9 1 −2 −3 4 −9
1 −1 −4 R2 − R1 → R2
10 9 0 1 −1 6 18
R3 + 2R1 → R3
−2 3 7 −14 0 0 −1 1 −6 −18
1 −2 −3 4 −9
R3 + R2 → R3 0 1 −1 6 18
0 0 0 0 0
1 0 −5 16 27
R1 + 2R2 → R1 0 1 −1 6 18
0 0 0 0 0
The corresponding system is
x1 − 5x3 + 16x4 = 27
x2 − x3 + 6x4 = 18
0 = 0
The leading variables are x1 and x2 , and the free ones are x3 and x4 . Set x3 = s
and x4 = t. Then the general solution in vector form is
27 + 5s − 16t 27 5 −16
18 + s − 6t 18
+ s 1 + t −6 = P + su + tv,
x= =
s 0 1 0
t 0 0 1
148 Chapter 2. Vectors in Rn and Linear Systems
27 5 −16
, v = −6 , and s, t can be any real numbers.
18 1
where P =
0
, u =
1 0
0 0 1
(b)
1 −2 3 1 −3 1 −2 3 1 −3
−1 R2 + R1 → R2
2 −2 1 5 0 0 1 2 2
R3 − 2R1 → R3
2 −4 5 0 −8 0 0 −1 −2 −2
1 −2 3 1 −3
R3 + R2 → R3 0 0 1 2 2
0 0 0 0 0
1 −2 0 −5 −9
R1 − 3R2 → R1 0 0 1 2 2
0 0 0 0 0
x1 − 2x2 − 5x4 = −9
x3 + 2x4 = 2
0 = 0
The leading variables are x1 and x3 , and the free ones are x2 and x4 . Set x2 = s
and x4 = t. Then the general solution is
x1 = −9 + 2s + 5t
x2 = s
x3 = 2 − 2t
x4 = t
−9 2 5
0
+ s 1 + t 0 = P + su + tv,
= 2 0 −2
0 0 1
−9 2 5
0 1 0
where P =
2 , u = 0 , and v = −2 .
0 0 1
−9
0
(e) A particular solution to the equation Ax = b is the vector P =
2 .
0
(f ) Yes. Indeed, a particular solution is P , and the general solution to the associated
homogeneous system Ax = 0 is su + tv where s and t can be any real numbers.
When s = 1 and t = 0, the vector 1u + 0v = u is a nontrivial solution to the
equation Ax = 0.
So
x1 = s
x2 = t
x3 = s + t
One can easily verify that this homogeneous system has the desired properties (three
equations and three variables, and the solution set is the plane spanned by u and v).
So
x1 = 3 − 4s
x2 = −1 + 2s
x3 = s
As before, the idea is to find one equation (or more) involving only the variables.
x1 + 4x3 = 3
x2 − 2x3 = −1
0 = 0
x1
x2
Let x =
x3 be a solution. Then
x4
x1 2 −3 7 2 − 3s + 7t
x2 0 1 0 s
x3 = 5 + s 0
+ t
−1 =
5−t
x4 0 0 1 t
So
x1 = 2 − 3s + 7t
x2 = s
x3 = 5−t
x4 = t
Substituting s with x2 and t with x4 into the first equation, we have x1 =
2 − 3x2 + 7x4 . This latter equation is equivalent to x1 + 3x2 − 7x4 = 2.
Substituting t with x4 into the third equation, we get x3 = 5 − x4 . This gives
the equation x3 + x4 = 5.
So the following linear system has the desired properties.
x1 + 3x2 − 7x4 = 2
x3 + x4 = 5
0 = 0
Linear Independence
x1 v1 + x2 v2 + · · · xp vp = 0
1
(a) We want to determine whether the set {v1 , v2 }, where v1 = and v2 =
−1
2
, is linearly independent. Let x1 and x2 be scalars (real numbers) such
3
that x1 v1 + x2 v2 = 0. Then
1 2 0
x1 + x2 =
−1 3 0
x1 + 2x2 0
This is equivalent to = , which gives the following system
−x1 + 3x2 0
of linear equations
x1 + 2x2 = 0
−x1 + 3x2 = 0
To solve this, we need to reduce the augmented matrix as usual.
1 2 0 1 2 0
R2 + R1 → R2
−1 3 0 0 5 0
1 1 2 0
R → R2
5 2 0 1 0
1 0 0
R1 − 2R2 → R1
0 1 0
x1 = 0
x2 = 0
So the system has only the trivial solution (0, 0). This means that the set {v1 , v2 }
is linearly independent.
We now want to find a specific linear dependence relation among v1 and v2 . Any
(nonzero) choice for s results in a choice of a dependence relation for v1 and
v2 . For example, if s = 1, then x1 = 3 and x2 = 1. This gives the relation
3v1 + v2 = 0.
Alternate Solution.
By inspection, we see that v2 is a multiple of v1 (v2 = −3v1 ). This provides
a specific relation among v1 and v2 , and shows that the set {v1 , v2 } is
linearly dependent.
x1 + 3x3 = 0
x2 + 2x3 = 0
The variables x1 and x2 are leading variables, while x3 is a free variable. Set
x3 = s. Then the general solution is
x1 = −3s
x2 = −2s
x3 = s
2x1 + 4x2 = 0
3x1 + 7x3 = 0
−5x2 + 6x3 = 0
2 4 0 0 1 2 0 0
1
3 0 7 0 R
2 1
→ R1 3 0 7 0
0 −5 6 0 0 −5 6 0
1 2 0 0
R2 − 3R1 → R2 0 −6 7 0
0 −5 6 0
1 2 0 0
R2 − R3 → R2 0 −1 1 0
0 −5 6 0
1 2 0 0
(−1)R2 → R2 0 1 −1 0
0 −5 6 0
1 2 0 0
R3 + 5R2 → R3 0 1 −1 0
0 0 1 0
2.4. Linear Independence 155
1 2 0 0
R2 + R3 → R2 0 1 0 0
0 0 1 0
1 0 0 0
R1 − 2R2 → R1 0 1 0 0
0 0 1 0
This matrix has a pivot in every row, but its columns are linearly dependent
since the third column is the sum of the first two columns.
(b) True. This is true because of the following theorem.
(a) We want to determine whether the columns of the following matrix are linearly
independent.
−3 4 27
A = 2 −1 −13
4 7 1
First, we need to put A into an echelon form.
−3 4 27 1 11 28
2 −1 −13 R1 + R3 → R1 2 −1 −13
4 7 1 4 7 1
1 11 28
R2 − 2R1 → R2 0 −23 −69
R3 − 4R1 → R3
0 −37 −111
1 11 28
1
− 23 R2 → R2 0 1 3
0 −37 −111
1 11 28
R3 + 37R2 → R3 0 1 3 =B
0 0 0
The upper left entry of B is a leading entry. So the first column is a pivot
column.
The entry in the second row and second column is also a leading entry. So
the second column is also a pivot column.
The third column of B is not a pivot column because it does not contain a
leading entry.
Since there is a column of B which is not a pivot column, it follows that the
columns of A are not linearly independent (so they are linearly dependent)
thanks to Theorem 2.4.1.
(b)
1 3 −4 1 3 −4
−1 −2 R2 + R1 → R2
9 0 1 5
R3 − R1 → R3
1 2 −8 0 −1 −4
R4 − 3R1 → R4
3 11 −1 0 2 11
1 3 −4
R3 + R2 → R3 0 1
5
R4 − 2R2 → R4 0 0 1
0 0 1
1 3 −4
0 1 5
R4 − R3 → R4
0 0
1
0 0 0
2.4. Linear Independence 157
Since every column of this latter matrix is a pivot column, it follows that the
columns of A are linearly independent (thanks to Theorem 2.4.1).
(a) We want to find h (if any) such that v3 = x1 v1 + x2 v2 for some scalars x1 and
x2 . This amounts to finding h such that the following system has at least one
solution.
x1 = 1
2x1 + x2 = h
3x1 + x2 = 4
We reduce the augmented matrix:
1 0 1 1 0 1
2 1 h R2 − 2R1 → R2 0 1 h−2
R3 − 3R1 → R3
3 1 4 0 1 1
1 0 1
R3 − R2 → R3 0 1 h−2
0 0 −h + 3
1 −2 −1
R3 − (h − 3)R2 → R3 0 0 1
0 0 0
Since there is a non-pivot column (the second column of this latter matrix is not a
pivot column) for every h, it follows that the set {v1 , v2 , v3 } is linearly dependent
for every h.
6. [Exercise on page 21]
(a) False. This is false because of the following result.
v2
v1
−1 v3 3 x
−1
1 −2 1 −8 1 −2 1 −8
−1 R2 + R1 → R2
3 −2 10 0 1 −1 2
R3 + 2R1 → R3
−2 1 2 10 0 −3 4 −6
1 −2 1 −8
R3 + 3R2 → R3 0 1 −1 2
0 0 1 0
1 −2 0 −8
R2 + R3 → R2 0 1 0 2
R1 − R3 → R1
0 0 1 0
1 0 0 −4
R1 + 2R2 → R1 0 1 0 2
0 0 1 0
Since there is a non-pivot column (the fourth column is not a pivot column), it
follows that the columns of A are linearly dependent.
(b) To answer this question, we need to find dependence relationships among v1 , v2 , v3 ,
and v4 . From part (a), the reduced echelon form of the augmented matrix of the
homogeneous equation Ax = 0 is
1 0 0 −4 0
0 1 0 2 0
0 0 1 0 0
x1 −4x4 = 0
x2 + 2x4 = 0
x3 = 0
1 2
(b) False. Indeed, the set {v1 , v2 }, where v1 = and v2 = , is linearly
0 0
dependent because v2 is a multiple of v1 (v2 = 2v1 ), but it does not contain the
zero vector. This example shows that the converse of Theorem 2.4.5 is not true.
(c) True. This is true because of the following theorem.
since the system Ax = 0 has at least one free variable (because the
number of variables, p, is greater than the number of equations, n),
162 Chapter 2. Vectors in Rn and Linear Systems
0
(d) Solution. False. Indeed, the set {v1 }, where v1 = is the zero vector in
0
R2 , is linearly dependent (thanks to Theorem 2.4.5), but p = 1 is not greater
than n = 2. This example shows that the converse of Theorem 2.4.6 is not true.
4 v2
v1
v4
−2 4 x
−2 v3
The bottom row of A is a zero row, but the columns of A are not linearly
dependent because each of them is a pivot column.
(b) True. Indeed, let {v1 , v2 , · · · , vp } be the columns of A. Since the rightmost
column is a zero column, it follows that vp is the zero vector. So the set
{v1 , v2 , · · · , vp } is linearly dependent.
Span{a1 , a2 , a3 } = Span{a1 , a2 }
But Span{a1 , a2 } cannot be equal to R3 because two vectors are not enough to
span the 3-dimensional space R3 . Thus, the columns of A do not span R3 .
Only the first matrix does not have the required property. So all possible reduced
echelon form of a 2 × 2 matrix whose columns are linearly dependent are
1 ∗ 0 1 0 0
, , and .
0 0 0 0 0 0
(a) True. By definition, if the equation Ax = 0 has a nontrivial solution, then the
columns of A are linearly dependent.
(b) False. Indeed, consider the matrix
1 0 0
A=
0 1 0
The columns of A span R2 since A has a pivot in every row. But the columns
of A are not linearly independent because one of the columns is the zero vector
(or because the number of columns is greater than the number of rows).
(c) False. Indeed, consider the following 3 × 2 matrix.
1 0
A= 0 1
0 0
The columns of A are linearly independent since each of them is a pivot column.
But they do not span R3 because there is a row (the third row) that does not
contain a pivot (or because two vectors are not enough to span R3 ).
(d) False. Indeed, consider the following.
1 0 0
A= 0 1 b= 0
0 0 1
As we saw in part (c), the columns of A are linearly independent. But the linear
system Ax = b has no solution since the third equation is of the form 0 = d with
d = 1 6= 0.
Then
x1 u + x1 v + x2 u − x2 v = 0
(x1 + x2 )u + (x1 − x2 )v = 0
Since u and v are linearly independent, the latter equation implies that x1 + x2 = 0
and x1 − x2 = 0. This gives the following system.
x 1 + x2 = 0
x 1 − x2 = 0
Linear Transformations
– T : R → R, T (x) = x2
2 3x
– T : R → R , T (x) =
4x + 5
−x2
x1
– T : R2 → R3 , T = x1 − x2
x2
−3x2 + 7x3
A linear transformation is a function T : Rn → Rm that satisfies the
following two conditions:
T (x + y) = T (x) + T (y)
T (cx) = cT (x)
167
168 Chapter 3. Linear Transformations
So T is a linear transformation.
2. [Exercise on page 25]
x1 y1
Condition (i). Let x = ,y = be vectors in R2 . Then
x2 y2
x1 y1 x1 + y 1
T (x + y) = T + =T
x2 y2 x2 + y 2
3.1. Introduction to Linear Transformations 169
3(x1 + y1 ) 3x1 + 3y1
= 0 = 0
(x1 + y1 ) − (x2 + y2 ) x1 + y 1 − x2 − y 2
3x1 + 3y1
= 0+0
(x1 − x2 ) + (y1 − y2 )
3x1 3y1
= 0 + 0 = T (x) + T (y)
x1 − x2 y1 − y2
Since the two conditions of the definition are met, T is a linear transformation.
3. [Exercise on page 25]
(a) True. This is true because of the following theorem.
Conversely, suppose that the equation (3.1.1) holds for every vectors
x, y in Rn and for every scalars c and k. We want to show that T is
linear.
170 Chapter 3. Linear Transformations
So T is a linear transformation.
T (0) = 0
Proof.
T (0) = T (0 + 0) 0=0+0
T (0) = T (0) + T (0) Since T is linear
T (0) = 2T (0)
T (0) − 2T (0) = 0 Subtract 2T (0) from both sides
−T (0) = 0
T (0) = 0 Multiply both sides by −1
T (1 + 1) 6= T (1) + T (1)
(a)
(b)
(c)
T (w) =T (c1 u + c2 v)
=T (c1 u) + T (c2 v) Because T is linear
=c1 T (u) + c2 T (v) Again because T is linear
This shows that T (w) is a linear combination of T (u) and T (v). In other words, T (w)
is in Span{T (u), T (v)}.
−3c1 + 2c2 = 32
2c1 − c2 = −19
8c1 + 7c2 = 1
1 −1 −13
R2 − 2R1 → R2 0 1 7
R3 − 8R1 → R3
0 15 105
1 −1 −13
R3 − 15R2 → R3 0 1 7
0 0 0
1 0 −6
R1 + R2 → R1 0 1 7
0 0 0
So the system has a unique solution: c1 = −6 and c2 = 7.
(b) Find T (v).
Solution. From part (a), v = −6u1 + 7u2 . Taking T of both sides, we get
T (v) = T (−6u1 + 7u2 )
= −6T (u1 ) + 7T (u2 ) Because T is linear
= −6u01 + 7u02 Because T (u1 ) = u01 and T (u2 ) = u02
5 −3
= −6 −1 + 7 −2
2 15
−30 − 21 −51
= 6 − 14 = −8
−12 + 105 93
10. [Exercise on page 26]
(a) The sentence “T is a linear transformation that maps e1 into v1 and e2 into v2 ”
means that T (e1 ) = v1 and T (e2 ) = v2 . So we know what the transformation
does on e1 and e2 .
12 12
To find T , we first need to write as a linear combina-
−13 −13
tion of e1 and e2 . We have
12 12 0 1 0
= + = 12 − 13 = 12e1 − 13e2
−13 0 −13 0 1
Now we have
12
T = T (12e1 − 13e2 ) = 12T (e1 ) − 13T (e2 )
−13
−1 3 −12 − 39 −51
= 12 − 13 = =
2 4 24 − 52 −28
(b)
x1 1 0
T (x) = T =T x1 + x2 = T (x1 e1 + x2 e2 )
x2 0 1
−1 3 −x1 + 3x2
= x1 T (e1 ) + x2 T (e2 ) = x1 + x2 =
2 4 2x1 + 4x2
3.1. Introduction to Linear Transformations 175
b1
(b) Here b = b2 with b1 = −6, b2 = −7, and b3 = 13. Using the formula we
b3
obtained in part (a), we have
−28b1 − 10b2 + 6b3
T (b) = −7b1 − 2b2 + 2b3
−98b1 − 31b2 + 23b3
−28(−6) − 10(−7) + 6(13) 316
= −7(−6) − 2(−7) + 2(13) = 82
−98(−6) − 31(−7) + 23(13) 1104
So T is a linear transformation.
The transformation T defined by T (x) = Ax is called a matrix trans-
formation.
(e) False. Indeed, the product of A by a vector x is defined if the number of columns
of A equals the number of entries of x. In that case, the number of entries of Ax
corresponds to the number of rows of A. So, if A has 7 rows and 11 columns,
the product Ax is defined if the vector x has 11 entries, that is, x is a vector in
R11 . Thus, if A is of size 7 × 11 and T : Ra → Rb is defined by T (x) = Ax, then
a = 11 and b = 7. In general, we have the following.
(f ) False. Indeed,
since the number of rows of A corresponds to the the dimension of the
178 Chapter 3. Linear Transformations
(a)
1 −3 1(2) − 3(6)
2 2 − 18 −16
T (u) = Au = = = =
5 −17 5(2) − 17(6)
6 10 − 102 −92
1 −3 −7 1(−7) − 3(8) −31
T (v) = Av = = =
5 −17 8 5(−7) − 17(8) −171
(b) To find a vector x in R2 such that T (x) = b, we need to solve the system Ax = b.
1 −3 4 1 −3 4
R2 − 5R1 → R2
5 −17 12 0 −2 −8
1 1 −3 4
− 2 R2 → R2
0 1 4
1 0 16
R1 + 3R2 → R1
0 1 4
16
So the system Ax = b has a unique solution: x = .
4
(a) By definition,
1 −2 x1 − 2x2
x1 x1 x1
T (x) = T =A = 3 1 = 3x1 + x2
x2 x2 x2
1 7 x1 + 7x2
x1 − 2x2 = 7
3x1 + x2 = 7
x1 + 7x2 = −11
3.1. Introduction to Linear Transformations 179
(a) To find a vector x whose image under T is y, we need to solve the equation
T (x) = y, that is, the equation Ax = y.
1 −1 1 1 1 −1 1 1
−2 R2 + 2R1 → R2
2 −2 1 0 0 0 3
R3 − 3R1 → R3
3 −4 −1 1 0 −1 −4 −2
1 −1 1 1
R2 ↔ R3 0 −1 −4 −2
0 0 0 3
1 −1 1 1 1 −1 1 1
−2 R2 + 2R1 → R2
2 −2 −2 0 0 0 0
R3 − 3R1 → R3
3 −4 −1 4 0 −1 −4 1
180 Chapter 3. Linear Transformations
1 −1 1 1
R2 ↔ R3 0 −1 −4 1
0 0 0 0
1 −1 1 1
−R2 → R2 0 1 4 −1
0 0 0 0
1 0 5 0
R1 + R2 → R1 0 1 4 −1
0 0 0 0
x1 + 5x3 = 0
x2 + 4x3 = −1
x1 = −5s
x2 = −1 − 4s
x3 = s
1 −1 1 b1
R2 ↔ R3 0 −1 −4 b3 − 3b1
0 0 0 b2 + 2b1
1 −1 1 b1
−R2 → R2 0 1 4 −b3 + 3b1
0 0 0 b2 + 2b1
1 0 5 4b1 − b3
R1 + R2 → R1 0 1 4 3b1 − b3
0 0 0 b2 + 2b1
So the equation T (x) = b is consistent if and only if
b2 + 2b1 = 0
Solving this latter equation for b2 , we get b2 = −2b1 . So
b1 b1 b1 0
b = b2 = −2b1 = −2b1 + 0
b3 b3 0 b3
1 0
= b1 −2 + b3 0
0 1
b1
So the set of all vectors b = b2 for which the equation T (x) = b has a
b3
3
solution is the plane in R generated by the vectors
1 0
−2 and 0
0 1
x1
(e) To describe the set of all x = x2 for which T (x) = 0, we need to solve the
x3
equation Ax = 0. Using the same elementary operations as in the part (d), the
reduced echelon form of the augmented matrix is
1 0 5 0
0 1 4 0
0 0 0 0
Let x3 = s. Then the general solution to the equation Ax = 0 is
x1 −5s −5
x = x2 = −4s = s −4
x3 s 1
where s can be any real number. So the set
of all
vectors x such that T (x) = 0
−5
3
is the line in R generated by the vector −4 .
1
182 Chapter 3. Linear Transformations
If 0 ≤ r ≤ 1, T is called a contraction.
T (v) T (u)
v u
T (x) T (w)
x w x1
x1
(b) Let x = be a vector in R2 . Then
x2
x1 −1 0 x1 −x1 + 0x2 −x1
T (x) = T = Ax = = =
x2 0 1 x2 0x1 + 1x2 x2
x1 −x1
So the transformation T maps to . Geometrically, this means
x2 x2
that T is the reflection through the x2 -axis. The effect of T on some vectors is
shown below.
3.2. The Matrix of a Linear Transformation 183
x2
w T (w)
T (v)
v
T (u) u x1
T (x) x
x1
(c) Let x = be a vector in R2 . Then
x2
0 0 x1 0x1 + 0x2 0
T (x) = Ax = = =
0 1 x2 0x1 + 1x2 x2
x1 0
So the transformation T maps to . Geometrically, this is the
x2 x2
projection onto the x2 -axis. The effect of T on some vectors is shown below.
x2
v
T (v)
T (u) u
x1
w T (w)
x1
(d) Let x = be a vector in R2 . Then
x2
0 1 x1 0x1 + 1x2 x2
T (x) = Ax = = =
1 0 x2 1x1 + 0x2 x1
x1 x2
So T is maps to . Geometrically, this is the reflection through
x2 x1
the line x2 = x1 . The effect of T on some vectors is shown below.
184 Chapter 3. Linear Transformations
x2
T (w) x2 = x1
v
w
T (u)
u T (v) x1
T (x) x
Sketching the Graph of the Image of the Unit Square Under a Linear
Transformation
u1 v1
Let u = and v = be two vectors in R2 . The line L connecting
u2 v2
u and v is the line through the points (u1 , u2 ) and (v1 , v2 ) as shown in the
following figure.
x2
v L
v2
u
u2
u1 v1 x1
If A = (x1 , x2 ) and B = (y1 , y2 ) are two points in the plane, the vector from
A to B, denoted AB, ~ is defined by
~ y 1 − x1
AB =
y 2 − x2
Theorem 3.2.1. We have the following.
The set of points on the line L is the same as the set of points
The set of points on the line segment connecting u and v is the same as
the set of points
Proof. Let M = (x1 , x2 ) be a point on the line L. Let A = (u1 , u2 ) be the point
corresponding to the vector u, and let B = (v1 , v2 ) be the point corresponding
to the vector v. (See figure below.)
x2
M
v
v2
B
u
u2
A
u1 v1 x1
x1 − u1 = t(v1 − u1 )
x2 − u2 = t(v2 − u2 )
From the equality x1 − u1 = t(v1 − u1 ), it follows that
So
x1 (1 − t)u1 + tv1 u1 v1
= = (1 − t) +t = (1 − t)u + tv
x2 (1 − t)u2 + tv2 u2 v2
Clearly, the point M = (x1 , x2 ) is between A and B if and only if 0 ≤ t ≤ 1.
Thanks to Theorem 3.2.1, we can prove the following result, which says that a
linear transformation maps a line to a line.
Theorem 3.2.2. Let T : R2 → R2 be a linear transformation. Let u and v be
two vectors in R2 , and let L be the line segment connecting u and v. Then the
image of L under T , T (L), is the line segment connecting T (u) and T (v).
x1
Proof. Let x = be a vector in R2 such that the corresponding point
x2
(x1 , x2 ) is on the line segment connecting u and v. Then, by Theorem 3.2.1,
there exists a number 0 ≤ t ≤ 1 such that
x = (1 − t)u + tv
186 Chapter 3. Linear Transformations
So the point corresponding to the vector T (x) is on the line segment connecting
the vectors T (u) and T (v). This ends the proof.
Thanks to Theorem 3.2.2, to find the image of the unit square under a linear
transformation, all we have to do is to find the image of each corner.
3 0
(a) T (x) = Ax where A =
0 3
Let S be the
unit
square
as shown
below.
To find the
imageS under T , we need
0 1 1 0
to find T ,T ,T , and T .
0 0 1 1
0 0 0
T =A =
0 0 0
1 1 3 0 1 3
T =A = =
0 0 0 3 0 0
1 1 3 0 1 3
T =A = =
1 1 0 3 1 3
0 0 3 0 0 0
T =A = =
1 1 0 3 1 3
So T (S) is the square whose corners are (0, 0), (3, 0), (3, 3), and (0, 3) as shown
below.
T (S)
3 3
2 2
S T
1 1
1 2 3 1 2 3
−2 0
(b) T (x) = Ax where A =
0 3
0 0 0
T =A =
0 0 0
1 1 −2 0 1 −2
T =A = =
0 0 0 3 0 0
3.2. The Matrix of a Linear Transformation 187
1 −21 −2 0 1
T =A = =
1 3
1 0 3 1
0 0 −2 0 0 0
T =A = =
1 1 0 3 1 3
So T (S) is the rectangle whose corners are (0, 0), (−2, 0), (−2, 3), and (0, 3) as
shown below.
T (S)
3 3
2 2
S T
1 1
1 2 3 −2 −1
1 2
(c) T (x) = Ax where A =
0 1
0 0 0
T =A =
0 0 0
1 1 1 2 1 1
T =A = =
0 0 0 1 0 0
1 1 1 2 1 3
T =A = =
1 1 0 1 1 1
0 0 1 2 0 2
T =A = =
1 1 0 1 1 1
So T (S) is the parallelogram whose corners are (0, 0), (1, 0), (3, 1), and (2, 1) as
shown below.
3 3
2 2
S T T (S)
1 1
1 2 3 1 2 3
This transformation is called a shear transformation.
Shear Transformations
Let A be an n × n matrix (a square matrix). Let In be the identity matrix.
188 Chapter 3. Linear Transformations
For example,
1 0 0 0
1 0 0
1 0 0 1 0 0
I1 = [1], I2 = , I3 = 0 1 0 ,
I4 =
0 1 0 0 1 0
0 0 1
0 0 0 1
1 0
(d) T (x) = Ax where A =
−3 1
0 0 0
T =A =
0 0 0
1 1 1 0 1 1
T =A = =
0 0 −3 1 0 −3
1 1 1 1 0 1
T =A = =
1 −2 1 −3 1 1
0 0 1 0 0 0
T =A = =
1 1 −3 1 1 1
So T (S) is the parallelogram whose corners are (0, 0), (1, −3), (1, −2), and (0, 1)
as shown below.
1
3
2 1 2 3
S T −1 T (S)
1
−2
1 2 3 −3
−2 −1
(e) T (x) = Ax where A =
1 2
3.2. The Matrix of a Linear Transformation 189
0 0 0
T =A =
0 0 0
1 1 −2 −1 1 −2
T =A = =
0 0 1 2 0 1
1 1 −2 −1 1 −3
T =A = =
1 1 1 2 1 3
0 0 −2 −1 0 −1
T =A = =
1 1 1 2 1 2
So T (S) is the parallelogram whose corners are (0, 0), (−2, 1), (−3, 3), and (−1, 2)
as shown below.
3 3
T (S)
2 2
S T
1 1
1 2 3 −3 −2 −1 1
" π
π
#
cos 4
− sin 4
(f ) T (x) = Ax where A = π
π
sin 4
cos 4
0 0 0
T =A =
0 0 0
" # " # " √ #
π π
π 2
cos − sin
1 1 4 4 1 cos 4
T =A = = = √2
0 0 π
π
0 π 2
sin 4
cos 4
sin 4 2
" π
π
#
cos − sin
T
1
=A
1
= 4 4 1
= √0
1 1 π π 1 2
sin 4
cos 4
" # " √ #
π π
− 2
cos − sin
0 0 4 4 0 2
T =A = = √
1 1 π π 1
2
sin 4
cos 4 2
2 2
S T T (S)
1
1 2 3 −2 −1 1 2
190 Chapter 3. Linear Transformations
π
Here T is the counterclockwise rotation through 4
about the origin.
when n = 1, e1 = 1 .
1 0
For n = 2, e1 = and e2 = .
0 1
1 0 0
For n = 3, e1 = 0 , e2 = 1 , and e3 = 0 .
0 0 1
Etc.
So
T (x) = T (x1 e1 + x2 e2 + · · · + xn en )
= x1 T (e1 ) + x2 T (e2 ) + · · · + xn T (en ) Since T is linear
3.2. The Matrix of a Linear Transformation 191
x1
x2 By definition of the
= T (e1 ) T (e2 ) · · · T (en ) product of a matrix by
..
.
a vector
xn
= Ax Where A = [T (e1 ) T (e2 ) · · · T (en )]
Multiplying B by e1 , we get
1
0
Be1 = b1 b2 · · · bn = 1b1 + 0b2 + · · · + 0bn = b1
..
.
0
3x1
(a) For the linear transformation T : R → R2 defined by T x1 = ,
−5x1
n = 1 and e1 = 1 . So the standard matrix is A = T (e1 ) . But
3(1) 3
T (e1 ) = T 1 = = .
−5(1) −5
3
Thus, the standard matrix of T is A = .
−5
(b) Since the domain of this is R2 , we have that n = 2 and
1 0
e1 = and e2 =
0 1
0 7(0) − 1 −1
T (e2 ) = T = =
1 3(1) 3
5(0) 0
0
T (e2 ) = T = 0 = 0
1 0−1
3
− 31
5 0
Thus, the standard matrix is A = 0
0
1
3
− 13
(e) We need to find T (e1 ), T (e2 ), and T (e3 ), where
1 0 0
e1 = 0 , e2 = 1 , and e3 = 0
0 0 1
3.2. The Matrix of a Linear Transformation 193
1 1 − 3(0) + 6(0) 1
T (e1 ) = T 0 = −1 + 4(0) − 7(0) = −1
0 2(1) + 5(0) + 0 2
0 0 − 3(1) + 6(0) −3
T (e2 ) = T 1 = −(0) + 4(1) − 7(0) = 4
0 2(0) + 5(1) + 0 5
0 0 − 3(0) + 6(1) 6
T (e3 ) = T 0 = −(0) + 4(0) − 7(1) = −7
1 2(0) + 5(0) + 1 1
x2
T (e2 )
e2
e1
−2 5 x1
T (e1 )
−2
4 1
(a) From the figure, we see that T (e1 ) = and T (e2 ) = . So the standard
−1 3
matrix of T is
4 1
A=
−1 3
1 1 0
(b) First, observe that u = = + = e1 + e2 . So, by using the fact
1 0 1
that T is linear, we find that
4 1 5
T (u) = T (e1 + e2 ) = T (e1 ) + T (e2 ) = + =
−1 3 2
194 Chapter 3. Linear Transformations
Alternate Solution
1
The image of the vector u = under T is
1
4 1 1 5
T (u) = Au = =
−1 3 1 2
T (e2 )
T (u)
e2
e1
−2 5 x1
T (e1 )
−2
x2
T (e1 )
e2
T (e2 )
−2 e1 5 x1
−2
−1 1
(a) From the figure, we see that T (e1 ) = and T (e2 ) = . So the standard
2 1
matrix of T is
−1 1
A=
2 1
(b) First, observe that
−2 −2 0 1 0
u= = + = −2 +3 = −2e1 + 3e2
3 0 3 0 1
So, by using the fact that T is linear, we get
−1 1 5
= −2 +3 =
2 1 −1
T (e1 ) e2
T (e2 )
e1 x1
T (u)
−2T (e1 )
e2
T (e1 )
−2 e1 2 x1
T (e2 )
−2
If we rotate e1through
π
2
(counterclockwise) about the origin, we get e2 . So
0
T (e1 ) = e2 = as shown in the following figure.
1
x2
2
e2 T (e1 )
−2 T (e2 ) e1 2 x1
−2
If we rotate e2 through
π
2
(counterclockwise) about the origin, we get −e1 .
−1
So T (e2 ) = −e1 = .
0
Thus, the standard matrix is
0 −1
A= T (e1 ) T (e2 ) =
1 0
1 0
(c) We need to find T (e1 ), T (e2 ), and T (e3 ), where e1 = 0 , e2 = 1 , and
0 0
0
e3 = 0 . The reflection through the x1 x2 -plane maps every point in the
1
x1 x2 -plane into
the same point. Since e1 and e2 lie in the x1 x2 -plane, we have
1 0
T (e1 ) = e1 = 0 and T (e2 ) = e2 = 1 .
0 0
0
On the other hand, T (e3 ) = −e3 = 0 . Thus, the matrix of T is
−1
1 0 0
A = T (e1 ) T (e2 ) T (e3 ) = 0 1 0
0 0 −1
1 0
(d) We need to find T (e1 ) and T (e2 ), where e1 = and e2 = . The images
0 1
of e1 and e2 under T are shown in the following figure.
3.2. The Matrix of a Linear Transformation 197
x2
e2
T (e2 ) e1 x1
T (e1 ) x2 = −x1
e2
T (e2 )
π
3
π
3
π
3 e1 x1
π
3
T (e1 )
198 Chapter 3. Linear Transformations
Matrix of a Rotation
Theorem 3.2.4. We have the following two statements.
Note. If T is linear, the term “at most one solution” means a unique solution
or no solution. “At least one solution” means a unique solution or infinitely
many solutions.
The term “one-to-one” is also referred to as “injective”. And the term “onto”
is also referred to as “surjective”.
1
From part (c), the equation T (x) = b has at least one solution for b = .
0
1
But T is not onto because, as we saw in part (c), the equation T (x) =
1
does not have at least one solution.
Conversely, suppose that the equation T (x) = 0 has only the trivial
solution and let u, v be vectors in Rn such that T (u) = T (v). Then
T (u) − T (v) = 0, that is, T (u − v) = 0 since T is linear. This latter
equation implies that u − v = 0 or u = v. So, by Theorem 3.3.1, T is
one-to-one.
This ends the proof.
(d) If n > m, then T is not one-to-one. (In other words, if the number of
columns of A is greater than the number of rows, then T is not one-to-
one.)
Proof. (a) Since A is the standard matrix of T , it follows that T (x) = Ax for
every x in Rn .
(c) Suppose that T is one-to-one. Then, from part (a), the columns of A
are linearly independent. This implies (by Theorem 2.4.6) that n (the
number of columns of A) is less than or equal to m (the number of rows
of A).
(e) False. Because the dimension of the domain, n = 3, is greater than the dimen-
sion of the codomain, m = 2 (by Theorem 3.3.3 -(d), if n > m, then T is not
one-to-one).
(f ) True. This is true because of the following result.
The variables x1 and x2 are leading variables and x3 is free. Because there is
a free variable and no equation of the form 0 = d with d 6= 0, the system has
infinitely many solutions, and this holds for every b in R2 . Therefore, according
to the definitions, the transformation T is onto, but not one-to-one.
Since every column is a pivot column, it follows (by Theorem 3.3.3 -(b))
that T is one-to-one.
(d)
5 −20 10 1 −4 2
2 −7 1
9 R
5 1
→ R1 2 −7 9
−4 15 −6 −4 15 −6
1 −4 2
R2 − 2R1 → R2 0 1 5
R3 + 4R1 → R3
0 −1 2
1 −4 2
R3 + R2 → R3 0 1 5 =B
0 0 7
Since every column of B is a pivot column, it follows (by Theorem 3.3.3 -(b))
that T is one-to-one.
Since every row of B contains a pivot, it follows that T is onto thanks to
Theorem 3.3.5 -(b).
206 Chapter 3. Linear Transformations
(e)
1 −2 4 5 0 1 −2 4 5 0
0
1 0 −1 2
R3 + R1 → R3 0
1 0 −1 2
−1 4 −3 −4 4 R4 + 2R1 → R4 0 2 1 1 4
−2 1 −9 −9 −6 0 −3 −1 1 −6
1 −2 4 5 0
R3 − 2R2 → R3 0
1 0 −1 2
R4 + 3R2 → R4 0 0 1 3 0
0 0 −1 −2 0
1 −2 4 5 0
0 1 0 −1 2
R4 + R3 → R4
0
=B
0 1 3 0
0 0 0 1 0
Since there is a non-pivot column (the rightmost column is not a pivot
column), it follows that T is not one-to-one thanks to Theorem 3.3.3 -
(b).
Alternate Solution
Since the number of columns of A is greater than the number of rows,
it follows that T is not one-to-one thanks to Theorem 3.3.3 -(d).
1 −2 −5 b1
−4 9 26 b2 . Performing the same operations as before, we get
5 −9 −19 b3
1 0 7 9b1 + 2b2
0 1 6 b2 + 4b1 (3.3.1)
0 0 0 −9b1 − b2 + b3
This system does not have at most one solution for every b. For example, if
b1 = 0, b2 = 0, b3 = 0, the system has infinitely many solutions:
x1 = −7s
x2 = −6s
x3 = s
where s is a real number. So T is not one-to-one.
Finding
three
different vectors in the domain that have the same image. Let
0
b = 0 . Then, as we saw above, the equation T (x) = 0 has infinitely many
0
solutions.
0
For s = 0, we have the vector u1 = 0 .
0
−7
For s = 1, we have the vector u2 = −6 .
1
−14
For s = 2, we have the vector u3 = −12 .
2
The vectors u1 , u2 , u3 have the same image: T (u1 ) = T (u2 ) = T (u3 ) = 0.
(c) The transformation T is not onto since the equation T(x) = b does
not have at
b1 2
least one solution for every b. For example, for b = b2 = 0 , the equation
b3 0
T (x) = b has no solution since −9b1 − b2 + b3 = −9(2) − 0 + 0 = −18 6= 0.
A vector b is not in the range of T is the equation T (x) = b has no solution.
From the augmented matrix (3.3.1), the equation T (x) = b has no solution if
and only if
−9b1 − b2 + b3 6= 0
For b1 = 1,
b2 = 0, b3 = 0, we have −9b1 − b2 + b3 = −9 6= 0. So the vector
1
v1 = 0 is not in the range of T .
0
0 0
Likewise, the vectors v2 = 1 and v3 = 0 are not in the range of T .
0 1
208 Chapter 3. Linear Transformations
b1
(d) From (3.3.1), a vector b = b2 is in the range of T if and only if
b3
−9b1 − b2 + b3 = 0
Matrix Algebra
If A is a matrix, the entry in the ith row and jth column of A is denoted
aij and is called the (i, j)-entry of A. Notation:
a11 a12 · · · a1n
a21 a22 · · · a2n
A = ..
.. ..
. . .
am1 am2 · · · amn
209
210 Chapter 4. Matrix Algebra
2 4 9
(a) False. Indeed, the matrix A = has 2 rows and 3 columns. So it is of
3 1 8
size 2 × 3.
(b) True. Because the matrix 1 9 7 has 1 row and 3 columns.
2 4 9
(c) False. Indeed, the (1, 2)-entry of is the entry in the first row, second
3 1 8
column, that is, 4.
2 4 9
(d) True. The (2, 3)-entry of is the entry in the second row, third column,
3 1 8
that is, 8.
2 4 9
(e) True. Indeed, the diagonal entries of are a11 = 2 and a22 = 1.
3 1 8
2 4 9
(f ) False. The diagonal entries of 3 1 8 are a11 = 2, a22 = 1, and a33 = 10.
5 7 10
(g) False. Indeed, by definition, a diagonal matrix is a square matrix whose every
nondiagonal entry is 0.
2 5
(h) False. The matrix 1 3 is not a square matrix because the number of rows,
0 0
m = 3, is not the same as the number of columns, n = 2.
0 6
(i) False. The matrix is not a diagonal matrix because there is a nonzero
9 0
nondiagonal entry, a12 = 6 6= 0 (also, a21 = 9 6= 0).
1 0 0
(j) True. The matrix 0 2 0 is a diagonal matrix since it is a square matrix in
0 0 3
which every nondiagonal entry is 0.
0 1
(k) False. Indeed, the matrix is not even a diagonal matrix.
1 0
1 0 0
(l) True. Indeed, the matrix 0 1 0 is a 3 × 3 diagonal matrix in which every
0 0 1
diagonal entry is 1.
(m) False. By definition, the sum of two matrices is defined if and only if they have
the same size.
Matrix Addition
Let A = [aij ] and B = [bij ] be matrices of the same size. Then the sum A + B
is the matrix whose (i, j)-entry is aij + bij . That is,
A + B = [aij + bij ]
(a)
1 7 −2 6
A+B = +
−3 8 5 −14
1 + (−2) 7+6 −1 13
= =
−3 + 5 8 + (−14) 2 −6
(b)
9 −7 17 −9
C + D = −3 0 + −4 1
11 −22 0 19
9 + 17 −7 − 9 26 −16
= −3 − 4 0 + 1 = −7 1
11 + 0 −22 + 19 11 −3
(c) The sum A + C is undefined because A and C do not have the same size.
(d) The sum C + D + B is undefined because D and B (as well as C and B) do not
have the same size.
(e)
Scalar Multiplication
cA = [caij ]
9 −7
For c = 2 and C = −3 0 , we have
11 −22
9 −7 2(9) 2(−7) 18 −14
2C = 2 −3 0 = 2(−3) 2(0) = −6 0
11 −22 2(11) 2(−22) 22 −44
212 Chapter 4. Matrix Algebra
(f )
9 −7 17 −9
3C − 2D = 3 −3 0 − 2 −4 1
11 −22 0 19
27 −21 −34 18
= −9 0 + 8 −2
33 −66 0 −38
27 − 34 −21 + 18 −7 −3
= −9 + 8 0 − 2 = −1 −2
33 + 0 −66 − 38 33 −104
1 7−2 6 1 0
−5A + 2B − 3I2 = −5 +2 −3
−3 8 5 −14 0 1
−5 −35 −4 12 −3 0
= + +
15 −40 10 −28 0 −3
−5 − 4 − 3 −35 + 12 + 0 −12 −23
= =
15 + 10 + 0 −40 − 28 − 3 25 −71
5 −3
Subtracting the matrix from both sides, we get
1 6
−2 8 5 −3 12 − 5 −48 + 3 7 −45
X = −6 − = =
1 0 1 6 −6 − 1 0−6 −7 −6
This is a property of matrix addition and scalar multiplication. Other properties are
listed in the following theorem.
(c) A + 0 = A
(g) A + (−A) = 0
Matrix Multiplication
Let A be an m × n matrix, and let B be an n × p matrix. Let b1 , b2 , · · · , bp be
the columns of B. The product AB is the m × p matrix whose columns are
Ab1 , Ab2 , · · · , Abp , that is,
AB = A b1 b2 · · · bp = Ab1 Ab2 · · · Abp
Note.
The product AB is defined if and only if the number of columns of A
214 Chapter 4. Matrix Algebra
19 22
So AB = .
43 50
(i) True by the result obtained in part (h).
Alternate Solution
One can compute AB directly as follows.
1 2 −3 5 1(−3) + 2(4) 1(5) + 2(−7)
AB = =
3 4 4 −7 3(−3) + 4(4) 3(5) + 4(−7)
−3 + 8 5 − 14 5 −9
= =
−9 + 16 15 − 28 7 −13
From now on, we will compute the product of two matrices this way.
(b)
−3 5 −3(1) + 5(3) −3(2) + 5(4)
1 2
BA = =
4 −7 3 44(1) − 7(3) 4(2) − 7(4)
−3 + 15 −6 + 20 12 14
= =
4 − 21 8 − 28 −17 −20
(c) The product BC is undefined because the number of columns of B is not equal
to the number of rows of C.
(d)
2 0 2(−3) + 0(4) 2(5) + 0(−7)
−3 5
CB = 5 −8 = 5(−3) − 8(4) 5(5) − 8(−7)
4 −7
13 −9 13(−3) − 9(4) 13(5) − 9(−7)
216 Chapter 4. Matrix Algebra
−6 + 0 10 + 0 −6 10
= −15 − 32 25 + 56 = −47 81
−39 − 36 65 + 63 −75 128
(e) The product CD is undefined because the number of columns of C is not equal
to the number of rows of D.
(f )
−2 1 3 2 0
DC = 4 8 9 5 −8
10 −4 5 13 −9
−2(2) + 1(5) + 3(13) −2(0) + 1(−8) + 3(−9)
= 4(2) + 8(5) + 9(13) 4(0) + 8(−8) + 9(−9)
10(2) − 4(5) + 5(13) 10(0) − 4(−8) + 5(−9)
−4 + 5 + 39 0 − 8 − 27 40 −35
= 8 + 40 + 117 0 − 64 − 81 = 165 −145
20 − 20 + 65 0 + 32 − 45 65 −13
(g)
2 0
−2 6 9
CE = 5 −8
3 −7 7
13 −9
2(−2) + 0(3) 2(6) + 0(−7) 2(9) + 0(7)
= 5(−2) − 8(3) 5(6) − 8(−7) 5(9) − 8(7)
13(−2) − 9(3) 13(6) − 9(−7) 13(9) − 9(7)
−4 + 0 12 + 0 18 + 0 −4 12 18
= −10 − 24 30 + 56 45 − 56 = −34 86 −11
−26 − 27 78 + 63 117 − 63 −53 141 54
(h)
2 0
−2 6 9 5 −8
EC =
3 −7 7
13 −9
−2(2) + 6(5) + 9(13) −2(0) + 6(−8) + 9(−9)
=
3(2) − 7(5) + 7(13) 3(0) − 7(−8) + 7(−9)
−4 + 30 + 117 0 − 48 − 81 143 −129
= =
6 − 35 + 91 0 + 56 − 63 62 −7
(i)
−2 1 3 0 9 4
DF = 4 8 9 −2 −2 −5
10 −4 5 1 −10 −1
4.1. Matrix Operations 217
−2(0) + 1(−2) + 3(1) −2(9) + 1(−2) + 3(−10) −2(4) + 1(−5) + 3(−1)
= 4(0) + 8(−2) + 9(1) 4(9) + 8(−2) + 9(−10) 4(4) + 8(−5) + 9(−1)
10(0) − 4(−2) + 5(1) 10(9) − 4(−2) + 5(−10) 10(4) − 4(−5) + 5(−1)
0 − 2 + 3 −18 − 2 − 30 −8 − 5 − 3 1 −50 −16
= 0 − 16 + 9 36 − 16 − 90 16 − 40 − 9 = −7 −70 −33
0+8+5 90 + 8 − 50 40 + 20 − 5 13 48 55
(a) If the product BA is of size 17 × 32, then the size of B is of the form 17 × n,
and the size of A is of the form n × 32 where n is a positive integer. So B has
17 rows.
(b) Since the size of A is of the form n × 32, the number of columns of A is 32.
b1j
an ... = b1j a1 + · · · + bnj an
Abj = a1 · · ·
bnj
This shows that every column of AB is a linear combination of the columns of A.
This proves the following result.
b11 0 0
Ab1 = a1 a2
= b11 a1 +b21 a2 . Since Ab1 = , b11 a1 +b21 a2 = .
b21 2 2
0
From the equation 2a1 − a2 = , it follows that b11 a1 + b21 a2 = 2a1 − a2 . So
2
b11 = 2 and b21 = −1.
Similarly, b12 = −4 and b22 = 3.
2 −4
So B = .
−1 3
a − 2c = 5
3a − c = 5
So a = 1 and c = −2.
b − 2d
Using the second column of AB, , and the same approach as before,
3b − d
we find b = 1 and d = −2.
1 1
Thus, B =
−2 −2
4.1. Matrix Operations 219
.
(c) First, we can find the standard matrix
of R ◦ T by using Theorem 4.1.3. Since
1 −1
the standard matrix of T is A = , and since the standard matrix of R
3 4
−2 5
is B = , it follows that the standard matrix of R ◦ T is
6 −7
−2 5 1 −1 −2 + 15 2 + 20 13 22
BA = = =
6 −7 3 4 6 − 21 −6 − 28 −15 −34
Another way to find this is to first find a formula for the composition R ◦ T .
x1 x1 x1 − x2
(R ◦ T ) =R T =R
x2 x2 3x1 + 4x2
220 Chapter 4. Matrix Algebra
−2(x1 − x2 ) + 5(3x1 + 4x2 )
=
6(x1 − x2 ) − 7(3x1 + 4x2 )
−2x1 + 2x2 + 15x1 + 20x2 13x1 + 22x2
= =
6x1 − 6x2 − 21x1 − 28x2 −15x1 − 34x2
13 22
So the standard matrix of R ◦ T is .
−15 −34
and
A(BC) = A(Bc1 ) A(Bc2 ) · · · A(Bcq )
p
To finish,
we need to show that for every vector x in R , A(Bx) = (AB)x. Let
x1
x2
x = .. be a vector in Rp , and let b1 , b2 , · · · , bp be the columns of B. Then
.
xp
x1
x2
A(Bx) = A b1 b2 · · · bp ..
.
xp
By the definition of the product
= A (x1 b1 + x2 b2 + · · · + xp bp )
of a matrix by a vector
= A(x1 b1 ) + A(x2 b2 ) + · · · + A(xp bp )
= x1 Ab1 + x2 Ab2 + · · · + xp Abp
x1
x2
By the definition of the product
= Ab1 Ab2 · · · Abp ..
. of a matrix by a vector
xp
By definition of matrix multipli-
= (AB)x
cation
So
A(BC) = A(Bc1 ) A(Bc2 ) · · · A(Bcq )
= (AB)c1 (AB)c2 · · · (AB)cq
= (AB)C
Warning
In general, AB 6= BA.
1 0 1 1 0 0
(d) False. Indeed, let A = ,B = , and C = . Then
0 0 0 0 1 1
1 1 0 0 1 1 1 1
BC = = and A(BC) =
0 0 1 1 0 0 0 0
Warning
If AB = AC, it is not true in general that B = C. This means that the
cancellation properties do not hold for matrix multiplication.
0 0 1 0
(f ) False. Indeed, let A = and let B = . Clearly, we have AB =
0 1 0 0
0 0
, but A 6= 0 and B 6= 0. This gives the following.
0 0
Warning
If AB = 0, it is not true in general that either A = 0 or B = 0. This means
that the zero product property does not hold for matrix multiplication.
Commutative Matrices
Let A and B be matrices. We say that A and B commute if AB = BA.
Warning: In general, AB 6= BA as we saw in Exercise 18-(a).
Power of a Matrix.
Let A be a square matrix, and let k be a positive number. Define Ak as
Ak = A
| ·{z
· · A}
k
1 2
For A = , we have
3 4
2 1 2 1 2 1(1) + 2(3) 1(2) + 2(4) 7 10
A = AA = = =
3 4 3 4 3(1) + 4(3) 3(2) + 4(4) 15 22
We now have
−7 −1 2 0 −1 2
B 3 = BBB = B 2 B = 3 −2 6 3 1 0
−2 2 −3 −2 0 1
−7(0) − 1(3) + 2(−2) −7(−1) − 1(1) + 2(0) −7(2) − 1(0) + 2(1)
= 3(0) − 2(3) + 6(−2) 3(−1) − 2(1) + 6(0) 3(2) − 2(0) + 6(1)
−2(0) + 2(3) − 3(−2) −2(−1) + 2(1) − 3(0) −2(2) + 2(0) − 3(1)
0−3−4 7 − 1 + 0 −14 + 0 + 2 −7 6 −12
= 0 − 6 − 12 −3 − 2 + 0 6 − 0 + 6 = −18 −5 12
0 + 6 + 6 +2 + 2 − 0 −4 + 0 − 3 12 4 −7
−1
AB = −2 3 5 2 = −2(−1) + 3(2) + 5(4) = 28
4
−1 −1(−2) −1(3) −1(5)
BA = 2 −2 3 5 = 2(−2) 2(3) 2(5)
4 4(−2) 4(3) 4(5)
2 −3 −5
= −4
6 10
−8 12 20
Transpose of a Matrix
0
For A = 0 6 7 , AT = 6 .
7
−3
For B = 9 , B T = −3 9 −8 .
−8
0 3
0 −1 2
For C = , C T = −1 1 .
3 1 0
2 0
4.1. Matrix Operations 225
−4 2 −2 −4 3 10
For D = 3 1 −7 , DT = 2 1 13 .
10 13 20 −2 −7 20
4 −2
T 4 −1 2 1 0
AA − 5I2 = −1 1 −5
−2 1 3 0 1
2 3
16 + 1 + 4 −8 − 1 + 6 1 0
= −5
−8 − 1 + 6 4+1+9 0 1
21 −3 −5 0 16 −3
= + =
−3 14 0 −5 −3 9
1 0 0 4 −2
T 4 −1 2
8I3 − A A = 8 0 1 0 − −1
1
−2 1 3
0 0 1 2 3
1 0 0 16 + 4 −4 − 2 8−6
= 8 0 1 0 − −4 − 2 1 + 1 −2 + 3
0 0 1 8 − 6 −2 + 3 4+9
8 0 0 20 −6 2 −12 6 −2
= 0 8 0 − −6 2 1 = 6 6 −1
0 0 8 2 1 13 −2 −1 −5
(a) (AT )T = A
(b) (A + B)T = AT + B T
(d) (AB)T = B T AT
1 1 1 0
(c) False. Indeed, let A = and B = .
0 1 1 1
1 1 1 0 2 1 2 1 T
AB = = and (AB) =
0 1 1 1 1 1 1 1
(A + B)2 = (A + B)(A + B) = A2 + AB + BA + B 2
2 2 2
But AB 6= BA in general, so the equality
(A + B) = A + 2AB + B is not true in
1 1 0 0
general. For example, take A = and B = . Then
0 0 1 1
1 1 1 1 1 1 2 2
A+B = and (A + B) = 2
= .
1 1 1 1 1 1 2 2
4.2. The Inverse of a Matrix 227
The first mistake is at the second line because XC−BX is not equal to X(C−B).
Indeed, X(C − B) = XC − XB, but XB 6= BX in general (see Exercise 18a).
The second mistake is at the last line. To understand why it is a mistake, see
Exercise 18f.
AB = I and BA = I
and
1 −1 2 1 0 1 1 0 0
BA = −1 0 1 2 2 3 = 0 1 0 = I,
0 1 −2 1 1 1 0 0 1
Since AB is not the identity matrix, it follows that B is not the inverse of A.
1 2
(a) For the matrix A = , the determinant is
2 6
−6 7
(b) The determinant of A = is
12 −14
−2 3
(c) The determinant of A = is
−4 8
15 −18
(d) The determinant of A = is
−10 12
Since det A = 0, it follows that A is not invertible. So A−1 does not exist.
Ax = b
A−1 Ax = A−1 b Left-multiply both sides by A−1
Ix = A−1 b Since A−1 A = I
x = A−1 b Since Ix = x
−3x1 − 6x2 = 10
(a) The matrix equation for the system is Ax = b with
−x1 + 3x2 = 10
−3 −6 x1 10
A= , x= , and b =
−1 3 x2 10
4.2. The Inverse of a Matrix 231
An Inversion Method
Let A be a square matrix.
So to find
the inverse of A (if any), we start off with the augmented matrix
A I . The goal is to use elementary row operations to transform A into I
(if this is possible). These operations are performed on A and I simultaneously.
If A is invertible, we will end up with the augmented matrix I A−1 .
232 Chapter 4. Matrix Algebra
1 1 1
(a) We find the inverse of A = 0 1 1 (if any). The augmented matrix
2 3 4
A I is
1 1 1 1 0 0
0 1 1 0 1 0
2 3 4 0 0 1
We perform elementary row operations, the idea being to transform A into the
identity matrix (if it is possible).
1 1 1 1 0 0 1 1 1 1 0 0
0 1 1 0 1 0 R3 − 2R1 → R3 0 1 1 0 1 0
2 3 4 0 0 1 0 1 2 −2 0 1
1 1 1 1 0 0
R3 − R2 → R3 0 1 1 0 1 0
0 0 1 −2 −1 1
1 1 0 3 1 −1
R1 − R3 → R1 0 1 0 2 2 −1
R2 − R3 → R2
0 0 1 −2 −1 1
1 0 0 1 −1 0
R1 − R2 → R1 0 1 0 2 2 −1
0 0 1 −2 −1 1
Since we were able to transform A into the identity matrix using elementary row
operations, it follows that A is invertible and
1 −1 0
A−1 = 2 2 −1
−2 −1 1
It is a good idea to check that AA−1 = I:
1 1 1 1 −1 0 1 0 0
AA−1 = 0 1 1 2 2 −1 = 0 1 0 = I
2 3 4 −2 −1 1 0 0 1
It is not necessary to check that A−1 A = I because A is invertible.
1 −3 −2
(b) A = −1 5 2
−3 7 7
We reduce the augmented matrix A I .
1 −3 −2 1 0 0 1 −3 −2 1 0 0
−1 R2 + R1 → R2
5 2 0 1 0 0 2 0 1 1 0
R3 + 3R1 → R3
−3 7 7 0 0 1 0 −2 1 3 0 1
4.2. The Inverse of a Matrix 233
1 −3 −2 1 0 0
R3 + R2 → R3 0 2 0 1 1 0
0 0 1 4 1 1
1 −3 −2 1 0 0
1 1 1
R
2 2
→ R2 0 1 0 2 2
0
0 0 1 4 1 1
1 −3 0 9 2 2
1 1
R1 + 2R3 → R1 0 1 0 0
2 2
0 0 1 4 1 1
21 7
1 0 0 2 2
2
1 1
R1 + 3R2 → R1 0 1 0 0
2 2
0 0 1 4 1 1
So A is invertible and
21 7
2 2
2
A−1 = 1 1
0
2 2
4 1 1
One can easily check that AA−1 = I.
(c) We reduce the augmented matrix A I .
1 −6 5 1 0 0 1 −6 5 1 0 0
−2 R2 + 2R1 → R2
13 −17 0 1 0 0 1 −7 2 1 0
R3 − 3R1 → R3
3 −19 22 0 0 1 0 −1 7 −3 0 1
1 −6 5 1 0 0
R3 + R2 → R3 0 1 −7 2 1 0
0 0 0 −1 1 1
1 −6 5
So an echelon form of A is 0 1 −7 . Since every entry in the third row of
0 0 0
this latter matrix is 0, it is not possible to transform A into the identity matrix.
Therefore A is not invertible, that is, A−1 does not exist.
(d) We reduce the augmented matrix A I .
0 2 1 1 0 0 0 2 1 1 0 0
− 12 R2
→ R2
−2 −2 −2 0 1 0 1 1 1 0 − 12 0
(−1)R3 → R3
−1 0 1 0 0 1 1 0 −1 0 0 −1
1 0 −1 0 0 −1
R1 ↔ R3 1 1 1 0 − 12 0
0 2 1 1 0 0
234 Chapter 4. Matrix Algebra
1 0 −1 0 0 −1
R2 − R1 → R2 0 1 2 0 − 12 1
0 2 1 1 0 0
1 0 −1 0 0 −1
R3 − 2R2 → R3 0 1 2 0 − 12 1
0 0 −3 1 1 −2
1 0 −1 0 0 −1
− 31 R3 → R3 0 1 2 0 − 12 1
1 1 2
0 0 1 −3 −3 3
1 0 0 − 13 − 13 − 13
R1 + R3 → R1 2 1
0 1 0 − 13
R2 − 2R3 → R2 3 6
0 0 1 − 13 − 13 2
3
− 13 − 31 − 13
2 1
Thus A−1 = − 13 .
3 6
1
−3 − 31 2
3
(e) We reduce the augmented matrix A I .
1 0 0 0 1 0 0 0 1 0 0 0 1 0 0 0
R2 − 2R1 → R2
2 1 0 0 0 1 0 0 0 1 0 0 −2 1 0 0
R3 − 3R1 → R3
3 1 1 0 0 0 1 0 0 1 1 0 −3 0 1 0
R4 − 4R1 → R4
4 1 1 1 0 0 0 1 0 1 1 1 −4 0 0 1
1 0 0 0 1 0 0 0
R3 − R2 → R3 0 1
0 0 −2 1 0 0
R4 − R2 → R4 0 0 1 0 −1 −1 1 0
0 0 1 1 −2 −1 0 1
1 0 0 0 1 0 0 0
0 1 0 0 −2 1 0 0
R4 − R3 → R4
0 0 1 0 −1 −1 1 0
0 0 0 1 −1 0 −1 1
1 0 0 0
−2 1 0 0
Thus, A−1 =
−1 −1
.
1 0
−1 0 −1 1
1 0
. First, we have
0 1
a x
2 −1 0 b y = 2a − b 2x − y
AB =
0 1 −1 b−c y−z
c z
So AB = I2 if and only if
2a − b = 1 2x − y = 0
and
b − c =0 y − z =1
We have two systems. The goal here is not to find the set of all solutions, but just
to find one solution. So we don’t need to solve those systems by row reducing
the augmented matrix as usual. The idea is to assign values to some variables
and find other variables.
Finding a solution to the first system. Let b = 1 (you can assign any value
to b). Then the first equation becomes 2a − 1 = 1. This implies that a = 1.
Substituting b with 1 into the second equation, we get 1 − c = 0, or c = 1.
Thus, if b = 1, then a = 1 and c = 1.
Finding a solution to the second system. We proceed in the same way. Let
y = 2. Then 2x − 2 = 0 and 2 − z = 1. These imply that x = 1 and z = 1.
Thus, if y = 2, then x = 1 and z = 1.
1 1
So B = 1 2 . (Of course, this is not the only solution to the problem.
1 1
Actually, there are infinitely many matrices B satisfying the equation AB = I2 .)
(b)
1 1 2 + 0 −1 + 1 0 − 1 2 0 −1
2 −1 0
BA = 1 2 = 2 + 0 −1 + 2 0 − 2 = 2 1 −2
0 1 −1
1 1 2 + 0 −1 + 1 0 − 1 2 0 −1
We observe that BA 6= I3 .
(c) Suppose that there is a 3 × 2 matrix C such that CA = I3 . Then
CAB = (CA)B = I3 B = B
CAB = C(AB) = CI2 = C
(a) We want to show that B −1 A−1 is the inverse of AB. Since A and B are both
invertible, AA−1 = I, A−1 A = I, BB −1 = I, and B −1 B = I.
236 Chapter 4. Matrix Algebra
Properties of Inverses
Theorem 4.2.4. Let A, B be square matrices of the same size.
We proved (c) and (e) in Exercise 9. The proofs of (a), (b), and (d) are similar.
The properties (f) and (g) follow immediately from (c).
4.2. The Inverse of a Matrix 237
1
2 0 −1 2
0
αA = , (αA) = 1
0 2 0 2
AB = 0
A−1 AB = A−1 0 Left-multiply both sides by A−1
IB = 0 A−1 A = I
B=0 IB = B
(a)
AX = C
A−1 AX = A−1 C Left-multiply both sides by A−1
IX = A−1 C A−1 A = I since A is invertible
X = A−1 C IX=X
(b)
XB = C
XBB −1 = CB −1 Right-multiply both sides by B −1
XI = CB −1 BB −1 = I since B is invertible
X = CB −1 XI = X
(c)
AXB −1 = C
A−1 AXB −1 = A−1 C Left-multiply both sides by A−1
IXB −1 = A−1 C A−1 A = I since A is invertible
XB −1 = A−1 C IX = X
XB −1 B = A−1 CB Right-multiply both sides by B
XI = A−1 CB B −1 B = I
X = A−1 CB XI = X
(d)
A−1 X T B = C T
Left-multiply both sides by A and
AA−1 X T BB −1 = AC T B −1
Right-multiply both sides by B −1
IX T I = AC T B −1 AA−1 = I and BB −1 = I
X T = AC T B −1
T
(X T )T = AC T B −1 Take the transpose of both sides
X = (B −1 )T (C T )T AT Use the property (X T )T = X
X = (B −1 )T CAT Use the property (C T )T = C
(e)
A−1 (3X + In )T B −1 = CB
4.2. The Inverse of a Matrix 239
−1
T 0 2 1 1 −1
2X + =
−8 4 6 0 1
" −1 −1
# −1
T 0 2 1 1 −1
2X + = Take the inverse of both sides
−8 4 6 0 1
−1
0 2 1 1 −1
T
2X + = Use the property (A−1 )−1 = A
−8 4 6 0 1
−1 −1
T 0 2 1 1 −1 Use the property
2X + =
−8 4 6 0 1 (kA)−1 = k −1 A−1
−1
0 2 1 −1
2X T + =6
−8 4 0 1
240 Chapter 4. Matrix Algebra
1 −1 1 −1
Since the determinant of is 1 − 0 = 1, it follows that the matrix
0 1 0 1
is invertible and −1
1 −1 1 1 1 1 1
= =
0 1 1 0 1 0 1
T 0 2 1 1
2X + =6
−8 4 0 1
T 0 2 6 6
2X + =
−8 4 0 6
T 6 6 0 2
2X = −
0 6 −8 4
T 6 4
2X =
8 2
T 3 2
X = Divide both sides by 2
4 1
T
T T 3 2
(X ) = Take the transpose of both sides
4 1
3 4
X= Use the property (AT )T = A
2 1
XA = B + XC
XA − XC = B Subtract XC from both sides
X(A − C) = B Left-factor out X
A − C = X −1 B Left-multiply both sides by X −1
XA = B + XC
4.2. The Inverse of a Matrix 241
BAX T = C + 2AX T
BAX T − 2AX T = C Subtract 2AX T from both sides
(BA − 2A)X T = C Right-factor out X T
The next thing we want to do is to left-multiply both sides of the latter equation by
(BA − 2A)−1 . But this requires BA − 2A to be invertible. So, before we continue, we
need to explain why BA − 2A is invertible. Consider the equation (BA − 2A)X T = C
above. Since X is invertible by hypothesis, it follows (by Theorem 4.2.4-(e)) that X T
is invertible. So we can right-multiply both sides of the equation (BA−2A)X T = C by
(X T )−1 . We obtain BA − 2A = C(X T )−1 . Since both C and (X T )−1 are invertible, it
follows that the product C(X T )−1 is invetible. This implies that BA−2A is invertible.
We come back to the equation.
BAX T = C + 2AX T
BAX T − 2AX T = C Subtract 2AX T from both sides
(BA − 2A)X T = C Right-factor out X T
Left-multiply both sides by
X T = (BA − 2A)−1 C
(BA − 2A)−1
T
(X T )T = (BA − 2A)−1 C Take the transpose of both sides
T Use the properties (AT )T =
X = C T (BA − 2A)−1
A and (AB)T = B T AT
(XA)−1 (A + X) = B
Use the property
A−1 X −1 (A + X) = B
(AB)−1 = B −1 A−1
A−1 X −1 A + A−1 X −1 X = B
A−1 X −1 A + A−1 I = B Use the property X −1 X = I
A−1 X −1 A + A−1 = B Use the property A−1 I = A−1
A−1 X −1 A = B − A−1 Subtract A−1 from both sides
A A−1 X −1 A = A(B − A−1 )
Left multiply both sides by A
242 Chapter 4. Matrix Algebra
A A−1 X −1 A A−1 = A(B − A−1 )A−1 Right multiply both sides by A−1
IMT1. A is invertible.
IMT12. AT is invertible.
P ⇐⇒ Q if (P =⇒ Q and Q =⇒ P )
(a) True. This is the equivalence IMT1 ⇐⇒ IMT2 from Theorem 4.2.5.
1 0
(b) False. Indeed, let A = 0 1. Clearly, the system Ax = 0 has only the trivial
0 0
solution. But there is not a pivot position in every row as the third row does
not have a pivot.
1 0
(c) False. The matrix A = 0 1 has a pivot in every column, but A is not
0 0
invertible as it is not even a square matrix. (Remember that only square matrices
have inverses.)
(d) True. Indeed, suppose A is a square n × n matrix and that there is a pivot
position in every column. Then there is a pivot position in every row. There-
fore, the statement IMT3 from Theorem 4.2.5 is true. Using the equivalence
IMT3 ⇐⇒ IMT1 in Theorem 4.2.5, it follows that A is invertible.
(e) False. Indeed, let
1 0
1 0 0
C= and A = 0 1
0 1 0
0 0
1 0
We have CA = = I. But A is not invertible as it is not even a square
0 1
matrix.
(f ) True. The sentence “A and C are square matrices such that CA = I” is precisely
the statement IMT10 from Theorem 4.2.5. Using the equivalence IMT10 ⇐⇒
IMT1 in Theorem 4.2.5, we can conclude that A is invertible.
1 0
(g) False. Indeed, let A = 0 1 and let T : R2 → R3 be the linear transformation
0 0
defined by T (x) = Ax. Since every column of A is a pivot column, T is one-
to-one. However, the matrix A is not invertible since it is not even a square
matrix.
(h) True. The sentence “an n×n matrix A is invertible” is the statement IMT1 from
Theorem 4.2.5. Using the equivalence IMT1 ⇐⇒ IMT8 from Theorem 4.2.5,
it follows that the columns of A span Rn .
244 Chapter 4. Matrix Algebra
(i) True. This is the equivalence IMT6 ⇐⇒ IMT9 from Thereom 4.2.5.
(j) False because the equivalence IMT1 ⇐⇒ IMT12 from Theorem 4.2.5 says that
A is invertible if and only if AT is invertible.
(k) True. Indeed, suppose A and B are square matrices such that AB = I. Then,
by the equivalence IMT11 ⇐⇒ IMT1 from Theorem 4.2.5, A is invertible. So
there exists a matrix X such that AX = I and XA = I. Since AB = I, it
follows that AB = AX. Left-multiplying both sides of this latter equation by
A−1 , we get B = X. So AB = I and BA = I, which implies that AB = BA.
This proves that A and B commute.
(l) False because the equivalence IMT1 ⇐⇒ IMT8 from Theorem 4.2.5 says that
an n × n matrix A is invertible if and only if its columns span Rn .
(m) True. Indeed, the sentence “A is a square matrix whose columns are not linearly
independent” means that the statement IMT5 from Theorem 4.2.5 is false. Since
IMT5 is equivalent to IMT1 and since IMT5 is false, it follows that IMT1 is also
false. So A is not invertible.
(n) True. This follows from the equivalence IMT6 ⇐⇒ IMT9 from Theorem 4.2.5.
Subspaces of Rn
(2) For every vectors u, v in H, the sum u + v is also in H. (We say that H is
closed under addition.)
(3) For every u in H, for every scalar c, the vector cu is in H. (We say that H is
closed under scalar multiplication.)
Example of subspaces of Rn include {0} and Rn itself. The subspace {0} is called
the zero subspace of Rn .
245
246 Chapter 5. Subspaces of Rn
1 2
it is not closed under addition. Indeed, let u = and v = be vectors
1 4
3
in H. Then the sum u + v = is not in H since 5 6= 32 .
5
(d) True by definition.
(e) False because the line of equation x2 = 2x1 + 3 is not a suspace of R2 as it does
not pass through the origin (since 0 6= 2(0) + 3).
(f ) False by definition.
(g) False by definition.
(h) False because any line in R2 through the origin is a subspace of R2 . Indeed,
the equation of a line in R2 is of the form x2 = mx1 , where m is a real number.
Let m be a real number and let H be the set of points (x1 , x2 ) in R2 such that
x2 = mx1 . We want to prove that H is a subspace of R2 , which amounts to
proving that H satisfies conditions (1), (2), and (3) of the definition of subspace.
Condition (1) is satisfied because 0 = m(0).
u1 v1
Checking condition (2). Let u = and v = be vectors in H.
u2 v2
Then
u2 = mu1 and v2 = mv1
u1 + v1
These imply that u2 + v2 = m(u1 + v1 ). So the sum u + v = is
u2 + v2
in H.
u1
Checking condition (3). Let u = be a vector in H and let c be a
u2
scalar. Then u2 = mu1 . Multiplying both sides of thislatter equation by c,
cu1
we get u2 = m(cu1 ). This shows that the vector cu = is in H.
cu2
Thus, any line in R2 through the origin is a subspace of R2 .
Subspaces of R2 and R3
{0},
R2
{0},
R3
5.1. Subspaces, Column and Null Spaces 247
1 0
(i) False because the vector u = is on the x1 -axis and the vector v =
0
1
1
is on the x2 -axis, but the sum u + v = is neither on the x1 -axis nor on the
1
x2 -axis.
a − 2b
Let u = a be a vector in H, and let c be a scalar. Then
2a + 3b
ca − 2cb α − 2β
cu = ca = α ,
2ca + 3cb 2α + 3β
u + v = (c1 v1 + · · · cp vp ) + (d1 v1 + · · · dp vp )
= (c1 + d1 )v1 + · · · + (cp + dp )vp = α1 v1 + · · · + αp vp ,
du = dc1 v1 + · · · dcp vp = β1 v1 + · · · + βp vp ,
(a) The subset H is not a subspace of R2 since it does not contain the zero vector.
(b) Here H is a line through the origin. So, by Theorem 5.1.1, it is a subspace of
R2 .
Alternate Solution
Let u be a nonzero vector in H. Then, since H is a line through the origin,
every vector in H is a multiple of u. This means that H = Span{u}. So,
by Theorem 5.1.3, H is a subspace of R2 .
(c) This is not a subspace of R2 because it is not closed under scalar multiplication.
Indeed, if we multiply a nonzero vector in H by a negative number, we get vector
that does not lie in H.
(d) For the same reason as in part (c), H is not a subspace of R2 .
(e) From the graph, we see that the subset H does not contain the origin. So it is
not a subspace of R2 .
3
H 2
2
S
1
−2 −1 1 2
1 2 3
The subset H is not a subspace of R2 because it does not contain the origin.
The subset S is not a subspace either
since
it is not closed under
scalar
mul-
0.5 2
tiplication. Indeed, the vector u = is in S, but 4u = is not in
0 0
S.
Column Spaces
Let a1 , a2 , · · · , an be vectors in Rm . Let A = a1 a2 · · · an be the matrix
whose columns are a1 , a2 , · · · , an . The column space of A, denoted Col(A),
is the set of all linear combinations of the columns of A, that is,
Col(A) = Span{a1 , a2 , · · · , an }
Suppose
that
the system Ax = b is consistent. Then there exists a vector
x1
x2
x = .. in Rn such that Ax = b. By definition of the multiplication
.
xn
of a matrix by a vector, we have
x1
x2
Ax = a1 a2 · · · an = x 1 a1 + x 2 a2 + · · · + x n an
..
.
xn
x 1 a1 + x 2 a2 + · · · + x n an = b
1 0
(h) False. Indeed, let A = 0 1 be an m × n matrix with m = 3 and n = 2.
0 0
The columns of A are linearly independent (since every column has a pivot), but
they do not span R3 since two vectors cannot span R3 .
5.1. Subspaces, Column and Null Spaces 253
u = c1 a1 + c2 a2 + c3 a3
−2b1 − 3b2 + b3 = 0
(a) First, recall the definition of the range. The range of T : Rn → Rm is the set
of all vectors b in Rm for which the equation Ax = b is consistent. Let b be a
vector in Rm . The following statements are equivalent.
(i) The vector b is in Col(A).
(ii) The linear system Ax = b is consistent.
(iii) The vector b is in the range of T .
The statement (i) is equivalent to (ii) by Theorem 5.1.5, while (ii) is equivalent
to (iii) by the definition of the range.
Since (i) is equivalent to (ii) and (ii) is equivalent to (iii), it follows that (i) is
equivalent to (iii). This shows that a vector b is in the column space of A if and
only if b is in the range of T . So Col(A) equals the range of T .
256 Chapter 5. Subspaces of Rn
Alternate Solution
The following alternate solution does not use Theorem 5.1.5. Let
a1 , a2 , · · · , an be the columns of A.
c 1 a1 + c 2 a2 + · · · + c n an = b
So b is in Col(A).
Alternate Solution
The following alternate solution does not use Theorem 5.1.5. Let
a1 , a2 , · · · , an be the columns of A.
1 2 4 b1
R3 + R2 → R3 0 1 −5 b2 − 3b1
0 0 0 b1 + b2 + b3
Thus, a vector
b is in Col(A)if and
only if b is a linear combination of the
−1 −1
vectors u = 1 and v = 0 . This means that Col(A) is the plane in
0 1
3
R spanned by the vectors u and v.
Proof. We need to show that Null(A) satisfies the three conditions of the defi-
nition of a subspace.
So Null(A) is a subspace of Rn
5.1. Subspaces, Column and Null Spaces 259
x1 − x2 = 0
= 0
x1 = s
x2 = s
Thus, the
solution
set to the equation Ax = 0 is the line in R2 spanned by the
1
vector .
1
(g) True. By definition, the columns of A are linearly independent if and only if the
system Ax = 0 has only the trivial solution, that is, if and only if Null(A) = {0}.
(h) True. Indeed, by definition Null(A) = {0} if and only if the equation Ax = 0
has only the trivial solution. By the Invertible Matrix Theorem, the system
Ax = 0 has only the trivial solution if and only if A is invertible.
x4
x1 3s − 2t 3 −2
x2 s = s 1 + t 0
x=
x3
=
−4t 0 −4
x4 t 0 1
where s and t are real numbers.
(c) To find a vector in Null(A), all we have to do is to assign numbers to s and t.
Let s = 1 and t = 0. Substituting s with 1 and t with 0 into the solution
3
1
obtained in part (b), we get the vector
0 in Null(A).
0
−2
0
When s = 0 and t = 1, we obtain the vector −4 in Null(A).
1
(d) Let ai , 1 ≤ i ≤ 4, be the ith column of A. By definition, a vector in Col(A) is a
linear combination of the columns of A. For example,
5.2. Basis, Dimension, and Rank 261
1
−3
1a1 + 0a2 + 0a3 + 0a4 = a1 =
−1 is in Col(A).
2
−3
9
The vector 0a1 + 1a2 + 0a3 + 0a4 = a2 =
3 is in Col(A) as well.
−6
(2) B spans H. (This means that every vector in H can be written as a linear
combination of v1 , · · · , vp .)
In other words, all the entries of ei are 0 except the ith entry, which is 1.
For example,
if n = 1, e1 = 1 .
1 0
If n = 2, we have e1 = and e2 = .
0 1
262 Chapter 5. Subspaces of Rn
1 0 0
If n = 3, we have e1 = 0 , e2 = 1 , and e3 = 0 .
0 0 1
(f ) False by definition.
Using the same elementary row operations as above, the augmented matrix of
the latter system can be transformed into
" 3
#
1 0 b
5 1
+ 15 b2
1
0 1 b
5 2
− 52 b1
We now explain how to use Theorem 5.2.3 to show that the set B = {v1 , v2 }
above is a basis for R2 . Since dim R2 = 2 and since B has two vectors, we
just need to show that B is linearly independent (thanks to Theorem 5.2.3)
to conclude that it is a basis for R2 . But we already proved this above.
(b) Let’s first recall the notion of coordinates of a vector relative to a basis.
c1
..
the basis B. The notation for this is [x]B = . .
cp
−9
To find the coordinates of b = relative to the basis B = {v1 , v2 }, we
7
need to write b asa linear
combination of v1 and v2 . In part (a), we showed that
b1
every vector b = can be written as
b2
3 1 2 1
b= b1 + b2 v1 + − b1 + b2 v2 .
5 5 5 5
Substituting b1 with −9 and b2 with 7, we get
3 1 2 1
b= (−9) + (7) v1 + − (−9) + (7) v2
5 5 5 5
27 7 18 7
= − + v1 + + v2 = −4v1 + 5v2
5 5 5 5
Hence, the coordinates
of b relative to the basis {v1 , v2 } are −4 and 5, that is,
−4
[x]B = .
5
Alternate Solution
Since dim R2 = 2, any basis for R2 must have 2 vectors. Since the set
B = {v} has only one vector, it cannot be a basis for R2 .
(b) By inspection, one can see that v1 = −3v2 . This implies that the set B = {v1 , v2 }
is not linearly independent, and therefore is is not a basis for R2 .
(c) The set B is not a basis for R3 because any basis for R3 must have 3 vectors
since dim R3 = 3. And B does not contain 3 vectors.
(d) First, we need to determine if B is linearly independent. Let A = v1 v2 v3
be the matrix whose columns are v1 , v2 , and v3 . We need to reduce A.
−4 −1 −5 −1 1 −5
3 2 0 R1 + R2 → R1 3 2 0
7 6 −4 7 6 −4
1 −1 5
(−1)R1 → R1 3 2 0
7 6 −4
1 −1 5
R2 − 3R1 → R2 0 5 −15
R3 − 7R1 → R3
0 13 −39
1
R → R 1 −1 5
5 2 2
1
0 1 −3
13
R 3 → R 3 0 1 −3
1 −1 5
R3 − R2 → R3 0 1 −3 =B
0 0 0
(a) Since H = Span{v1 , v2 }, the set B = {v1 , v2 } spans H. To show that B is linearly
independent, let c1 , c2 be scalars such that c1 v1 + c2 v2 = 0. Then we have the
5.2. Basis, Dimension, and Rank 267
following system.
2c1 + c2 = 0
− c2 = 0
c1 + 3c2 = 0
Solving this, we get c1 = 0 and c2 = 0. This implies that B is linearly indepen-
dent. Hence, B is a basis for H.
(b) We want to determine if there exist scalars c1 and c2 such tha c1 v1 + c2 v2 = x.
This amounts to determining if the following system is consistent.
2c1 + c2 = 1
− c2 = 3
c1 + 3c2 = −7
We reduce the augmented matrix:
2 1 1 1 3 −7
0 −1 3 R1 ↔ R3 0 −1 3
1 3 −7 2 1 1
1 3 −7
R3 − 2R1 → R3 0 −1 3
0 −5 15
1 3 −7
(−1)R2 → R2 0 1 −3
− 15 R3 → R3
0 1 −3
1 3 −7
R3 − R2 → R3 0 1 −3
0 0 0
1 0 2
R1 − 3R2 → R1 0 1 −3
0 0 0
Since x= 2v1− 3v2 , the coordinates of x relative to B are 2 and −3, that is,
2
[x]B = .
−3
(a) To find a basis for Null(A), we first need to find Null(A). By definition, Null(A)
is the set of solutions to the equation Ax = 0. So, to find Null(A), we first need
to solve the system Ax = 0. Row reducing the augmented matrix, we have
1 2 4 0 1 2 4 0
R2 − 3R1 → R2
3 7 7 0 0 1 −5 0
R3 + 4R1 → R3
−4 −9 −11 0 0 −1 5 0
268 Chapter 5. Subspaces of Rn
1 2 4 0
R3 + R2 → R3 0 1 −5 0
0 0 0 0
1 0 14 0
R1 − 2R2 → R1 0 1 −5 0
0 0 0 0
x1 + 14x3 = 0
x2 − 5x3 = 0
Null(A) = Span{v}
Now, we find a basis for Col(A). We will need the following theorem.
1 2 4
From the row reduction above, an echelon form of A = 3 7 7 is
−4 −9 −11
1 2 4
B= 0 1 −5
0 0 0
The pivot columns of B are column 1 and column 2. The corresponding columns
in A are the vectors
1 2
u1 = 3 and u2 = 7
−4 −9
Null(A) = Span{v1 , v2 }
So the set {v1 , v2 } spans Null(A). Moreover, the vectors v1 and v2 are linearly in-
dependent since v2 is not a multiple of v1 (otherwise, the third entry of v2 would
270 Chapter 5. Subspaces of Rn
be zero, which is not the case). Thus, the set {v1 , v2 } is a basis for Null(A).
Now, we find a basis for Col(A). From the row reduction above, an echelon form
of A is
1 −6 8
B= 0 0 0
0 0 0
The matrix B has only one pivot
column:
the first column. The corresponding
5
column in A is the vector u = 3 . By Theorem 5.2.5, the set {u} is a basis
−4
for Col(A).
So the set {u, v} is a basis for H. Since this basis has two vectors, it follows
that dim H = 2.
(j) True by the Basis Theorem.
(k) True by the Basis Theorem.
(l) True. This is true because of the following theorem.
The first
two columns of B are pivot columns. The corresponding columns in A are
1 −3
u1 = −1 and u2 =
5 . Thanks to Theorem 5.2.5, the set {u1 , u2 } is a basis
−4 10
for Span{v1 , v2 , v3 }.
272 Chapter 5. Subspaces of Rn
it follows that
The fact that rank(A) ≤ n follows immediately from the Rank-Nullity The-
orem. The inequality rank(A) ≤ m follows from the fact that the rank of A
corresponds to the number of pivot positions of A and the fact that the number
of pivot positions of A cannot be greater than the number of rows of A.
0 0 0
that rank(A) = 3.
(j) False. Indeed, if the system Ax = 0 has two free variables, then dim Null(A) = 2
(by Theorem 5.2.6). Therefore, by the Rank-Nullity Theorem, rank(A) = 6−2 =
4.
(a) (i) By definition, Null(A) is the set of solutions to the homogeneous equation
Ax = 0. So to find a basis for Null(A), we first need to solve the system Ax = 0:
1 −1 1 0 1 −1 1 0
0 1 1 0 R3 + R1 → R3 0 1 1 0
−1 2 1 0 0 1 2 0
1 −1 1 0
R3 − R2 → R3 0 1 1 0
0 0 1 0
1 −1 0 0
R1 − R3 → R1 0 1 0 0
R2 − R3 → R2
0 0 1 0
1 0 0 0
R1 + R2 → R1 0 1 0 0
0 0 1 0
(iii) To find a basis for the column space of A, we will Theorem 5.2.5. From the
5.2. Basis, Dimension, and Rank 275
1 −1 1
row reduction above, an echelon form of A = 0 1 1 is
−1 2 1
1 −1 1
B= 0 1 1
0 0 1
The pivot columns of B are column 1, column 2, and Column 3. The corre-
sponding columns in A are the vectors
1 −1 1
u1 = 0 , u2 =
1 , and u3 = 1
−1 2 1
(iv) Since a basis for Col(A) has three vectors, dim Col(A) = 3.
(v) By definition, rank(A) = dim Col(A). Since dim Col(A) = 3, it follows that
rank(A) = 3.
(b) (i) To find a basis for Null(A), we first need to solve the equation Ax = 0. We
reduce the augmented matrix:
1 2 −1 0 1 2 −1 0
−1 −1 R2 + R1 → R2
4 0 0 1 3 0
R3 + 3R1 → R3
−3 −6 3 0 0 0 0 0
1 0 −7 0
R1 − 2R2 → R1 0 1 3 0
0 0 0 0
x1 − 7x3 = 0
x2 + 3x3 = 0
x1 = 7s
x2 = −3s s is an arbitrary number.
x3 = s
7
where v = −3 . Since every vector of Null(A) is a multiple of v, the set
1
B = {v} spans Null(A). Moreover, B is linearly independent because v 6= 0.
(Remember that a set containing one vector v is linearly independent if and
only if v 6= 0.) Thus, B is a basis for Null(A).
(iv) Since the latter basis has two vectors, dim Col(A) = 2.
(v) By definition, rank(A) = dim Col(A). Since dim Col(A) = 2 (from part (iv)),
it follows that rank(A) = 2.
1 3 0 2
0 0 2 −8
(c) A = −1 −3
2 −10
2 6 −2 12
(i) To find a basis for Null(A), we first need to solve the equation Ax = 0.
1 3 0 2 0 1 3 0 2 0
0
0 2 −8 0 R3 + R1 → R3 0
0 2 −8 0
−1 −3 2 −10 0 R4 − 2R1 → R4 0 0 2 −8 0
2 6 −2 12 0 0 0 −2 8 0
1 3 0 2 0
R3 − R2 → R3
0 0 2 −8 0
R4 + R2 → R4 0 0 0 0 0
0 0 0 0 0
1 3 0 2 0
1
0 0 1 −4 0
R
2 2
→ R2
0 0 0 0 0
0 0 0 0 0
5.2. Basis, Dimension, and Rank 277
x4
−3s − 2t −3 −2
s = s 1 + t 0 = sv1 + tv2 ,
x= 4t 0 4
t 0 1
−3 −2
1
and v2 = 0 . This shows that Null(A) is spanned
where v1 = 0 4
0 1
by the set B = {v1 , v2 }. Moreover, B is linearly independent since v2 is not a
multiple of v1 (otherwise the third entry of v2 would be 0, which is not the case).
Hence, B is a basis for Null(A).
0 1
scalars such that x1 v1 + x2 v2 = 0. Then
−3x1 − 2x2 0
x1 = 0
4x2 0
x2 0
This implies that
−3x1 − 2x2 = 0, x1 = 0, 4x2 = 0, and x2 = 0
278 Chapter 5. Subspaces of Rn
2 −2
By Theorem 5.2.5, the set {u1 , u2 } forms a basis for Col(A).
(iv) Since this latter basis contains two vectors, dim Col(A) = 2.
(v) By definition, rank(A) = dim Col(A). Since dim Col(A) = 2 (from part (iv)),
it follows that rank(A) = 2.
(d) (i) To find a basis for Null(A), we first need to solve the system Ax = 0. Row
reducing the augmented matrix, we have
1 −5 2 0 −1 1 −5 2 0 −1
3 −15 R2 − 3R1 → R2
6 1 −6 0 0 0 1 −3
R3 + R1 → R3
−1 5 −2 −1 4 0 0 0 −1 3
1 −5 2 0 −1
R3 + R2 → R3 0 0 0 1 −3
0 0 0 0 0
The corresponding system is
x1 − 5x2 + 2x3 − x5 = 0
x4 − 3x5 = 0
Set x2 = s1 , x3 = s2 , and x5 = s3 . Then
x1 = 5s1 − 2s2 + s3 and x4 = 3s3
So every vector x in Null(A) is of the form
x1 5s1 − 2s2 + s3 5s1 −2s2 s3
x2 s 1 s1 0 0
x = x3 =
s2 = 0
+
s2 +
0
x4 3s3 0 0 3s3
x5 s3 0 0 1
5.2. Basis, Dimension, and Rank 279
5 −2 1
1
0
0
= s1
0 + s2
1 + s3
0 = s1 v1 + s2 v2 + s3 v3
0 0 3
0 0 1
where
5 −2 1
1
0
0
v1 =
0 ,
v2 =
1 ,
and v3 =
0
0 0 3
0 0 1
This shows that a vector x is in Null(A) if and only if x is a linear combination of
the vectors v1 , v2 , and v3 . In other words, the set B = {v1 , v2 , v3 } spans Null(A).
To determine whether B is linearly independent, let c1 , c2 , c3 be scalars such that
c1 v1 + c2 v2 + c3 v3 = 0. Then
5c1 − 2c2 + c3 0
c1 0
c2 = 0
3c3 0
c3 0
This implies that
Since the equation c1 v1 + c2 v2 + c3 v3 = 0 has only the trivial solution, the set
{v1 , v2 , v3 } is linearly independent.
Conclusion. Since the set B = {v1 , v2 , v3 } is linearly independent and spans
Null(A), it follows that B is a basis for Null(A).
(iii) Finding a basis for Col(A). From the row reduction above, an echelon form
of A is
1 −5 2 0 −1
B= 0 0 0 1 −3
0 0 0 0 0
The pivot columns of B are columns 1 and 4. The corresponding columns in A
are the vectors
1 0
u1 = 3 and u2 = 1
−1 −1
By Theorem 5.2.5, the set {u1 , u2 } is a basis for Col(A).
(iv) From part (iii), a basis for Col(A) has two vectors. So dim Col(A) = 2.
280 Chapter 5. Subspaces of Rn
(v) By definition, rank(A) = dim Col(A). Since dim Col(A) = 2 from part (iv),
it follows that rank(A) = 2.
(a) To find the rank of A, we first need to put A into echelon form.
3 1 −1 2 1 3 1 −1 2 1
−3 −1 1 −4 6 R2 + R1 → R2 0 0 0 −2 7
=B
0 0 0 0 0 R4 + 2R1 → R4 0 0 0 0 0
−6 −2 2 −4 −2 0 0 0 0 0
Since B has two pivot columns (columns 1 and 4), it follows that the dimension
of the column space is 2. So
rank(A) = dim Col(A) = 2
(b) Using the Rank-Nullity Theorem, we have rank(A) + dim Null(A) = 5. This
implies that dim Null(A) = 5 − 2 = 3.
Alternate Solution
Let a5 be the fifth column of A. Using the same reasoning as above, the
columns a1 , a3 , a4 , a5 , and a6 are linearly dependent.
(d) By definition, a basis for a subspace H is a set that is linearly independent and
spans H. From part (a), the set B = {a1 , a3 , a4 , a6 } is a basis for Col(A). So B is
linearly independent and spans Col(A). This implies that the columns a1 , a3 , a4 ,
and a6 are linearly independent.
(e) i. Col(A) is a 4-dimensional subspace of R4 . Indeed, from part (a), dim Col(A) =
4. Since A has 4 rows, every column of A is a vector in R4 . So Col(A) is a
subspace of R4 .
ii. Null(A) is a 2-dimensional subspace of R6 . Indeed, by the Rank-Nullity
Theorem,
(a) Since an echelon form of a 9 × 14 matrix A has exactly five rows of all 0’s, there
are 9 − 5 = 4 pivot columns. This implies that dim Col(A) = 4.
(b) Using the Rank-Nullity Theorem, dim Null(A) = 14 − 4 = 10.
The column space of A is not equal to R4 because every vector in Col(A) has 5
entries (the number of entries corresponds to the number of rows of A), while
every vector in R4 has 4.
Since A has four pivot columns, dim Col(A) = 4. By the Rank-Nullity Theorem,
we deduce that dim Null(A) = 10 − 4 = 6.
We stated the Invertible Matrix Theorem (IMT) in Theorem 4.2.5. The theo-
rem says that 12 statements are equivalent. The following theorem provides 6
more statements to the IMT.
IMT1. A is invertible.
IMT14. Col(A) = Rn
IMT16. rank(A) = n
(a) True. This follows from the implication IMT1 =⇒ IMT17 in Theorem 5.2.8.
1 0
(b) False. Indeed, let A = 0 1. Consider the system Ax = 0:
0 0
x1 = 0
x2 = 0
0 = 0
(a) First, observe that A is a square matrix. So, since the columns of A are linearly
independent by hypothesis, it follows that A is invertible (by the Invertible
Matrix Theorem). This implies that A3 is invertible (using one of the properties
5.2. Basis, Dimension, and Rank 283
Determinants
Determinant of a 1 × 1 Matrix
Let A = a11 be a 1 × 1 matrix. The determinant of A, denoted det A, is
defined by
det A = a11
(a) det 1 = 1
(b) det 23 = 23
(c) det −7 = −7
(d) det 0 = 0
Determinant of a 2 × 2 Matrix
a11 a12
Let A = be a 2 × 2 matrix. The determinant of A, denoted det A,
a21 a22
is defined by
det A = a11 a22 − a12 a21 (6.1.1)
3 4
(a) det = (3)(7) − (4)(5) = 21 − 20 = 1.
5 7
−4 3
(b) det = (−4)(2) − (3)(5) = −8 − 15 = −23.
5 2
−2 −5
(c) det = (−2)(−8) − (−5)(9) = 16 + 45 = 61.
9 −8
−10 −8
(d) det = (−10)(−12) − (−8)(−15) = 120 − 120 = 0.
−15 −12
285
286 Chapter 6. Determinants
Note.
Each Aij in the definition above is an (n − 1) × (n − 1) matrix. So the
determinant of an n × n matrix is defined by determinants of (n − 1) ×
(n − 1) matrices. Such a definition is said to be recursive.
Equation (6.1.2) can be rewritten as
det A = a11 det A11 − a12 det A12 + · · · + (−1)1+n det A1n
where the signs alternate.
Note that
the equation
(6.1.2) coincides with (6.1.1) when n = 2. Indeed,
a a
let A = 11 12 . Then, by (6.1.2), det A = a11 C11 + a12 C12 . But
a21 a22
C11 = (−1)1+1 det A11 = (−1)2 det a22 = (1)a22 = a22
So
det A = a11 C11 + a12 C12 = a11 (a22 ) + a12 (−a21 ) = a11 a22 − a12 a21
6.1. Introduction to Determinants 287
3 4 2
(a) We want to compute the determinant of A = −5 −3 1 using the defi-
2 −1 −2
nition. Here n = 3, a11 = 3, a12 = 4, and a13 = 2. So, by definition,
det A = a11 C11 + a12 C12 + a13 C13 = 3C11 + 4C12 + 2C13
−5 1
C12 = (−1)1+2 det A12 = (−1)3 = (−1) (10 − 2) = −8.
2 −2
−5 −3
C13 = (−1)1+3 det A13 = (−1)4 = (1) (5 − (−6)) = 5 + 6 = 11.
2 −1
So the determinant of A is
det A = a11 C11 + a12 C12 + a13 C13 = 3C11 + 4C12 + 2C13
= 3(7) + 4(−8) + 2(11) = 21 − 32 + 22 = 11
−2 −5 3
(b) First we need to find the cofactors C11 , C12 , and C13 of A = 2 1 −4 .
6 −6 −7
1 −4
C11 = (−1)1+1 det A11 = (−1)2 = (1) (−7 − 24) = −31.
−6 −7
2 −4
C12 = (−1)1+2 det A12 = (−1)3 = (−1) (−14 + 24) = −10.
6 −7
2 1
C13 = (−1)1+3 det A13 = (−1)4 = (1) (−12 − 6) = −18.
6 −6
Using (6.1.2), we have
det A = a11 C11 + a12 C12 + a13 C13 = −2C11 + (−5)C12 + 3C13
= −2(−31) − 5(−10) + 3(−18) = 62 + 50 − 54 = 58
Cofactor Expansions
Let A be an n × n matrix.
The sum (6.1.2) is called the cofactor expansion across the first row
288 Chapter 6. Determinants
of A.
The following theorem says that the determinant does not depend on the row
or column we choose for the cofactor expansion.
1 2 3
(a) We want to compute the determinant of A = 2 4 1 using cofactor expan-
3 4 2
sion.
Step 1. Select one row or one column, preferably the one that contains
the most zeros. Let’s select row 1. Then
1 2 3
A= 2 4 1
3 4 2
Step 2. Add signs to the row or column selected. The rule is that
the sign we add to the upper left corner is always +. And the signs
alternate across each row and down each column. Another way to
add the sign is to use the formula (−1)i+j : the sign we add to the
1 2 3
+ − +
entry aij is (−1)i+j . Adding the signs to row 1, we get 2 4 1
3 4 2
Step 3. Expand det A across the row or down the column selected.
Here we selected row 1, so we are going to expand det A across that
row.
6.1. Introduction to Determinants 289
Deleting
the first row and the first column, we get the matrix
4 1
. Taking the determinant of this latter matrix, we get
4 2
4 1
. Multiplying this by the entry at the intersection be-
4 2
4 1
tween the row and column we deleted, we get (1) . Mul-
4 2
4 1
tiplying all this by the sign we added, we get (+)(1) ,
4 2
which is the term obtained by deleting row 1 and column 1.
Similarly, the term obtained by deleting row 1 and column 2 is
2 1
(−)(2) .
3 2
The term obtained by deleting row 1 and column 3 is
2 4
(+)(3) .
3 4
The determinant of A is the sum of those three terms. That is,
4 1 2 1 2 4
det A = (+)(1) + (−)(2) + (+)(3)
4 2 3 2 3 4
= 1(8 − 4) − 2(4 − 3) + 3(8 − 12)
= 1(4) − 2(1) + 3(−4) = 4 − 2 − 12 = −10
Alternate Solution
We compute det A by expanding down a column. Let’s select column 2.
1 2 3
−
Adding the signs to column 2, we get 2 4 1
+
3 4 2
−
(b) To take advantage of the presence of zero, we can expand det A across row 3
or downcolumn 1. Let’sselect column 1. Adding the signs to that column,
2 −3 1
+
−2 4 1
we get
−
0 −5 −1
+
Expanding det A down column 1, we have
4 1 −3 1 −3 1
det A = (+)(2) + (−)(−2) + (+)(0)
−5 −1 −5 −1 4 1
= 2[−4 − (−5)] + 2[3 − (−5)] + 0(−3 − 4)
= 2(−4 + 5) + 2(3 + 5) + 0 = 2 + 16 = 18
15 12 −6
(c) We want to find det A, where A = 7 0 0
20 −1 1
To takeadvantage of the
zeros, we select row 2. Adding the signs to row 2,
15 12 −6
we get 7 0 0 .
− + −
20 −1 1
Expanding det A across row 2, we get
12 −6 15 −6 15 12
det A = (−)(7) + (+)0 + (−)(0)
−1 1 20 1 20 −1
= −7(12 − 6) + 0 + 0 = −42
12 −3 0
(d) We want to find det A, where A = 77 0 −5 . Expending det A down
−8 −1 0
column 3, we have
77 0 12 −3 12 −3
det A = (+)(0) − (−5) + (+)(0)
−8 −1 −8 −1 77 0
= 0 + 5(−12 − 24) + 0 = −180
(a) To take advantage of the zeros, we select column 3. Adding the signs to
that column, we get
0 4 0 −5
+
1 −6 −3 5
−
0 7 0 −2
+
4 −6 0 2
−
6.1. Introduction to Determinants 291
0 4 −5
det A = 0 + (−)(−3) 0 7 −2 + 0 − 0
4 −6 2
0 4 −5
0 4 −5 +
0 7 −2 Select column 1 and
=3 0 7 −2 =3 −
add signs
4 −6 2 4 −6 2
+
4 −5
= 3 0 − 0 + (+)(4) Expand down column 1
7 −2
= 3 (4(−8 + 35)) = 3(4)(27) = 324
(b)
2 8 −1 6
−3 21 4 −4 Select row 3 and add
det A = 0 −2 0 0
+ − + − signs
5 19 −7 3
2 −1 6
= (−)(−2) −3 4 −4 Expand across row 3
5 −7 3
2 −1 6 2 −1 6
+ − + Select row 1 and add
= 2 −3 4 −4 = 2 −3 4 −4 signs
5 −7 3 5 −7 3
4 −4
= 2 (+)(2) +
−7 3
−3 −4 −3 4
(−)(−1) + (+)(6) Expand across row 1
5 3 5 −7
= 2 [2(12 − 28) + 1(−9 + 20) + 6(21 − 20)]
= 2 (−32 + 11 + 6) = 2(−15) = −30
(c)
−2 −1 1 2 0
+
1 3 2 1 −4
−
0 0 2 0 0 Select column 5 and
det A = + add signs
−1 1 5 6 0
−
2 1 6 3 0
+
292 Chapter 6. Determinants
−2 −1 1 2
0 0 2 0
= (−)(−4) Expand down column 5
−1 1 5 6
2 1 6 3
−2 −1 1 2 −2 −1 1 2
0 0 2 0 0 0 2 0 Select row 2 and add
=4 =4 − + − +
−1 1 5 6 −1 1 5 6 signs
2 1 6 3 2 1 6 3
−2 −1 2
= 4 0 + 0 + (−)(2) −1
1 6 + 0 Expand across row 2
2 1 3
−2 −1 2 −2 −1 2
+ − + Select row 1 and add
= −8 −1 1 6 = −8 −1 1 6 signs
2 1 3 2 1 3
1 6
= −8 (+)(−2) +
1 3
−1 6 −1 1
(−)(−1) + (+)(2) Expand across row 1
2 3 2 1
= −8 [−2(3 − 6) + 1(−3 − 12) + 2(−1 − 2)]
= −8(6 − 15 − 6) = 120
(d)
3 0 7 −2 −4
−2 0 8 0 9
1 −6 −9 2 −8 Select row 5 and add
det A =
−5 0 6 −1 −3 signs
0 0 5 0 0
+ − + − +
3 0 −2 −4
−2 0 0 9
= (+)(5) Expand across row 5
1 −6 2 −8
−5 0 −1 −3
3 0 −2 −4
−
−2 0 0 9
+ Select column 2 and
=5
1 −6 2 −8 add signs
−
−5 0 −1 −3
+
3 −2 −4
= 5 (−)(−6) −2 0 9 Expand down column 2
−5 −1 −3
6.1. Introduction to Determinants 293
3 −2 −4
Select row 2 and add
= 30 −2 0
+
9
−
− signs
−5 −1 −3
−2 −4
= 30 (−)(−2) + 0+
−1 −3
3 −2
(−)(9) Expand across row 2
−5 −1
= 30 [2(6 − 4) − 9(−3 − 10)]
= 30(4 + 117) = 3630
−5 13 −25
(a) Since A = 0 −4 37 is a triangular matrix, it follows (by Theorem 6.1.2)
0 0 3
that
det A = (−5)(−4)(3) = 60
3 0 0 0
89 −7 0 0
(b) Since A = −58
is a triangular matrix, it follows (by Theo-
7 −1 0
61 −6 12 −2
rem 6.1.2) that
det A = (3)(−7)(−1)(−2) = −42
3a 3b
det(3A) = = (3a)(3d) − (3b)(3c)
3c 3d
= 9ad − 9bc = 9(ad − bc) = 9 det A
4 9 −6
(h) False. By definition, the matrix 0 2 −2 is upper triangular (not lower
0 0 −3
triangular!).
(i) True by Theorem 6.1.2.
Note. The only operation that does not affect the determinant is Ri + αRj →
Ri .
(a) We have
A R1 ↔ R3 A1 det A1 = − det A
R2 + 6R1 → R2 A2 det A2 = det A1
−3R2 → R2 A3 det A3 = −3 det A2
R3 − 7R1 → R3 A4 det A4 = det A3
4 4
R
5 3
→ R3 B det B = det A4
5
Putting these together, we have
4 4
det B = det A4 Since det B = det A4
5 5
296 Chapter 6. Determinants
4
= det A3 Since det A4 = det A3
5
4
= (−3 det A2 ) Since det A3 = −3 det A2
5
12
= − det A1 Since det A2 = det A1
5
12
= − (− det A) Since det A1 = − det A
5
12 12
So det B = 5
det A. Dividing both sides of this latter equation by 5
, we get
5
det A = det B
12
5
(b) If det B = −60, then det A = 12
(−60) = −25.
5 −10 10
(a) We want to compute −2 8 1 by using elementary row operations. The
3 −10 4
idea is to transform A into an echelon matrix.
5 −10 10 1 −2 2
1
A = −2 8 1 R
5 1
→ R1 −2 8 1 = A1
3 −10 4 3 −10 4
1 −2 2
R2 + 2R1 → R2 0 4 5 = A2
R3 − 3R1 → R3
0 −4 −2
1 −2 2
R3 + R2 → R3 0 4 5 = A3
0 0 3
So
1
det A3 = det A2 = det A1 = det A
5
This implies that det A3 = 15 det A. Since A3 is a triangular matrix, det A3 is the
product of the entries on the main diagonal, that is, det A3 = (1)(4)(3) = 12.
Therefore, the equation det A3 = 15 det A becomes 51 det A = 12. Multiplying
both sides of this latter equation by 5, we get det A = 60.
(b) We transform A into an echelon matrix.
0 −5 −3 −8 1 −1 2 3
−1 4 0 3 −1 4 0 3
A= 1 −1
R1 ↔ R3 0 −5 −3 −8 = A1
2 3
2 2 4 13 2 2 4 13
6.2. Properties of Determinants 297
1 −1 2 3
R2 + R1 → R2 0
3 2 6
= A2
R4 − 2R1 → R4 0 −5 −3 −8
0 4 0 7
1 −1 2 3
0 2
1
1 3
2
R → R2 = A3
3 2
0 −5 −3 −8
0 4 0 7
1 −1 2 3
0 2
R3 + 5R2 → R3 1 3
2
= A4
R4 − 4R2 → R4 1
0 0 3
2
8
0 0 −3 −1
1 −1 2 3
0 1 23 2
R4 + 8R3 → R4 = A5
0 13
0 2
0 0 0 15
So
1 1 1 1
det A5 = det A4 = det A3 = det A2 = det A1 = (− det A) = − det A
3 3 3 3
1
(15) = 5, it follows that 5 = − 13 det A. This implies
Since det A5 = (1)(1) 3
that det A = −15.
0 1 4 −2
−3 7 −1 −5
4. Let A = . Compute det A. [Exercise on page 57]
0 2 3 −2
3 −7 6 8
0 1 4 −2
−3 7 −1 −5
To create more zeros in A = , we perform the operation R4 +
0 2 3 −2
3 −7 6 8
R2 → R4 . We obtain
0 1 4 −2
−3 7 −1 −5
=B
0 2 3 −2
0 0 5 3
Since the operation R4 + R2 → R4 does not affect the determinant, it follows that
det A = det B. We now compute the determinant of B by expanding down column
298 Chapter 6. Determinants
1.
0 1 4 −2
+
−3 7 −1 −5 1 4 −2
−
det B = = (−)(−3) 2 3 −2
0 2 3 −2
+ 0 5 3
0 0 5 3
−
1 4 −2
+
2 3 −2 Select column 1 again
=3 −
and add signs
0 5 3
+
3 −2 4 −2
= 3 (+)(1) + (−)(2) +0 Expand down column 1
5 3 5 3
= 3 (1(9 + 10) − 2(12 + 10)) = 3(19 − 44) = −75
In general we have the following result, which follows from Theorem 6.2.5 and
the Invertible Matrix Theorem.
6.2. Properties of Determinants 299
Theorem 6.2.4. Let A be a square matrix. Then det A = 0 if and only if the
columns (or rows) of A are linearly dependent.
5 −10 10
(a) Since the matrix A = 0 0 0 has a zero row (the second row), it follows
3 −10 4
that det A = 0.
4 −11 0
(b) Since the matrix A = −7 9 0 has a zero column (the third column),
11 −10 0
it follows that det A = 0.
0 1 4 1
−3 7 −1 7
(c) Since the second and fourth columns of A =
0
are identical,
2 3 2
3 −7 6 −7
it follows that det A = 0
(d) Observe that the third row of A is a multiple of the first row. Using that we can
create a zero row:
0 −1 4 −2 0 −1 4 −2
−3 7 −1 −5 R3 − 3R1 → R3 −3 7 −1 −5
A= =B
0 3 −12 6 0 0 0 0
3 −7 6 8 3 −7 6 8
Since det B = 0 (because B has a zero row) and since det A = det B (because
the operation R3 − 3R1 → R3 does not affect the determinant), it follows that
det A = 0.
Alternate Solution
Since the third row of A is a multiple of the first row, it follows that the rows
of A are linearly dependent. So, by Theorem 6.2.4, det A = 0.
To find det B, we will use elementary row operations. The idea is to carry B to
A:
2a 2b 2c a b c
1
B = d−a e−b f −c R → R1 d − a e − b f − c = B1
2 1
g + 3a h + 3b i + 3c g + 3a h + 3b i + 3c
a b c
R2 + R1 → R2
d e f =A
R3 − 3R1 → R3
g h i
300 Chapter 6. Determinants
So
1 1
det A = − det C2 = − det C1 = − det C = − det C.
3 3
This gives the equation det A = − 13 det C. Since det A = 7, it follows that
2 1 4
+
0 9 3 9 3 1 4
det A = −
= (+)(2) − 0 + (+)(1)
2 3 9 3
1 2 3
+
1 0 3 2
−1 5 1 −8
(b) Let A = 2
be the matrix whose columns are v1 , v2 , v3 , and
4 3 12
−3 −8 −3 −22
v4 . We need to find the determinant of A.
1 0 3 2 1 0 3 2
−1 R2 + R1 → R2
5 1 −8 0 5 4 −6
A= R3 − 2R1 → R3 = A1
2 4 3 12 0 4 −3 8
R4 + 3R1 → R4
−3 −8 −3 −22 0 −8 6 −16
1 0 3 2
0 5 4 −6
R4 + 2R3 → R4
0 4 −3
= A2
8
0 0 0 0
k 3
det A = (+)(1) = k(k + 1) − 6 = k 2 + k − 6 = (k − 2)(k + 3)
2 k+1
So det A = 0 if and only if k = 2 or k = −3. This implies (by Theorem 6.2.5) that A
is invertible if and only if k 6= 2 and k 6= −3.
(c)
(d)
det A2 = det A
Here we used the fact that det(−A) = (−1)3 det A. In general, we have the
following.
The Determinant of kA
Fact. Let A be an n × n matrix. Then one can prove that for every real
number k,
det(kA) = k n det A
1 0 −2
(n) False. Indeed, let A = 0 1 −4. The determinant of A is 0 since A has a
0 0 0
row of zeros. But the statement “for every b in R3 the system Ax = 0 has no
304 Chapter 6. Determinants
0
solution” is false. Indeed, if b = 0 , the system Ax = b has infinitely many
0
solutions (x1 = 2s, x2 = 4s, x3 = s, where s can be any real number).
Chapter7
Definition
Let A be an n × n matrix. A vector x in Rn is called an eigenvector of
A if x 6= 0 and if there exists a scalar λ such that
Ax = λx
305
306 Chapter 7. Eigenvectors and Eigenvalues
3 −3
First, we form the matrix A − λI, where A = , λ = 5, and I =
−2 2
1 0
0 1
3 −3 1 0 3 − 5 −3 − 0 −2 −3
A − λI = −5 = = .
−2 2 0 1 −2 − 0 2−5 −2 −3
7.1. Eigenvectors and Eigenvalues 307
Eigenspaces
Let A be a square matrix, and let λ be an eigenvalue of A. The eigenspace
of A corresponding to λ is the set of all solutions to the homogeneous equation
(A − λI)x = 0. In other words, the eigenspace of A corresponding to λ is the
null space of the matrix A − λI, that is, Null(A − λI).
Note. Every vector in the eigenspace, except the zero vector, is an eigenvector
of A.
3 −3
(a) A = , λ = 5.
−2 2
3 −3
In Exercise 3 we considered the matrix A = and the eigenvalue
−2 2
λ = 5. And we showed that the solutions to the equation (A − 5I)x = 0
set3 of
−2
is the set of multiples of v = . This implies that the eigenspace of A,
1
Null(A − 5I), is the set of vectors of the form sv, where s is a real number.
So the set B = {v} spans Null(A − 5I). Moreover, since v 6= 0, B is linearly
independent. Thus, B is a basis for Null(A − 5I).
(b) First, we form the matrix A − (−1)I = A + I:
0 1 0 1 0 0 1 1 0
A+I = 3 0 1 + 0 1 0 = 3 1 1
2 0 0 0 0 1 2 0 1
308 Chapter 7. Eigenvectors and Eigenvalues
A(Ax) =A(λx)
=λAx
=λ(λx) = λ2 x
Proof by Induction
Step 2. Assume that P(n − 1) is true and prove that P(n) is also true.
Here P(n) is
λn is an eigenvalue of An associated with u
AAn−1 u = A(λn−1 u)
An = λn−1 (Au)
An = λn−1 (λu) Since Au = λu
An = λn u
So P(n) is true.
310 Chapter 7. Eigenvectors and Eigenvalues
This proves that λ−1 is an eigenvalue of A−1 with the same eigenvector x.
8. [Exercise on page 62]
Consider the following figure.
x2
x2 = x1
u v
x1
x2 = −x1
Geometrically, if x is vector that lies along the line x2 = −x1 , then T (x) = x
(since T is the reflection through the line x2 = −x1 ). So every nonzero vector
on the line x2 = −x1 is an eigenvector and the associated eigenvalue is λ = 1.
Again from the figure, if x is perpendicular to u, then T (x) = −x. (For example,
T (v) = −v.) So every nonzero vector on the line x2 = x1 is an eigenvector and
the associated eigenvalue is λ = −1.
If x is neither on the line x2 = −x1 nor on the line x2 = x1 , then T (x) is not a
multiple of x.
Au − λIu = 0
(A − λI) u = 0 Right-factor out u
(A − λI)u = 0
Au − λu = 0
Au = λu
312 Chapter 7. Eigenvectors and Eigenvalues
5 4
(a) To determine if λ = 2 is an eigenvalue of A = , we need to find
−6 −6
det(A − λI). First, we form the matrix A − λI:
5 4 1 0 5 4 2 0
A − λI = A − 2I = −2 = −
−6 −6 0 1 −6 −6 0 2
5−2 4 3 4
= =
−6 −6 − 2 −6 −8
3 4
det(A − 2I) = = (3)(−8) − (4)(−6) = −24 + 24 = 0
−6 −8
3 2
det(A − (−4)I) = =9−4=5
2 3
0−7 3 0
det(A − λI) = det(A − 7I) = 3 0−7 0
4 −5 7 − 7
−7 3 0
= 3 −7 0 =0 Expand down column 3
4 −5 0
5 − λ −3
det(A − λI) = = (5 − λ)(6 − λ) − (−3)(−10)
−10 6 − λ
= 30 − 5λ − 6λ + λ2 − 30 = λ2 − 11λ = λ(λ − 11)
4 7 −5
Since A = 0 −2 5 is a triangular matrix, the eigenvalues are the diag-
0 0 3
onal entries, that is, 4, −2, and 3.
3 0 0
(c) Since A = −1 2 0 is a triangular matrix, the eigenvalues are the entries
8 5 3
on the main diagonal of A, that is, 2 and 3.
(d) First, we find A − λI:
0 4 0 1 0 0 −λ 4 0
A − λI = 1 0 0 − λ 0 1 0 = 1 −λ 0
0 0 1 0 0 1 0 0 1−λ
−λ 4 0
1 −λ 0 −λ 4
det(A − λI) = = 0 − 0 + (+)(1 − λ)
0 0 1−λ 1 −λ
+ − +
= (1 − λ) λ2 − 4 = (1 − λ)(λ − 2)(λ + 2)
(c) Let λ1 and λ2 be the eigenvalues of A. If λ1 = 3 and tr(A) = −5, then (by
part (b)) λ1 + λ2 = −5, that is, 3 + λ2 = −5. Subtracting 3 from both sides of
the latter equation, we find λ2 = −8.
(d) If the characteristic polynomial of A is λ2 − λ, then det(A − λI) = λ2 − λ + 0.
This implies that det A = 0 since the determinant of A is the constant term of
the characteristic polynomial (by part (a)). So A is not invertible.
Symmetric Matrices
Let A be a square matrix. We say that A is a symmetric matrix if it is equal
to its transpose, that is, if A = AT .
a b
Let A = . Suppose A is symmetric. Then A = AT , that is
c d
a b a c
=
c d b d
a b
This implies that b = c. So A = . We want to show that all the eigenvalues of
b d
A are real numbers. First, the characteristic polynomial of A is
a−λ b
det(A − λI) =
b d−λ
= (a − λ)(d − λ) − b2 = ad − (a + d)λ + λ2 − b2
= λ2 − (a + d)λ + ad − b2
Using the quadratic formula, the solutions to the equation det(A − λI) = 0 are
p √
(a + d) ± (a + d)2 − 4(ad − b2 ) (a + d) ± a2 + 2ad + d2 − 4ad + 4b2
=
2 2
√
(a + d) ± a − 2ad + d2 + 4b2
2
=
2
p
(a + d) ± (a − d)2 + 4b2
=
2
p
Since (a − d)2 + 4b2 is always greater than or equal to 0, (a − d)2 + 4b2 is always
a real number. This implies that the solutions to the equation det(A − λI) = 0 are
real numbers. In other words, the eigenvalues of A are real numbers.
entries; that is, 2 and 3. So the eigenvalues of A are real numbers. But A is not a
symmetric matrix since AT 6= A.
Multiplicity of an Eigenvalue
Let A be a square matrix and let λ be an eigenvalue of A. The multiplicity
of λ is m if λ occurs m times as a root of det(A − λI). For example, if
det(A − λI) = (λ − 1)3 (λ − 2)5 , then the eigenvalue 1 has multiplicity 3 and
the eigenvalue 2 has multiplicity 5.
7.3 Diagonalization
1. [Exercise on page 64]
Diagonal Matrices
A square matrix is called diagonal if all its non-diagonal entries are 0. It is
very easy to do computations with diagonal matrices.
a11 0 ··· 0
0 a22 · · · 0
Theorem 7.3.1. Let D = .. .. be a diagonal matrix. Then
.. . .
. . . .
0 0 · · · ann
for every integer k ≥ 1, we have
ak11 0 ··· 0
0 ak · · · 0
k 22
D = ..
.. . . ..
. . . .
k
0 0 · · · ann
Diagonalization – Definition
Let A be an n × n matrix. We say that A is diagonalizable if there exist a
diagonal matrix D and an invertible matrix P such that
A = P DP −1
(a) False because there is a nonzero entry off the main diagonal (the (2, 1)-entry,
4, is nonzero).
(b) True because all the entries of D off the main diagonal are zeros.
(c) True by Theorem 7.3.1.
(d) True by definition.
(e) False since only square matrices can be diagonalizable.
(f ) True. Indeed, suppose A is a diagonal matrix and let D = A. Let P = I be the
identity matrix of the same size as A. Since I −1 = I, it follows that
P DP −1 = IDI = D = A
So A is diagonalizable.
Multiplicity of an Eigenvalue
Let A be a square matrix and let λ be an eigenvalue of A. The multiplicity
of λ is m if λ occurs m times as a root of det(A − λI). For example, if
det(A − λI) = (λ − 2)5 (λ − 3)6 (λ − 4), then the eigenvalue 2 has multiplicity
5, the eigenvalue 3 has multiplicity 6, and the eigenvalue 4 has multiplicity 1.
Diagonalization–Theorem
To determine whether a matrix is diagonalizable, we will use the following
theorem.
Theorem 7.3.2. Let A be a square matrix that has only real eigenvalues. Then
A is diagonalizable if and only if for each eigenvalue λ,
9 6
(a) A =
−7 −4
9 6
We want to diagonalize A = , if possible.
−7 −4
Step 1. Find the characteristic polynomial and the eigenvalues.
The characteristic polynomial is
9−λ 6
det(A − λI) = = (9 − λ)(−4 − λ) − (6)(−7)
−7 −4 − λ
= −36 − 9λ + 4λ + λ2 + 42 = λ2 − 5λ + 6 = (λ − 2)(λ − 3)
" #
1
1 76 0
R
7 1
→ R1
0 0 0
−1
where v2 = . So a basis for Null(A − 3I) is B2 = {v2 }, and
1
dim Null(A − 3I) = 1.
Step 3. Check if condition (7.3.1) from Theorem 7.3.2 is satisfied
for each λ, and decide whether A is diagonalizable. For every eigen-
value λ, we need to check if its multiplicity is equal to the dimension of the
corresponding eigenspace. Recall the characteristic polynomial:
det(A − λI) = (λ − 2)(λ − 3) = (λ − 2)1 (λ − 3)1
– For λ1 = 2, the multiplicity is m1 = 1. From Step 2, dim Null(A−I) = 1.
So condition (7.3.1) is satisfied for λ1 .
– Clearly, condition (7.3.1) is also satisfied for λ2 = 3.
Thus, A is diagonalizable.
2 −3
(b) A =
3 8
Finding the eigenvalues. The characteristic polynomial is
2 − λ −3
det(A − λI) = = (2 − λ)(8 − λ) − (−3)(3)
3 8−λ
= 16 − 2λ − 8λ + λ2 + 9 = λ2 − 10λ + 25 = (λ − 5)2
− 13 R1 → R1
−3 −3 0 1 1 0
3 3 0 1 1 1 0
R → R2
3 2
1 1 0
R2 − R1 → R2
0 0 0
−2 − λ 5 5
det(A − λI) = 0 3−λ 0
2 −2 1 − λ
−2 − λ 5
= (3 − λ) Expand across row 2
2 1−λ
= (3 − λ) [(−2 − λ)(1 − λ) − (5)(2)]
= (3 − λ)(−2 + 2λ − λ + λ2 − 10)
7.3. Diagonalization 323
= (3 − λ)(λ2 + λ − 12)
= (3 − λ)(λ − 3)(λ + 4) = −(λ − 3)2 (λ + 4)
So the eigenvalues of A are λ1 = 3 and λ2 = −4.
Finding the eigenspaces.
– Finding the eigenspace corresponding to the eigenvalue λ1 = 3. First,
we need to form the matrix A − 3I:
−2 5 5 1 0 0 −5 5 5
A − 3I = 0 3 0 − 3 0 1 0 = 0 0 0
2 −2 1 0 0 1 2 −2 −2
Then we need to solve the system (A − 3I)x = 0. Row reducing the
augmented matrix, we have
−5 5 5 0 1
− 5 R1 → R1 1 −1 −1 0
0 0 0 0 0 0 0 0
1
2 −2 −2 0 2
R 3 → R3 1 −1 −1 0
1 −1 −1 0
R3 − R1 → R3 0 0 0 0
0 0 0 0
The corresponding system is x1 − x2 − x3 = 0. Set x2 = s and x3 = t.
Then x1 = s + t, and therefore the general solution is
x1 s+t 1 1
x = x2 =
s = s 1 + t 0 = sv1 + tv2 ,
x3 t 0 1
1 1
where v1 = 1 and v2 = 0 . So a basis for Null(A − 3I) is
0 1
B1 = {v1 , v2 }.
– Finding the eigenspace corresponding to the eigenvalue λ2 = −4. First,
we have
−2 5 5 1 0 0 2 5 5
A−(−4)I = A+4I = 0 3 0 +4 0 1 0 = 0 7 0
2 −2 1 0 0 1 2 −2 5
Now, we solve the system (A + 4I)x = 0. Row reducing the augmented
matrix, we have
2 5 5 0 1 2 5 5 0
0 → R2
R
7 2
7 0 0 0 1 0 0
R3 − R1 → R3
2 −2 5 0 0 −7 0 0
2 5 5 0
R3 + 7R2 → R3 0 1 0 0
0 0 0 0
324 Chapter 7. Eigenvectors and Eigenvalues
1 25 25
0
1
R
2 1
→ R1 0 1 0 0
0 0 0 0
1 0 25
0
R1 − 52 R2 → R1 0 1 0 0
0 0 0 0
− 52
1 −3 −1 0
(−1)R1 → R1 0 0 0 0
0 0 0 0
The corresponding system is x1 − 3x2 − x3 = 0. Set x2 = s and x3 = t.
Then x1 = 3s + t, and the general solution is
x1 3s + t 3 1
x = x2 = s = s 1 + t 0 = sv2 + tv3 ,
x3 t 0 1
3 1
where v2 = 1 and v3 = 0 . So a basis for Null(A − I) is
0 1
B2 = {v2 , v3 }. This implies that dim Null(A − I) = 2.
Since for each eigenvalue λ the dimension of the corresponding eigenspace
is equal to the multiplicity of λ, it follows that A is diagonalizable.
The matrices P and D are
1 3 1
P = v1 v2 v3 = −1 1 0
3 0 1
λ1 0 0 0 0 0
D = 0 λ2 0 = 0 1 0
0 0 λ2 0 0 1
2 3 0
(e) A = 3 2
0
2 3 −1
Finding the eigenvalues of A.
2−λ 3 0
det(A − λI) = 3 2−λ 0
2 3 −1 − λ
2−λ 3
= (−1 − λ) Expand down column 3
3 2−λ
= (−1 − λ) (2 − λ)2 − 9
This implies that A is not diagonalizable. (We don’t need to find the
eigenspace corresponding to λ2 = 5 since A is not diagonalizable.)
Since A is not diagonalizable, it is not possible to find P and D such that
A = P DP −1 .
−1 − λ 2 −2
det(A − λI) = 0 1−λ 0
− + −
4 −4 5 − λ
−1 − λ −2
=(1 − λ)
4 5−λ
=(1 − λ) [(−1 − λ)(5 − λ) − (−2)(4)]
328 Chapter 7. Eigenvectors and Eigenvalues
=(1 − λ)(−5 + λ − 5λ + λ2 + 8)
=(1 − λ)(λ2 − 4λ + 3)
=(1 − λ)(λ − 1)(λ − 3) = −(λ − 1)2 (λ − 3)
−2 2 −2 0 − 21 R1 → R1 1 −1 1 0
0 0 0 0 0 0 0 0
1
4 −4 4 0 4 3
R → R3 1 −1 1 0
1 −1 1 0
R3 − R1 → R3 0 0 0 0
0 0 0 0
The corresponding system is x1 − x2 + x3 = 0. Set x2 = s and x3 = t.
Then x1 = s − t, and the general solution is
x1 s−t 1 −1
x = x2 = s = s 1 + t 0 = sv1 + tv2 ,
x3 t 0 1
1 −1
where v1 = 1 and v2 = 0 . So a basis for Null(A − I) is
0 1
{v1 , v2 }. This implies that dim Null(A − I) = 2.
– Finding the eigenspace corresponding to λ2 = 3. The matrix A − 3I is
−1 2 −2 1 0 0 −4 2 −2
A − 3I = 0 1 0 − 3 0 1 0 = 0 −2 0
4 −4 5 0 0 1 4 −4 2
1 − 12 21
0
− 14 R1 → R1 0 1 0 0
0 0 0 0
1 0 21
0
R1 + 21 R2 → R1 0 1 0 0
0 0 0 0
Powers of a Matrix
Theorem 7.3.4. Let P be an invertible matrix and let
a11 0 · · · 0
0 a22 · · · 0
D = ..
..
. .
0 0 · · · ann
Dk is the diagonal matrix whose ith diagonal entry is akii , that is,
ak11 0 · · · 0
0 ak · · · 0
k 22
D = .. and
..
. .
0 0 · · · aknn
330 Chapter 7. Eigenvectors and Eigenvalues
k
(P DP −1 ) = P Dk P −1 .
Ak = P Dk P −1 for every k ≥ 1
The product P D is
4 −1 1 0 4 2
PD = =
5 4 0 −2 5 −8
The matrix A is
" 4 1
# " 2 4
#
4 2 21 21 7 7
A = P DP −1 = =
5 −8 −5 4 20 −9
21 21 7 7
Dimension of Eigenspaces
Theorem 7.3.5. Let A be a matrix. Let λ be an eigenvalue of A with multi-
plicity m. Then the dimension of the eigenspace corresponding to λ is greater
than or equal to 1 and less than or equal to m. That is,
Let A be a 4 × 4 matrix with two distinct eigenvalues, say λ1 and λ2 . Suppose that
each eigenspace is two-dimensional. We want to determine if A is diagonalizable. Let
m1 (respectively m2 ) be the multiplicity of λ1 (respectively λ2 ). Then, since each
eigenspace is two-dimensional,
dim Null(A − λ1 I) = 2 and dim Null(A − λ2 I) = 2
Using Theorem 7.3.5, we have
1 ≤ 2 ≤ m1 and 1 ≤ 2 ≤ m2
So
m1 ≥ 2 and m2 ≥ 2 (7.3.2)
On the other hand, the degree of the characteristic polynomial is 4 since A is a 4 × 4
matrix. Since the sum of the multiplicities is equal to the degree of the characteristic
polynomial, it follows that
m1 + m2 = 4 (7.3.3)
Combining (7.3.2) and (7.3.3), we get m1 = 2 and m2 = 2. So
dim Null(A − λ1 I) = m1 and dim Null(A − λ2 I) = m2
This implies that A is diagonalizable.
7. [Exercise on page 65]
(a) True by Theorem 7.3.3.
(b) True. This is true because of the following theorem.
Eigenvector Bases of Rn
Theorem 7.3.6. Let A be an n × n matrix. Then A is diagonalizable if
and only if Rn has a basis of eigenvectors of A.
(c) False because the dimension of Null(A − λ1 I) is not equal to the multiplicity of
λ1 .
(d) True. Indeed, suppose that A is invertible and diagonalizable. Then, by def-
inition, there exist an invertible matrix P and a diagonal matrix D such that
A = P DP −1 . Taking the inverse of both sides of this latter equation, we get
−1
A−1 = P DP −1
−1 −1 −1
A−1 = P −1 D P (ABC)−1 = C −1 B −1 A−1
A−1 = P D−1 P −1 (P −1 )−1 = P
A−1 = P QP −1 Where Q = D−1
Since the inverse of a diagonal matrix is still a diagonal matrix, it follows that Q
is a diagonal matrix. Therefore, the equation A−1 = P QP −1 implies that A−1
is diagonalizable.
7.3. Diagonalization 333
2 −3
(e) True. For example, the matrix A = from Exercise 2b is invertible
3 8
(since det A = 16 + 9 = 25 6= 0), but not diagonalizable.
1 0
(f ) True. Indeed, let A = . The matrix A is a diagonal matrix. So it is
0 0
diagonalizable (in Exercise 1f–page 318, we proved that any diagonal matrix is
diagonalizable). But A is not invertible since det A = 0.
(g) True. Indeed, suppose that A is diagonalizable. Then, by definition, there exist
an invertible matrix P and a diagonal matrix D such that A = P DP −1 . Taking
the transpose of both sides of this latter equation, we get
T
AT = P DP −1
T
AT = P −1 DT P T (ABC)T = C T B T AT
T −1
h i−1
T −1
−1 T
T T
PT = P −1
A = P D P
−1
AT = RQR−1 Where R = P T and Q = DT
Since the transpose of a diagonal matrix is still a diagonal matrix, it follows that
−1
Q is a diagonal matrix. Since P is invertible, it follows that P T = R is also
invertible. Thus, the equation AT = RQR−1 implies that AT is diagonalizable.